1 Crore+ students have signed up on EduRev. Have you? |
Read the given passage and answer the question that follows.
Employment exchanges - one of the surviving bastions of babudom - face the prospect of becoming irrelevant in an era of reform. Even in the heart of the nation’s capital, the premises are often dilapidated structures with dirty passages and manned by surly staff. Not surprisingly, job-seekers hardly throng these exchanges. Paradoxically, when jobs are getting scarce due to pressure of liberalisation, job-seekers are spurning an institution intended to help them secure placements. The reasons are simply enough. Employment exchanges still concentrate on government and public sector placements, which are fast losing ground in the labour market. For most government jobs, the eligibility criterion is still registration with the employment exchanges. But what is the use of going through the formalities of registration when government jobs themselves are dwindling? The placement effected by all the 939-odd exchanges in the country in 2001 was of the order of 1.69 lakh against annual registration levels of 60 lakh. As there are too few jobs when compared to the number of job-seekers, the accumulated backlog of registrations is close to 4.16 crore. The latter of course doesn’t indicate unemployment levels as those registered with the employment exchanges are not necessarily unemployed.
How can the employment exchanges be revamped? The thinking in the Union labour ministry is to transform them into employment promotion and guidance centres. The plan includes modernisation, changing the mindset of the staff and making them into an effective instrument for monitoring and coordinating various employment generation schemes. This objective calls for developing a better database on the fast changing employment situation with a comprehensive coverage of new economic establishments. For instance, the various economic censuses are an important source of information on the changing employment profile of, say, the nation’s capital. Far from being a bureaucrat-dominated city, Delhi over the years has become more of an industrial metropolis. According to the fourth economic census, manufacturing accounted for 40 per cent of jobs in the capital. The employment exchanges in the capital thus have their work cut out notably, to shift the focus away from government and public sector jobs more towards placements in the private sector, especially in manufacturing and services, including the burgeoning retail trade sector. By doing so, they will better reflect the imperatives of economic reform and remain relevant in today’s times.
Q. Which of the revamped role can be entrusted to employment exchanges?
Read the given passage and answer the question that follows.
Employment exchanges - one of the surviving bastions of babudom - face the prospect of becoming irrelevant in an era of reform. Even in the heart of the nation’s capital, the premises are often dilapidated structures with dirty passages and manned by surly staff. Not surprisingly, job-seekers hardly throng these exchanges. Paradoxically, when jobs are getting scarce due to pressure of liberalisation, job-seekers are spurning an institution intended to help them secure placements. The reasons are simply enough. Employment exchanges still concentrate on government and public sector placements, which are fast losing ground in the labour market. For most government jobs, the eligibility criterion is still registration with the employment exchanges. But what is the use of going through the formalities of registration when government jobs themselves are dwindling? The placement effected by all the 939-odd exchanges in the country in 2001 was of the order of 1.69 lakh against annual registration levels of 60 lakh. As there are too few jobs when compared to the number of job-seekers, the accumulated backlog of registrations is close to 4.16 crore. The latter of course doesn’t indicate unemployment levels as those registered with the employment exchanges are not necessarily unemployed.
How can the employment exchanges be revamped? The thinking in the Union labour ministry is to transform them into employment promotion and guidance centres. The plan includes modernisation, changing the mindset of the staff and making them into an effective instrument for monitoring and coordinating various employment generation schemes. This objective calls for developing a better database on the fast changing employment situation with a comprehensive coverage of new economic establishments. For instance, the various economic censuses are an important source of information on the changing employment profile of, say, the nation’s capital. Far from being a bureaucrat-dominated city, Delhi over the years has become more of an industrial metropolis. According to the fourth economic census, manufacturing accounted for 40 per cent of jobs in the capital. The employment exchanges in the capital thus have their work cut out notably, to shift the focus away from government and public sector jobs more towards placements in the private sector, especially in manufacturing and services, including the burgeoning retail trade sector. By doing so, they will better reflect the imperatives of economic reform and remain relevant in today’s times.
Q. Choose the word that is same in meaning as the word “secure” as used in the passage.
Read the given passage and answer the question that follows.
Employment exchanges - one of the surviving bastions of babudom - face the prospect of becoming irrelevant in an era of reform. Even in the heart of the nation’s capital, the premises are often dilapidated structures with dirty passages and manned by surly staff. Not surprisingly, job-seekers hardly throng these exchanges. Paradoxically, when jobs are getting scarce due to pressure of liberalisation, job-seekers are spurning an institution intended to help them secure placements. The reasons are simply enough. Employment exchanges still concentrate on government and public sector placements, which are fast losing ground in the labour market. For most government jobs, the eligibility criterion is still registration with the employment exchanges. But what is the use of going through the formalities of registration when government jobs themselves are dwindling? The placement effected by all the 939-odd exchanges in the country in 2001 was of the order of 1.69 lakh against annual registration levels of 60 lakh. As there are too few jobs when compared to the number of job-seekers, the accumulated backlog of registrations is close to 4.16 crore. The latter of course doesn’t indicate unemployment levels as those registered with the employment exchanges are not necessarily unemployed.
How can the employment exchanges be revamped? The thinking in the Union labour ministry is to transform them into employment promotion and guidance centres. The plan includes modernisation, changing the mindset of the staff and making them into an effective instrument for monitoring and coordinating various employment generation schemes. This objective calls for developing a better database on the fast changing employment situation with a comprehensive coverage of new economic establishments. For instance, the various economic censuses are an important source of information on the changing employment profile of, say, the nation’s capital. Far from being a bureaucrat-dominated city, Delhi over the years has become more of an industrial metropolis. According to the fourth economic census, manufacturing accounted for 40 per cent of jobs in the capital. The employment exchanges in the capital thus have their work cut out notably, to shift the focus away from government and public sector jobs more towards placements in the private sector, especially in manufacturing and services, including the burgeoning retail trade sector. By doing so, they will better reflect the imperatives of economic reform and remain relevant in today’s times.
Q. Which of the following is not true in the context of the passage?
Read the given passage and answer the question that follows.
In the second week of August 1998, just a few days after the incidents of bombing the US embassies in Nairobi and Dar-es-Salaam, a high-powered, brain-storming session was held near Washington D.C. to discuss various aspects of terrorism. The meeting was attended by ten of America’s leading experts in various fields such as germ and chemical warfare, public health, disease control and also by the doctors and the law-enforcing officers. Being asked to describe the horror of possible bio-attack, one of the experts narrated the following gloomy scenario.
A culprit in a crowded business centre or in a busy shopping mall of a town empties a test tube containing some fluid, which in turn creates an unseen cloud of germ of a dreaded disease like anthrax capable of inflicting a horrible death within 5 days on any one who inhales it. At first 500 or so victims feel that they have mild influenza which may recede after a day or two. Then the symptoms return again and their lungs start filling with fluid. They rush to local hospitals for treatment, but the panic-stricken people may find that the medicare services run quickly out of drugs due to excessive demand. But no one would be able to realise that a terrorist attack has occurred. One cannot deny the possibility that the germ involved would be of contagious variety capable of causing an epidemic. The meeting concluded that such attacks, apart from causing immediate human tragedy, would have dire long-term effects on the political and social fabric of a country by way of ending people’s trust on the competence of the government.
The experts also said that the bombs used in Kenya and Tanzania were of the old-fashioned variety and involved quantities of high explosives, but new terrorism will prove to be more deadly and probably more elusive than hijacking an aeroplane or a gelignite of previous decades. According to Bruce Hoffman, an American specialist on political violence, old terrorism generally had a specific manifesto - to overthrow a colonial power or the capitalist system and so on. These terrorists were not shy about planting a bomb or hijacking an aircraft and they set some limit to their brutality. Killing so many innocent people might turn their natural supporters off. Political terrorists want a lot of people watching but not a lot of people dead. “Old terrorism sought to change the world while the new sort is often practised by those who believe that the world has gone beyond redemption”, he added.
Hoffman says, “New terrorism has no long-term agenda but is ruthless sin its short-term intentions. It is often just a cacophonous cry of protest or an outburst of religious intolerance or a protest against the West in general and the US in particular. Its perpetrators may be religious fanatics or diehard opponents of a government and see no reason to show restraint. They are simply intent on inflicting the maximum amount of pain on the victim.”
Q. According to the author of the passage, the root cause of terrorism is
(A) Religious fanaticism
(B) Socio-political changes in countries
(C) The enormous population growth
‘Religious intolerance’ as cited in the last paragraph stands behind terrorism.
Read the given passage and answer the question that follows.
In the second week of August 1998, just a few days after the incidents of bombing the US embassies in Nairobi and Dar-es-Salaam, a high-powered, brain-storming session was held near Washington D.C. to discuss various aspects of terrorism. The meeting was attended by ten of America’s leading experts in various fields such as germ and chemical warfare, public health, disease control and also by the doctors and the law-enforcing officers. Being asked to describe the horror of possible bio-attack, one of the experts narrated the following gloomy scenario.
A culprit in a crowded business centre or in a busy shopping mall of a town empties a test tube containing some fluid, which in turn creates an unseen cloud of germ of a dreaded disease like anthrax capable of inflicting a horrible death within 5 days on any one who inhales it. At first 500 or so victims feel that they have mild influenza which may recede after a day or two. Then the symptoms return again and their lungs start filling with fluid. They rush to local hospitals for treatment, but the panic-stricken people may find that the medicare services run quickly out of drugs due to excessive demand. But no one would be able to realise that a terrorist attack has occurred. One cannot deny the possibility that the germ involved would be of contagious variety capable of causing an epidemic. The meeting concluded that such attacks, apart from causing immediate human tragedy, would have dire long-term effects on the political and social fabric of a country by way of ending people’s trust on the competence of the government.
The experts also said that the bombs used in Kenya and Tanzania were of the old-fashioned variety and involved quantities of high explosives, but new terrorism will prove to be more deadly and probably more elusive than hijacking an aeroplane or a gelignite of previous decades. According to Bruce Hoffman, an American specialist on political violence, old terrorism generally had a specific manifesto - to overthrow a colonial power or the capitalist system and so on. These terrorists were not shy about planting a bomb or hijacking an aircraft and they set some limit to their brutality. Killing so many innocent people might turn their natural supporters off. Political terrorists want a lot of people watching but not a lot of people dead. “Old terrorism sought to change the world while the new sort is often practised by those who believe that the world has gone beyond redemption”, he added.
Hoffman says, “New terrorism has no long-term agenda but is ruthless sin its short-term intentions. It is often just a cacophonous cry of protest or an outburst of religious intolerance or a protest against the West in general and the US in particular. Its perpetrators may be religious fanatics or diehard opponents of a government and see no reason to show restraint. They are simply intent on inflicting the maximum amount of pain on the victim.”
Q. In what way would the new terrorism be different from that of the earlier years?
(A) More dangerous and less baffling
(B) More hazardous for victims
(C) Less complicated for terrorists
“New terrorism has no long-term agenda but is ruthless in its short-term intentions”. This statement from the passage supports (C). While, in the light of passage, (B) also seem suitable.
Read the given passage and answer the question that follows.
For many women, including myself, "wife" can feel like a loaded word. It carries a history. If you grew up in the 1960s and 1970s as I did, wives seemed to be a genus of white women who lived inside television sitcoms—cheery, coiffed, corseted. They stayed at home, fussed over the children, and had dinner ready on the stove. They sometimes got into the sherry or flirted with the vacuum-cleaner salesman, but the excitement seemed to end there. The irony, of course, was that I used to watch those shows in our living room on Euclid Avenue while my own stay-at-home mom fixed dinner without complaint and my own clean-cut dad recovered from a day at work. My parents' arrangement was as traditional as anything we saw on TV.
Personally, as a kid, I preferred The Mary Tyler Moore Show, which I absorbed with fascination. Mary had a job, a snappy wardrobe, and really great hair. She was independent and funny, and unlike those of the other ladies on TV, her problems were interesting. She had conversations that weren't about children or homemaking. She didn't let Lou Grant boss her around, and she wasn't fixated on finding a husband. She was youthful and at the same time grown-up. If you were a girl with a brain and a dawning sense that you wanted to grow into something more than a wife, Mary Tyler Moore was your goddess.
And here I was now, twenty-nine years old, sitting in the very same apartment where I'd watched all that TV and consumed all those meals dished up by the patient and selfless Marian Robinson. I had so much—an education, a healthy sense of self, a deep arsenal of ambition—and I was wise enough to credit my mother, in particular, with instilling it in me. She'd taught me how to read before I started kindergarten, helping me sound out words as I sat curled like a kitten in her lap, studying a library copy of Dick and Jane. She'd cooked for us with care, putting broccoli and Brussels sprouts on our plates and requiring that we eat them. The point was, she'd given diligently and she'd given everything.
My considerable blessings in life were now causing a kind of psychic whiplash. I'd been raised to be confident and see no limits, to believe I could go after and get absolutely anything I wanted. And I wanted everything. I wanted to live with the hat-tossing, independent-career-woman zest of Mary Tyler Moore, and at the same time I gravitated toward the stabilizing, self-sacrificing, seemingly bland normalcy of being a wife and mother.
Q. Why did the author suggest that 'wife' is a loaded word?
The author states in the beginning of the passage that she finds 'wife' to be a loaded word because it carries history. She then describes what she used to perceive what a 'wife' really was by watching the TV shows. So, she feels that there is some underlying historical meaning to the word. Therefore, the correct answer is option 3.
Read the given passage and answer the question that follows.
For many women, including myself, "wife" can feel like a loaded word. It carries a history. If you grew up in the 1960s and 1970s as I did, wives seemed to be a genus of white women who lived inside television sitcoms—cheery, coiffed, corseted. They stayed at home, fussed over the children, and had dinner ready on the stove. They sometimes got into the sherry or flirted with the vacuum-cleaner salesman, but the excitement seemed to end there. The irony, of course, was that I used to watch those shows in our living room on Euclid Avenue while my own stay-at-home mom fixed dinner without complaint and my own clean-cut dad recovered from a day at work. My parents' arrangement was as traditional as anything we saw on TV.
Personally, as a kid, I preferred The Mary Tyler Moore Show, which I absorbed with fascination. Mary had a job, a snappy wardrobe, and really great hair. She was independent and funny, and unlike those of the other ladies on TV, her problems were interesting. She had conversations that weren't about children or homemaking. She didn't let Lou Grant boss her around, and she wasn't fixated on finding a husband. She was youthful and at the same time grown-up. If you were a girl with a brain and a dawning sense that you wanted to grow into something more than a wife, Mary Tyler Moore was your goddess.
And here I was now, twenty-nine years old, sitting in the very same apartment where I'd watched all that TV and consumed all those meals dished up by the patient and selfless Marian Robinson. I had so much—an education, a healthy sense of self, a deep arsenal of ambition—and I was wise enough to credit my mother, in particular, with instilling it in me. She'd taught me how to read before I started kindergarten, helping me sound out words as I sat curled like a kitten in her lap, studying a library copy of Dick and Jane. She'd cooked for us with care, putting broccoli and Brussels sprouts on our plates and requiring that we eat them. The point was, she'd given diligently and she'd given everything.
My considerable blessings in life were now causing a kind of psychic whiplash. I'd been raised to be confident and see no limits, to believe I could go after and get absolutely anything I wanted. And I wanted everything. I wanted to live with the hat-tossing, independent-career-woman zest of Mary Tyler Moore, and at the same time I gravitated toward the stabilizing, self-sacrificing, seemingly bland normalcy of being a wife and mother.
Q. Why does the author state that she liked watching The Mary Tyler Moore Show?
The author's description of Mary Tyler Moore in the second paragraph portrays an image of a woman that goes against the traditional image of a wife. Because of this, the author found the character fascinating and preferred to watch the show. Moreover, the author states 'If you were a girl with a brain and a dawning sense that you wanted to grow into something more than a wife, Mary Tyler Moore was your goddess' suggesting that Mary Tyler Moore served as a role model for the author. Therefore, the correct answer is option 2.
Read the given passage and answer the question that follows.
For many women, including myself, "wife" can feel like a loaded word. It carries a history. If you grew up in the 1960s and 1970s as I did, wives seemed to be a genus of white women who lived inside television sitcoms—cheery, coiffed, corseted. They stayed at home, fussed over the children, and had dinner ready on the stove. They sometimes got into the sherry or flirted with the vacuum-cleaner salesman, but the excitement seemed to end there. The irony, of course, was that I used to watch those shows in our living room on Euclid Avenue while my own stay-at-home mom fixed dinner without complaint and my own clean-cut dad recovered from a day at work. My parents' arrangement was as traditional as anything we saw on TV.
Personally, as a kid, I preferred The Mary Tyler Moore Show, which I absorbed with fascination. Mary had a job, a snappy wardrobe, and really great hair. She was independent and funny, and unlike those of the other ladies on TV, her problems were interesting. She had conversations that weren't about children or homemaking. She didn't let Lou Grant boss her around, and she wasn't fixated on finding a husband. She was youthful and at the same time grown-up. If you were a girl with a brain and a dawning sense that you wanted to grow into something more than a wife, Mary Tyler Moore was your goddess.
And here I was now, twenty-nine years old, sitting in the very same apartment where I'd watched all that TV and consumed all those meals dished up by the patient and selfless Marian Robinson. I had so much—an education, a healthy sense of self, a deep arsenal of ambition—and I was wise enough to credit my mother, in particular, with instilling it in me. She'd taught me how to read before I started kindergarten, helping me sound out words as I sat curled like a kitten in her lap, studying a library copy of Dick and Jane. She'd cooked for us with care, putting broccoli and Brussels sprouts on our plates and requiring that we eat them. The point was, she'd given diligently and she'd given everything.
My considerable blessings in life were now causing a kind of psychic whiplash. I'd been raised to be confident and see no limits, to believe I could go after and get absolutely anything I wanted. And I wanted everything. I wanted to live with the hat-tossing, independent-career-woman zest of Mary Tyler Moore, and at the same time I gravitated toward the stabilizing, self-sacrificing, seemingly bland normalcy of being a wife and mother.
Q. What does the word 'dawning' as used in the passage mean?
The word 'dawning' in the context means something starting to happen or become known or obvious. Here the author has a growing realisation that she wanted to grow into something more than a wife. So the correct answer is option 4.
Read the given passage and answer the question that follows.
For many women, including myself, "wife" can feel like a loaded word. It carries a history. If you grew up in the 1960s and 1970s as I did, wives seemed to be a genus of white women who lived inside television sitcoms—cheery, coiffed, corseted. They stayed at home, fussed over the children, and had dinner ready on the stove. They sometimes got into the sherry or flirted with the vacuum-cleaner salesman, but the excitement seemed to end there. The irony, of course, was that I used to watch those shows in our living room on Euclid Avenue while my own stay-at-home mom fixed dinner without complaint and my own clean-cut dad recovered from a day at work. My parents' arrangement was as traditional as anything we saw on TV.
Personally, as a kid, I preferred The Mary Tyler Moore Show, which I absorbed with fascination. Mary had a job, a snappy wardrobe, and really great hair. She was independent and funny, and unlike those of the other ladies on TV, her problems were interesting. She had conversations that weren't about children or homemaking. She didn't let Lou Grant boss her around, and she wasn't fixated on finding a husband. She was youthful and at the same time grown-up. If you were a girl with a brain and a dawning sense that you wanted to grow into something more than a wife, Mary Tyler Moore was your goddess.
And here I was now, twenty-nine years old, sitting in the very same apartment where I'd watched all that TV and consumed all those meals dished up by the patient and selfless Marian Robinson. I had so much—an education, a healthy sense of self, a deep arsenal of ambition—and I was wise enough to credit my mother, in particular, with instilling it in me. She'd taught me how to read before I started kindergarten, helping me sound out words as I sat curled like a kitten in her lap, studying a library copy of Dick and Jane. She'd cooked for us with care, putting broccoli and Brussels sprouts on our plates and requiring that we eat them. The point was, she'd given diligently and she'd given everything.
My considerable blessings in life were now causing a kind of psychic whiplash. I'd been raised to be confident and see no limits, to believe I could go after and get absolutely anything I wanted. And I wanted everything. I wanted to live with the hat-tossing, independent-career-woman zest of Mary Tyler Moore, and at the same time I gravitated toward the stabilizing, self-sacrificing, seemingly bland normalcy of being a wife and mother.
Q. As mentioned in the passage, how does the author feel about her mother?
In the third paragraph, the author describes how much her mother did for her. This is clued by the author's statement about instilling her with independence, sense of self and ambition as well as by teaching her how to read and providing meals to her. This suggests that the mother focused on the author's learning and development, contributing to her development as a strong and independent woman.
Read the given passage and answer the question that follows.
For many women, including myself, "wife" can feel like a loaded word. It carries a history. If you grew up in the 1960s and 1970s as I did, wives seemed to be a genus of white women who lived inside television sitcoms—cheery, coiffed, corseted. They stayed at home, fussed over the children, and had dinner ready on the stove. They sometimes got into the sherry or flirted with the vacuum-cleaner salesman, but the excitement seemed to end there. The irony, of course, was that I used to watch those shows in our living room on Euclid Avenue while my own stay-at-home mom fixed dinner without complaint and my own clean-cut dad recovered from a day at work. My parents' arrangement was as traditional as anything we saw on TV.
Personally, as a kid, I preferred The Mary Tyler Moore Show, which I absorbed with fascination. Mary had a job, a snappy wardrobe, and really great hair. She was independent and funny, and unlike those of the other ladies on TV, her problems were interesting. She had conversations that weren't about children or homemaking. She didn't let Lou Grant boss her around, and she wasn't fixated on finding a husband. She was youthful and at the same time grown-up. If you were a girl with a brain and a dawning sense that you wanted to grow into something more than a wife, Mary Tyler Moore was your goddess.
And here I was now, twenty-nine years old, sitting in the very same apartment where I'd watched all that TV and consumed all those meals dished up by the patient and selfless Marian Robinson. I had so much—an education, a healthy sense of self, a deep arsenal of ambition—and I was wise enough to credit my mother, in particular, with instilling it in me. She'd taught me how to read before I started kindergarten, helping me sound out words as I sat curled like a kitten in her lap, studying a library copy of Dick and Jane. She'd cooked for us with care, putting broccoli and Brussels sprouts on our plates and requiring that we eat them. The point was, she'd given diligently and she'd given everything.
My considerable blessings in life were now causing a kind of psychic whiplash. I'd been raised to be confident and see no limits, to believe I could go after and get absolutely anything I wanted. And I wanted everything. I wanted to live with the hat-tossing, independent-career-woman zest of Mary Tyler Moore, and at the same time I gravitated toward the stabilizing, self-sacrificing, seemingly bland normalcy of being a wife and mother.
Q. Why does the author think that her blessings are causing "a psychic whiplash"?
The correct answer is option 1. This is derived from the following sentence: 'I wanted to live with the hat-tossing, independent-career-woman zest of Mary Tyler Moore, and at the same time I gravitated toward the stabilizing, self-sacrificing, seemingly bland normalcy of being a wife and mother'; which suggests that by becoming a wife and a mother, she becomes a different person than what she hoped for. Option 2 might seem correct, but there is nothing to suggest regret. Options 3 and 4 are not stated in the passage.
Read the given passage and answer the question that follows.
It is a strange that, according to his position in life, an extravagant man is admired or despised. A successful businessman does nothing to increase his popularity by being careful with his money. He is expected to display his success, to have smart car, an expensive life, and to be lavish with his hospitality. If he is not so, he is considered mean and his reputation in business may even suffer in consequence. The paradox remains that if he had not been careful with his money in the first place, he would never have achieved his present wealth. Among the two income groups, a different set of values exists.
The young clerk who makes his wife a present of a new dress when the hadn’t paid his house rent condemned as extravagant. Carefulness with money to the point of meanness is applauded as a virtue. Nothing in his life is considered more worthy than paying his bills. The ideal wife for such a man separates her housekeeping money into joyless little piles-so much for rent, for food, for the children’s shoes; she is able to face the milkman with equanimity and never knows the guilt of buying something she can’t able to face the milkman with equanimity and never knows the guilt of buying something she can’t really afford. As for myself, I fall into neither of these categories. If I have money to spare I can be extravagant, but when, as is usually the cause, I am hard up, then I am then meanest man imaginable.
Q. The phrase lavish with his hospitality signifies
Read the given passage and answer the question that follows.
It is a strange that, according to his position in life, an extravagant man is admired or despised. A successful businessman does nothing to increase his popularity by being careful with his money. He is expected to display his success, to have smart car, an expensive life, and to be lavish with his hospitality. If he is not so, he is considered mean and his reputation in business may even suffer in consequence. The paradox remains that if he had not been careful with his money in the first place, he would never have achieved his present wealth. Among the two income groups, a different set of values exists.
The young clerk who makes his wife a present of a new dress when the hadn’t paid his house rent condemned as extravagant. Carefulness with money to the point of meanness is applauded as a virtue. Nothing in his life is considered more worthy than paying his bills. The ideal wife for such a man separates her housekeeping money into joyless little piles-so much for rent, for food, for the children’s shoes; she is able to face the milkman with equanimity and never knows the guilt of buying something she can’t able to face the milkman with equanimity and never knows the guilt of buying something she can’t really afford. As for myself, I fall into neither of these categories. If I have money to spare I can be extravagant, but when, as is usually the cause, I am hard up, then I am then meanest man imaginable.
Q. The word paradox means.
Read the given passage and answer the question that follows.
It is a strange that, according to his position in life, an extravagant man is admired or despised. A successful businessman does nothing to increase his popularity by being careful with his money. He is expected to display his success, to have smart car, an expensive life, and to be lavish with his hospitality. If he is not so, he is considered mean and his reputation in business may even suffer in consequence. The paradox remains that if he had not been careful with his money in the first place, he would never have achieved his present wealth. Among the two income groups, a different set of values exists.
The young clerk who makes his wife a present of a new dress when the hadn’t paid his house rent condemned as extravagant. Carefulness with money to the point of meanness is applauded as a virtue. Nothing in his life is considered more worthy than paying his bills. The ideal wife for such a man separates her housekeeping money into joyless little piles-so much for rent, for food, for the children’s shoes; she is able to face the milkman with equanimity and never knows the guilt of buying something she can’t able to face the milkman with equanimity and never knows the guilt of buying something she can’t really afford. As for myself, I fall into neither of these categories. If I have money to spare I can be extravagant, but when, as is usually the cause, I am hard up, then I am then meanest man imaginable.
Q. Which of the following is opposite in meaning to the word applauded in the passage?
Read the given passage and answer the question that follows.
It is a strange that, according to his position in life, an extravagant man is admired or despised. A successful businessman does nothing to increase his popularity by being careful with his money. He is expected to display his success, to have smart car, an expensive life, and to be lavish with his hospitality. If he is not so, he is considered mean and his reputation in business may even suffer in consequence. The paradox remains that if he had not been careful with his money in the first place, he would never have achieved his present wealth. Among the two income groups, a different set of values exists.
The young clerk who makes his wife a present of a new dress when the hadn’t paid his house rent condemned as extravagant. Carefulness with money to the point of meanness is applauded as a virtue. Nothing in his life is considered more worthy than paying his bills. The ideal wife for such a man separates her housekeeping money into joyless little piles-so much for rent, for food, for the children’s shoes; she is able to face the milkman with equanimity and never knows the guilt of buying something she can’t able to face the milkman with equanimity and never knows the guilt of buying something she can’t really afford. As for myself, I fall into neither of these categories. If I have money to spare I can be extravagant, but when, as is usually the cause, I am hard up, then I am then meanest man imaginable
Q. The statement she is able to face the milkman with equanimity implies that:
Read the given passage and answer the question that follows.
Let us consider for a moment the discovery of the cause of malaria. This discovery, due to the Englishman, Ross consists in having found out that the plasmodium of malariae is inoculated in man by a special kind of mosquito. Let us inquire what was the state of science prior to this discovery. In 1880 Laveran had described an animal micro-organism, which preyed upon the red corpuscles of the blood, producing an attack of fever with the cycle of its existence. Subsequent studies confirmed and elucidated this fact, and the plasmodium malaria became a matter of common knowledge. It was known that animal micro-organisms, unlike vegetable micro-organisms, after a cycle of life in which reproduction takes place by scission; that is, by subdivision of a single body into several other bodies equal to the first, give place to sexual forms, masculine and feminine, which are separate, and incapable of scission, but are designed for fusion into one another, after which the organism recommences its cycle of scissions until it again reaches the sexual forms.
Laveran had found that in the blood of sufferers who recover spontaneously from malarial fever there are a great number of corpuscles which have no longer the rounded forms of the plasmodia, but are crescent-shaped and rayed. He took these to be transformations of the plasmodia, "modified in form" and "incapable of producing disease," and pronounced them to be "degenerate" organisms, almost as if they had been deformed and exhausted by the "excess of work" they had previously performed. After the discovery of the transmission of malaria in 1900, Laveran's "degenerative forms" were recognized as the sexual individuals of the reproductive cycle: individuals which were incapable of conjugation in the blood of man, and could only produce new organisms in the body of the mosquito. We may well wonder: Why did not Laveran simply recognize those sexual forms, and why did he not seek for the period of conjugation in the plasmodia, which were animal micro-organisms?
Another biological acquisition was the assurance that the circulatory system of the blood is a closed system of vessels, and that the enclosing epithelium is not permeable by non-incisive solid bodies such as vegetable microbes, and still less by rounded protozoa, which are much larger than microbes and soft in substance. This well-known and clearly demonstrated fact ought to have suggested a problem to the minds of students: How do the protozoa of malaria enter the circulatory current of the blood? But ever since the days of Hippocrates, Pliny, Celsius and Galen it had been held that this fever was caused by the "poisonous atmosphere" of marsh lands, the bad air of the morning and the evening, so much so that even a few years before the discovery of the real cause of malaria, eucalyptus trees were planted in the belief that they would filter and disinfect the air.
Until Ross discovered that birds are inoculated with malaria by a particular kind of mosquito.
Q. What is the contextual meaning of the word "Elucidated" as used in the paragraph?
Read the given passage and answer the question that follows.
Let us consider for a moment the discovery of the cause of malaria. This discovery, due to the Englishman, Ross consists in having found out that the plasmodium of malariae is inoculated in man by a special kind of mosquito. Let us inquire what was the state of science prior to this discovery. In 1880 Laveran had described an animal micro-organism, which preyed upon the red corpuscles of the blood, producing an attack of fever with the cycle of its existence. Subsequent studies confirmed and elucidated this fact, and the plasmodium malaria became a matter of common knowledge. It was known that animal micro-organisms, unlike vegetable micro-organisms, after a cycle of life in which reproduction takes place by scission; that is, by subdivision of a single body into several other bodies equal to the first, give place to sexual forms, masculine and feminine, which are separate, and incapable of scission, but are designed for fusion into one another, after which the organism recommences its cycle of scissions until it again reaches the sexual forms.
Laveran had found that in the blood of sufferers who recover spontaneously from malarial fever there are a great number of corpuscles which have no longer the rounded forms of the plasmodia, but are crescent-shaped and rayed. He took these to be transformations of the plasmodia, "modified in form" and "incapable of producing disease," and pronounced them to be "degenerate" organisms, almost as if they had been deformed and exhausted by the "excess of work" they had previously performed. After the discovery of the transmission of malaria in 1900, Laveran's "degenerative forms" were recognized as the sexual individuals of the reproductive cycle: individuals which were incapable of conjugation in the blood of man, and could only produce new organisms in the body of the mosquito. We may well wonder: Why did not Laveran simply recognize those sexual forms, and why did he not seek for the period of conjugation in the plasmodia, which were animal micro-organisms?
Another biological acquisition was the assurance that the circulatory system of the blood is a closed system of vessels, and that the enclosing epithelium is not permeable by non-incisive solid bodies such as vegetable microbes, and still less by rounded protozoa, which are much larger than microbes and soft in substance. This well-known and clearly demonstrated fact ought to have suggested a problem to the minds of students: How do the protozoa of malaria enter the circulatory current of the blood? But ever since the days of Hippocrates, Pliny, Celsius and Galen it had been held that this fever was caused by the "poisonous atmosphere" of marsh lands, the bad air of the morning and the evening, so much so that even a few years before the discovery of the real cause of malaria, eucalyptus trees were planted in the belief that they would filter and disinfect the air.
Until Ross discovered that birds are inoculated with malaria by a particular kind of mosquito.
Q. Why did Laveran call the sexual form of the plasmodium malariae a ‘degenerative’ form?
The dictionary meaning of the word degenerate: having lost the physical, mental, or moral qualities considered normal and desirable; showing evidence of decline.
Laveran found this shape change in patients who had spontaneously recovered – so he felt that the reduction in potency of this form was the reason for the recovery – hence he called it degenerate.
Option (A) – The change in shape does not connect to potency
Option (C) – This information was not available to Laveran.
Option (D) – Even if this statement is correct, it does not relate to degeneracy.
Read the given passage and answer the question that follows.
It was the biggest decision of her life, the one for which she is most remembered, but Freda Bedi didn't tell her children that she was being ordained as a Buddhist nun. There was no family council, no private conversation, not even, it seems, a letter to announce her intention.
"There was this terrible feeling of betrayal," Kabir Bedi recalls. It was 1966 and the height of the Delhi summer. Kabir was 20, a student at one of India's most prestigious university colleges, St Stephen's, and still recovering from a broken back. He understood that Buddhism loomed increasingly large in his mother's life, but hadn't been prepared for her ordination as a nun.
He was angry and said so. Why? he demanded of his mother; why now? He still remembers her response. "It is something I felt I had to do and I knew if I started discussing it with everybody, God knows what might have happened." Kabir was seven when his mother found Buddhism while on a United Nations mission to Burma (now Myanmar). He had accompanied her back there when she studied meditation, and had himself enrolled briefly as a novitiate. He had worn the robes and shaved off his hair—in much the same manner as his mother had now done. He had spent time with his mother at the camps in Assam set up for the Tibetans who fled across the mountains to escape Chinese rule—that's where she first became immersed in Tibetan belief and culture. He had taught at the Young Lamas' Home School she established. It had felt like a shared journey. Now Freda, Sister Palmo as she became known, had decided to press on alone. "I raised all the silly arguments I could think of: Your daughter's still in college, she's not married, how's she going to manage? All silly things. But basically, I was angry because I felt betrayed. There was a terrible sense of loss. It's like, you've lost your mother."
A few days after the ceremony, still at Rumtek, Freda received what was clearly an anguished letter from Kabir. Manorma Dewan was part of the extended family—her husband's flat was the venue of Kabir's meeting with his newly-robed mother—and remembers the central message of that letter: "You have become very selfish." Manorma agreed with that view. Freda replied immediately by telegram, and followed that up with a three-page handwritten missive to her 'darling son'. Kabir still has that letter. "I have been in a maze of pain, feeling your and Guli's," she wrote. "You all knew one day this step would be taken; we even joked about my losing my hair! Somehow, now had to be the time."
Q. According to the author, how did Freda Bedi's family feel about her becoming a nun?
The correct answer is option 4. In the first paragraph, we get a sense of the decision being abrupt because she made the decision by herself without informing anyone. In terms of being inconsiderate, there are several instances, where this is implied by the following 'There was this terrible feeling of betrayal'; 'He was angry and said so'; 'I was angry because I felt betrayed'; '"You have become very selfish.'
Read the given passage and answer the question that follows.
It was the biggest decision of her life, the one for which she is most remembered, but Freda Bedi didn't tell her children that she was being ordained as a Buddhist nun. There was no family council, no private conversation, not even, it seems, a letter to announce her intention.
"There was this terrible feeling of betrayal," Kabir Bedi recalls. It was 1966 and the height of the Delhi summer. Kabir was 20, a student at one of India's most prestigious university colleges, St Stephen's, and still recovering from a broken back. He understood that Buddhism loomed increasingly large in his mother's life, but hadn't been prepared for her ordination as a nun.
He was angry and said so. Why? he demanded of his mother; why now? He still remembers her response. "It is something I felt I had to do and I knew if I started discussing it with everybody, God knows what might have happened." Kabir was seven when his mother found Buddhism while on a United Nations mission to Burma (now Myanmar). He had accompanied her back there when she studied meditation, and had himself enrolled briefly as a novitiate. He had worn the robes and shaved off his hair—in much the same manner as his mother had now done. He had spent time with his mother at the camps in Assam set up for the Tibetans who fled across the mountains to escape Chinese rule—that's where she first became immersed in Tibetan belief and culture. He had taught at the Young Lamas' Home School she established. It had felt like a shared journey. Now Freda, Sister Palmo as she became known, had decided to press on alone. "I raised all the silly arguments I could think of: Your daughter's still in college, she's not married, how's she going to manage? All silly things. But basically, I was angry because I felt betrayed. There was a terrible sense of loss. It's like, you've lost your mother."
A few days after the ceremony, still at Rumtek, Freda received what was clearly an anguished letter from Kabir. Manorma Dewan was part of the extended family—her husband's flat was the venue of Kabir's meeting with his newly-robed mother—and remembers the central message of that letter: "You have become very selfish." Manorma agreed with that view. Freda replied immediately by telegram, and followed that up with a three-page handwritten missive to her 'darling son'. Kabir still has that letter. "I have been in a maze of pain, feeling your and Guli's," she wrote. "You all knew one day this step would be taken; we even joked about my losing my hair! Somehow, now had to be the time."
Q. Why did Freda feel it important to make the decision without consulting her family?
The correct answer is option 3. The answer can be derived from the following sentence; 'He still remembers her response. 'It is something I felt I had to do and I knew if I started discussing it with everybody, God knows what might have happened.'' This suggests that she likely believed that if she were to discuss the matter, her family would put forth several arguments that might pose a challenge for her.
Read the given passage and answer the question that follows.
It was the biggest decision of her life, the one for which she is most remembered, but Freda Bedi didn't tell her children that she was being ordained as a Buddhist nun. There was no family council, no private conversation, not even, it seems, a letter to announce her intention.
"There was this terrible feeling of betrayal," Kabir Bedi recalls. It was 1966 and the height of the Delhi summer. Kabir was 20, a student at one of India's most prestigious university colleges, St Stephen's, and still recovering from a broken back. He understood that Buddhism loomed increasingly large in his mother's life, but hadn't been prepared for her ordination as a nun.
He was angry and said so. Why? he demanded of his mother; why now? He still remembers her response. "It is something I felt I had to do and I knew if I started discussing it with everybody, God knows what might have happened." Kabir was seven when his mother found Buddhism while on a United Nations mission to Burma (now Myanmar). He had accompanied her back there when she studied meditation, and had himself enrolled briefly as a novitiate. He had worn the robes and shaved off his hair—in much the same manner as his mother had now done. He had spent time with his mother at the camps in Assam set up for the Tibetans who fled across the mountains to escape Chinese rule—that's where she first became immersed in Tibetan belief and culture. He had taught at the Young Lamas' Home School she established. It had felt like a shared journey. Now Freda, Sister Palmo as she became known, had decided to press on alone. "I raised all the silly arguments I could think of: Your daughter's still in college, she's not married, how's she going to manage? All silly things. But basically, I was angry because I felt betrayed. There was a terrible sense of loss. It's like, you've lost your mother."
A few days after the ceremony, still at Rumtek, Freda received what was clearly an anguished letter from Kabir. Manorma Dewan was part of the extended family—her husband's flat was the venue of Kabir's meeting with his newly-robed mother—and remembers the central message of that letter: "You have become very selfish." Manorma agreed with that view. Freda replied immediately by telegram, and followed that up with a three-page handwritten missive to her 'darling son'. Kabir still has that letter. "I have been in a maze of pain, feeling your and Guli's," she wrote. "You all knew one day this step would be taken; we even joked about my losing my hair! Somehow, now had to be the time."
Q. What does the phrase 'loomed large' as used in the passage mean?
The correct answer is option 1. The thought of converting herself into a Buddhist was a decision of great importance and something that would cause a lot of worry to Freda's family as implied in the text.
Read the given passage and answer the question that follows.
It was the biggest decision of her life, the one for which she is most remembered, but Freda Bedi didn't tell her children that she was being ordained as a Buddhist nun. There was no family council, no private conversation, not even, it seems, a letter to announce her intention.
"There was this terrible feeling of betrayal," Kabir Bedi recalls. It was 1966 and the height of the Delhi summer. Kabir was 20, a student at one of India's most prestigious university colleges, St Stephen's, and still recovering from a broken back. He understood that Buddhism loomed increasingly large in his mother's life, but hadn't been prepared for her ordination as a nun.
He was angry and said so. Why? he demanded of his mother; why now? He still remembers her response. "It is something I felt I had to do and I knew if I started discussing it with everybody, God knows what might have happened." Kabir was seven when his mother found Buddhism while on a United Nations mission to Burma (now Myanmar). He had accompanied her back there when she studied meditation, and had himself enrolled briefly as a novitiate. He had worn the robes and shaved off his hair—in much the same manner as his mother had now done. He had spent time with his mother at the camps in Assam set up for the Tibetans who fled across the mountains to escape Chinese rule—that's where she first became immersed in Tibetan belief and culture. He had taught at the Young Lamas' Home School she established. It had felt like a shared journey. Now Freda, Sister Palmo as she became known, had decided to press on alone. "I raised all the silly arguments I could think of: Your daughter's still in college, she's not married, how's she going to manage? All silly things. But basically, I was angry because I felt betrayed. There was a terrible sense of loss. It's like, you've lost your mother."
A few days after the ceremony, still at Rumtek, Freda received what was clearly an anguished letter from Kabir. Manorma Dewan was part of the extended family—her husband's flat was the venue of Kabir's meeting with his newly-robed mother—and remembers the central message of that letter: "You have become very selfish." Manorma agreed with that view. Freda replied immediately by telegram, and followed that up with a three-page handwritten missive to her 'darling son'. Kabir still has that letter. "I have been in a maze of pain, feeling your and Guli's," she wrote. "You all knew one day this step would be taken; we even joked about my losing my hair! Somehow, now had to be the time."
Q. Based on the information set out in the passage, which of the following is most accurate?
The only correct answer out of the four as per the information given in the passage is option 4. Option 1 can be ruled out because the passage does not state that Kabir was afraid of the Chinese rule. Option 2 is incorrect because it was Freda who picked up the faith while on a United Nations mission to Burma, not Kabir. Option 3 is incorrect because this is not stated in the passage. Option 4 is correct because the author states this in these lines: "It had felt like a shared journey. Now Freda, Sister Palmo as she became known, had decided to press on alone. "I raised all the silly arguments I could think of: Your daughter's still in college, she's not married, how's she going to manage? All silly things. But basically, I was angry because I felt betrayed. There was a terrible sense of loss. It's like, you've lost your mother."
Read the given passage and answer the question that follows.
It was the biggest decision of her life, the one for which she is most remembered, but Freda Bedi didn't tell her children that she was being ordained as a Buddhist nun. There was no family council, no private conversation, not even, it seems, a letter to announce her intention.
"There was this terrible feeling of betrayal," Kabir Bedi recalls. It was 1966 and the height of the Delhi summer. Kabir was 20, a student at one of India's most prestigious university colleges, St Stephen's, and still recovering from a broken back. He understood that Buddhism loomed increasingly large in his mother's life, but hadn't been prepared for her ordination as a nun.
He was angry and said so. Why? he demanded of his mother; why now? He still remembers her response. "It is something I felt I had to do and I knew if I started discussing it with everybody, God knows what might have happened." Kabir was seven when his mother found Buddhism while on a United Nations mission to Burma (now Myanmar). He had accompanied her back there when she studied meditation, and had himself enrolled briefly as a novitiate. He had worn the robes and shaved off his hair—in much the same manner as his mother had now done. He had spent time with his mother at the camps in Assam set up for the Tibetans who fled across the mountains to escape Chinese rule—that's where she first became immersed in Tibetan belief and culture. He had taught at the Young Lamas' Home School she established. It had felt like a shared journey. Now Freda, Sister Palmo as she became known, had decided to press on alone. "I raised all the silly arguments I could think of: Your daughter's still in college, she's not married, how's she going to manage? All silly things. But basically, I was angry because I felt betrayed. There was a terrible sense of loss. It's like, you've lost your mother."
A few days after the ceremony, still at Rumtek, Freda received what was clearly an anguished letter from Kabir. Manorma Dewan was part of the extended family—her husband's flat was the venue of Kabir's meeting with his newly-robed mother—and remembers the central message of that letter: "You have become very selfish." Manorma agreed with that view. Freda replied immediately by telegram, and followed that up with a three-page handwritten missive to her 'darling son'. Kabir still has that letter. "I have been in a maze of pain, feeling your and Guli's," she wrote. "You all knew one day this step would be taken; we even joked about my losing my hair! Somehow, now had to be the time."
Q. What can be inferred from Freda Bedi's response to Kabir's letter?
The correct answer is option 2. The answer can be derived from the following lines: "Kabir still has that letter. "I have been in a maze of pain, feeling your and Guli's," she wrote. "You all knew one day this step would be taken; we even joked about my losing my hair! Somehow, now had to be the time.""
Read the given passage and answer the question that follows.
Job performance is affected by a number of factors. Motivation alone does not lead to increase in performance. Ability and technology moderates the relationship between motivation and performance. The higher the levels of ability and motivation, the higher the level of performance will be. However, increasing motivation beyond an optimal level tends to produce a dysfunctional result because it is accompanied by an increasing level of anxiety. A high level of anxiety often disrupts performances.
The relationship between satisfaction and performance is not clear. Satisfaction may or may not lead to high performance depending on the perceived availability of valued outcomes and the perceived expectancy that a person’s effort and performance will lead to receiving the valued rewards. If the person expects that his performance will lead to increased rewards which he values, the level of his motivational effort will increase, if he anticipates less, his motivational effort will increase, if he anticipates less, his motivational effort will be lower.
The relationship between job dissatisfaction and poor performance seems to be clearer than that between satisfaction and performance. Dissatisfaction leads to poor performance by means of apathy, absenteeism, turnover, sabotage, and strike. In addition, high performers are more vulnerable to job dissatisfaction because they tend to expect more from their jobs than low performers.
Job satisfaction is more closely related to the decision to join and remain in an organisation than to the motivation to produce. The motivation to produce largely depends on the availability of valued outcomes (valence), the perceived instrumentality of performance for receiving incentive rewards, and the perceived expectancy that effort leads to performance. The task of satisfying employees is much easier than the task of motivating them because the former can be achieved by rewarding them while the latter requires such additional constraints as establishing performance-reward contingencies and designing motivating work systems.
Q. The individual’s decision to remain in the organisation depends on
Read the given passage and answer the question that follows.
Job performance is affected by a number of factors. Motivation alone does not lead to increase in performance. Ability and technology moderates the relationship between motivation and performance. The higher the levels of ability and motivation, the higher the level of performance will be. However, increasing motivation beyond an optimal level tends to produce a dysfunctional result because it is accompanied by an increasing level of anxiety. A high level of anxiety often disrupts performances.
The relationship between satisfaction and performance is not clear. Satisfaction may or may not lead to high performance depending on the perceived availability of valued outcomes and the perceived expectancy that a person’s effort and performance will lead to receiving the valued rewards. If the person expects that his performance will lead to increased rewards which he values, the level of his motivational effort will increase, if he anticipates less, his motivational effort will increase, if he anticipates less, his motivational effort will be lower.
The relationship between job dissatisfaction and poor performance seems to be clearer than that between satisfaction and performance. Dissatisfaction leads to poor performance by means of apathy, absenteeism, turnover, sabotage, and strike. In addition, high performers are more vulnerable to job dissatisfaction because they tend to expect more from their jobs than low performers.
Job satisfaction is more closely related to the decision to join and remain in an organisation than to the motivation to produce. The motivation to produce largely depends on the availability of valued outcomes (valence), the perceived instrumentality of performance for receiving incentive rewards, and the perceived expectancy that effort leads to performance. The task of satisfying employees is much easier than the task of motivating them because the former can be achieved by rewarding them while the latter requires such additional constraints as establishing performance-reward contingencies and designing motivating work systems.
Q. Which of the following tasks is easier according to the passage?
Read the given passage and answer the question that follows.
Job performance is affected by a number of factors. Motivation alone does not lead to increase in performance. Ability and technology moderates the relationship between motivation and performance. The higher the levels of ability and motivation, the higher the level of performance will be. However, increasing motivation beyond an optimal level tends to produce a dysfunctional result because it is accompanied by an increasing level of anxiety. A high level of anxiety often disrupts performances.
The relationship between satisfaction and performance is not clear. Satisfaction may or may not lead to high performance depending on the perceived availability of valued outcomes and the perceived expectancy that a person’s effort and performance will lead to receiving the valued rewards. If the person expects that his performance will lead to increased rewards which he values, the level of his motivational effort will increase, if he anticipates less, his motivational effort will increase, if he anticipates less, his motivational effort will be lower.
The relationship between job dissatisfaction and poor performance seems to be clearer than that between satisfaction and performance. Dissatisfaction leads to poor performance by means of apathy, absenteeism, turnover, sabotage, and strike. In addition, high performers are more vulnerable to job dissatisfaction because they tend to expect more from their jobs than low performers.
Job satisfaction is more closely related to the decision to join and remain in an organisation than to the motivation to produce. The motivation to produce largely depends on the availability of valued outcomes (valence), the perceived instrumentality of performance for receiving incentive rewards, and the perceived expectancy that effort leads to performance. The task of satisfying employees is much easier than the task of motivating them because the former can be achieved by rewarding them while the latter requires such additional constraints as establishing performance-reward contingencies and designing motivating work systems.
Q. Which of the following statement/s/is/are true in the context of the passage?
(A) Ability leads to performance.
(B) Job satisfaction certainly leads to higher performance.
(C) High anxiety adversely affects performance.
Read the given passage and answer the question that follows.
Job performance is affected by a number of factors. Motivation alone does not lead to increase in performance. Ability and technology moderates the relationship between motivation and performance. The higher the levels of ability and motivation, the higher the level of performance will be. However, increasing motivation beyond an optimal level tends to produce a dysfunctional result because it is accompanied by an increasing level of anxiety. A high level of anxiety often disrupts performances.
The relationship between satisfaction and performance is not clear. Satisfaction may or may not lead to high performance depending on the perceived availability of valued outcomes and the perceived expectancy that a person’s effort and performance will lead to receiving the valued rewards. If the person expects that his performance will lead to increased rewards which he values, the level of his motivational effort will increase, if he anticipates less, his motivational effort will increase, if he anticipates less, his motivational effort will be lower.
The relationship between job dissatisfaction and poor performance seems to be clearer than that between satisfaction and performance. Dissatisfaction leads to poor performance by means of apathy, absenteeism, turnover, sabotage, and strike. In addition, high performers are more vulnerable to job dissatisfaction because they tend to expect more from their jobs than low performers.
Job satisfaction is more closely related to the decision to join and remain in an organisation than to the motivation to produce. The motivation to produce largely depends on the availability of valued outcomes (valence), the perceived instrumentality of performance for receiving incentive rewards, and the perceived expectancy that effort leads to performance. The task of satisfying employees is much easier than the task of motivating them because the former can be achieved by rewarding them while the latter requires such additional constraints as establishing performance-reward contingencies and designing motivating work systems.
Q. Which of the following combination of factors affects job performance?
Read the given passage and answer the question that follows.
Job performance is affected by a number of factors. Motivation alone does not lead to increase in performance. Ability and technology moderates the relationship between motivation and performance. The higher the levels of ability and motivation, the higher the level of performance will be. However, increasing motivation beyond an optimal level tends to produce a dysfunctional result because it is accompanied by an increasing level of anxiety. A high level of anxiety often disrupts performances.
The relationship between satisfaction and performance is not clear. Satisfaction may or may not lead to high performance depending on the perceived availability of valued outcomes and the perceived expectancy that a person’s effort and performance will lead to receiving the valued rewards. If the person expects that his performance will lead to increased rewards which he values, the level of his motivational effort will increase, if he anticipates less, his motivational effort will increase, if he anticipates less, his motivational effort will be lower.
The relationship between job dissatisfaction and poor performance seems to be clearer than that between satisfaction and performance. Dissatisfaction leads to poor performance by means of apathy, absenteeism, turnover, sabotage, and strike. In addition, high performers are more vulnerable to job dissatisfaction because they tend to expect more from their jobs than low performers.
Job satisfaction is more closely related to the decision to join and remain in an organisation than to the motivation to produce. The motivation to produce largely depends on the availability of valued outcomes (valence), the perceived instrumentality of performance for receiving incentive rewards, and the perceived expectancy that effort leads to performance. The task of satisfying employees is much easier than the task of motivating them because the former can be achieved by rewarding them while the latter requires such additional constraints as establishing performance-reward contingencies and designing motivating work systems.
Q. The task of motivating employees is difficult due to
Read the given passage and answer the question that follows.
Here we come to the heart of the matter: I've never left Istanbul – never left the houses, streets and neighbourhoods of my childhood. Although I've lived in other districts from time to time, fifty years on I find myself back in the Pamuk Apartments, where my first photographs were taken and where my mother first held me in her arms to show me the world. I know this persistence owes something to my imaginary friend, and to the solace I took from the bond between us. But we live in an age defined by mass migration and creative immigrants, and so I am sometimes hard-pressed to explain why I've stayed not only in the same place, but the same building. My mother's sorrowful voice comes back to me, 'Why don't you go outside for a while, why don't you try a change of scene, do some travelling …?'
Conrad, Nabokov, Naipaul – these are writers known for having managed to migrate between languages, cultures, countries, continents, even civilisations. Their imaginations were fed by exile, a nourishment drawn not through roots but through rootlessness; mine, however, requires that I stay in the same city, on the same street, in the same house, gazing at the same view. Istanbul's fate is my fate: I am attached to this city because it has made me who I am.
Flaubert, who visited Istanbul a hundred and two years before my birth, was struck by the variety of life in its teeming streets; in one of his letters he predicted that in a century's time it would be the capital of the world. The reverse came true: after the Ottoman Empire collapsed, the world almost forgot that Istanbul existed. The city into which I was born was poorer, shabbier, and more isolated than it had ever been its two-thousand-year history. For me it has always been a city of ruins and of end-of-empire melancholy. I've spent my life either battling with this melancholy, or (like all Istanbullus) making it my own.
At least once in a lifetime, self-reflection leads us to examine the circumstances of our birth. Why were we born in this particular corner of the world, on this particular date? These families into which we were born, these countries and cities to which the lottery of life has assigned us – they expect love from us, and in the end, we do love them, from the bottom of our hearts – but did we perhaps deserve better? I sometimes think myself unlucky to have been born in an ageing and impoverished city buried under the ashes of a ruined empire. But a voice inside me always insists this was really a piece of luck.
Q. What could be the reason for the author never leaving Istanbul?
The correct answer is option 2. This is derived from the following sentence; 'I am attached to this city because it has made me who I am.' Option 1 might seem correct, but the author merely says that at times he feels unlucky. Option 3 is incorrect because 'the world's capital' is mentioned by Flaubert and not the author; hence it is out of context. Option 4 is implied, but does not provide a specific reason for not leaving the city. It just rephrases the question.
Read the given passage and answer the question that follows.
Here we come to the heart of the matter: I've never left Istanbul – never left the houses, streets and neighbourhoods of my childhood. Although I've lived in other districts from time to time, fifty years on I find myself back in the Pamuk Apartments, where my first photographs were taken and where my mother first held me in her arms to show me the world. I know this persistence owes something to my imaginary friend, and to the solace I took from the bond between us. But we live in an age defined by mass migration and creative immigrants, and so I am sometimes hard-pressed to explain why I've stayed not only in the same place, but the same building. My mother's sorrowful voice comes back to me, 'Why don't you go outside for a while, why don't you try a change of scene, do some travelling …?'
Conrad, Nabokov, Naipaul – these are writers known for having managed to migrate between languages, cultures, countries, continents, even civilisations. Their imaginations were fed by exile, a nourishment drawn not through roots but through rootlessness; mine, however, requires that I stay in the same city, on the same street, in the same house, gazing at the same view. Istanbul's fate is my fate: I am attached to this city because it has made me who I am.
Flaubert, who visited Istanbul a hundred and two years before my birth, was struck by the variety of life in its teeming streets; in one of his letters he predicted that in a century's time it would be the capital of the world. The reverse came true: after the Ottoman Empire collapsed, the world almost forgot that Istanbul existed. The city into which I was born was poorer, shabbier, and more isolated than it had ever been its two-thousand-year history. For me it has always been a city of ruins and of end-of-empire melancholy. I've spent my life either battling with this melancholy, or (like all Istanbullus) making it my own.
At least once in a lifetime, self-reflection leads us to examine the circumstances of our birth. Why were we born in this particular corner of the world, on this particular date? These families into which we were born, these countries and cities to which the lottery of life has assigned us – they expect love from us, and in the end, we do love them, from the bottom of our hearts – but did we perhaps deserve better? I sometimes think myself unlucky to have been born in an ageing and impoverished city buried under the ashes of a ruined empire. But a voice inside me always insists this was really a piece of luck.
Q. As mentioned in the passage, how does the author characterise his life in the city?
The correct answer is option 3. The answer is derived from this part: 'For me it has always been a city of ruins and of end-of-empire melancholy. I've spent my life either battling with this melancholy, or (like all Istanbullus) making it my own'. This suggests that the author would at times be depressed, but also struggled to make the best of his situation as indicated by 'making it my own'. This option is also supported when the author states 'I sometimes think myself unlucky to have been born in an ageing and impoverished city buried under the ashes of a ruined empire. But a voice inside me always insists this was really a piece of luck.'
Read the given passage and answer the question that follows.
Here we come to the heart of the matter: I've never left Istanbul – never left the houses, streets and neighbourhoods of my childhood. Although I've lived in other districts from time to time, fifty years on I find myself back in the Pamuk Apartments, where my first photographs were taken and where my mother first held me in her arms to show me the world. I know this persistence owes something to my imaginary friend, and to the solace I took from the bond between us. But we live in an age defined by mass migration and creative immigrants, and so I am sometimes hard-pressed to explain why I've stayed not only in the same place, but the same building. My mother's sorrowful voice comes back to me, 'Why don't you go outside for a while, why don't you try a change of scene, do some travelling …?'
Conrad, Nabokov, Naipaul – these are writers known for having managed to migrate between languages, cultures, countries, continents, even civilisations. Their imaginations were fed by exile, a nourishment drawn not through roots but through rootlessness; mine, however, requires that I stay in the same city, on the same street, in the same house, gazing at the same view. Istanbul's fate is my fate: I am attached to this city because it has made me who I am.
Flaubert, who visited Istanbul a hundred and two years before my birth, was struck by the variety of life in its teeming streets; in one of his letters he predicted that in a century's time it would be the capital of the world. The reverse came true: after the Ottoman Empire collapsed, the world almost forgot that Istanbul existed. The city into which I was born was poorer, shabbier, and more isolated than it had ever been its two-thousand-year history. For me it has always been a city of ruins and of end-of-empire melancholy. I've spent my life either battling with this melancholy, or (like all Istanbullus) making it my own.
At least once in a lifetime, self-reflection leads us to examine the circumstances of our birth. Why were we born in this particular corner of the world, on this particular date? These families into which we were born, these countries and cities to which the lottery of life has assigned us – they expect love from us, and in the end, we do love them, from the bottom of our hearts – but did we perhaps deserve better? I sometimes think myself unlucky to have been born in an ageing and impoverished city buried under the ashes of a ruined empire. But a voice inside me always insists this was really a piece of luck.
Q. What does the word 'melancholy' as used in the passage mean?
The correct answer is option 3. The author implies the meaning in the following line: "For me it has always been a city of ruins and of end-of-empire melancholy. I've spent my life either battling with this melancholy, or (like all Istanbullus) making it my own." When the author looks at the ruins of the city a feeling of sadness which he says he is "battling" with gives an idea that it is something negative in connotation.
Read the given passage and answer the question that follows.
Here we come to the heart of the matter: I've never left Istanbul – never left the houses, streets and neighbourhoods of my childhood. Although I've lived in other districts from time to time, fifty years on I find myself back in the Pamuk Apartments, where my first photographs were taken and where my mother first held me in her arms to show me the world. I know this persistence owes something to my imaginary friend, and to the solace I took from the bond between us. But we live in an age defined by mass migration and creative immigrants, and so I am sometimes hard-pressed to explain why I've stayed not only in the same place, but the same building. My mother's sorrowful voice comes back to me, 'Why don't you go outside for a while, why don't you try a change of scene, do some travelling …?'
Conrad, Nabokov, Naipaul – these are writers known for having managed to migrate between languages, cultures, countries, continents, even civilisations. Their imaginations were fed by exile, a nourishment drawn not through roots but through rootlessness; mine, however, requires that I stay in the same city, on the same street, in the same house, gazing at the same view. Istanbul's fate is my fate: I am attached to this city because it has made me who I am.
Flaubert, who visited Istanbul a hundred and two years before my birth, was struck by the variety of life in its teeming streets; in one of his letters he predicted that in a century's time it would be the capital of the world. The reverse came true: after the Ottoman Empire collapsed, the world almost forgot that Istanbul existed. The city into which I was born was poorer, shabbier, and more isolated than it had ever been its two-thousand-year history. For me it has always been a city of ruins and of end-of-empire melancholy. I've spent my life either battling with this melancholy, or (like all Istanbullus) making it my own.
At least once in a lifetime, self-reflection leads us to examine the circumstances of our birth. Why were we born in this particular corner of the world, on this particular date? These families into which we were born, these countries and cities to which the lottery of life has assigned us – they expect love from us, and in the end, we do love them, from the bottom of our hearts – but did we perhaps deserve better? I sometimes think myself unlucky to have been born in an ageing and impoverished city buried under the ashes of a ruined empire. But a voice inside me always insists this was really a piece of luck.
Q. Why does the author mention great literary names of "Conrad, Nabokov, Naipaul" in the passage?
The correct answer is option 3. The answer can be derived from the following sentences: "Conrad, Nabokov, Naipaul – these are writers known for having managed to migrate between languages, cultures, countries, continents, even civilisations. Their imaginations were fed by exile, a nourishment drawn not through roots but through rootlessness; mine, however, requires that I stay in the same city, on the same street, in the same house, gazing at the same view."
Read the given passage and answer the question that follows.
Here we come to the heart of the matter: I've never left Istanbul – never left the houses, streets and neighbourhoods of my childhood. Although I've lived in other districts from time to time, fifty years on I find myself back in the Pamuk Apartments, where my first photographs were taken and where my mother first held me in her arms to show me the world. I know this persistence owes something to my imaginary friend, and to the solace I took from the bond between us. But we live in an age defined by mass migration and creative immigrants, and so I am sometimes hard-pressed to explain why I've stayed not only in the same place, but the same building. My mother's sorrowful voice comes back to me, 'Why don't you go outside for a while, why don't you try a change of scene, do some travelling …?'
Conrad, Nabokov, Naipaul – these are writers known for having managed to migrate between languages, cultures, countries, continents, even civilisations. Their imaginations were fed by exile, a nourishment drawn not through roots but through rootlessness; mine, however, requires that I stay in the same city, on the same street, in the same house, gazing at the same view. Istanbul's fate is my fate: I am attached to this city because it has made me who I am.
Flaubert, who visited Istanbul a hundred and two years before my birth, was struck by the variety of life in its teeming streets; in one of his letters he predicted that in a century's time it would be the capital of the world. The reverse came true: after the Ottoman Empire collapsed, the world almost forgot that Istanbul existed. The city into which I was born was poorer, shabbier, and more isolated than it had ever been its two-thousand-year history. For me it has always been a city of ruins and of end-of-empire melancholy. I've spent my life either battling with this melancholy, or (like all Istanbullus) making it my own.
At least once in a lifetime, self-reflection leads us to examine the circumstances of our birth. Why were we born in this particular corner of the world, on this particular date? These families into which we were born, these countries and cities to which the lottery of life has assigned us – they expect love from us, and in the end, we do love them, from the bottom of our hearts – but did we perhaps deserve better? I sometimes think myself unlucky to have been born in an ageing and impoverished city buried under the ashes of a ruined empire. But a voice inside me always insists this was really a piece of luck.
Q. What can we infer from the passage about the author's regard for Istanbul?
The correct option 3. The author states his feelings in these sentences: 'I've never left Istanbul'; 'I am attached to this city because it has made me who I am'; '(like all Istanbullus) making it my own'; and 'a voice inside me always insists this was really a piece of luck'. All these lines suggest that he was proud of Instanbul as well as indicate how well he is attached to it.
Read the given passage and answer the question that follows.
Over the last few weeks, photographs of “red snow” around Ukraine’s Vernadsky Research Base, off the coast of Antarctica’s northernmost peninsula, have gone viral. “Red snow” or “watermelon” is a phenomenon that has been known since ancient times. Now, it raises concerns about climate change. (1) is believed to be one of the first to give a written account of red snow, over 2,000 years ago. In History of Animals, (1) wrote: “And, by the way, living animals are found in substances that are usually supposed to be incapable of putrefaction; for instance, worms are found in long-lying snow; and snow of this description gets reddish in colour, and the grub that is engendered in it is red, as might have been expected, and it is also hairy.” What (1) described as worms and grub, the scientific world today calls algae. The Greek philosopher was right: it is the algae that give the snow its red tinge. This alga species, Chlamydomonas Chlamydomonas nivalis, exists in snow in the polar and glacial regions, and carries a red pigment to keep itself warm. In turn, the red snow causes the surrounding ice to melt faster, a 2017 study from Alaska Pacific University said. The more the algae packed together, the redder the snow. And the darker the tinge, the more the heat absorbed by the snow. Subsequently, the ice melts faster. While the melt is good for the microbes that need the liquid water to survive and thrive, it’s bad for glaciers that are already melting from a myriad of other causes, the study said. These algae change the snow’s albedo — which refers to the amount of light or radiation the snow surface is able to reflect back. Changes in albedo lead to more melting. In the melting of snow in the Arctic, the key drivers have been snow and ice albedo, according to a 2016 study in the journal Nature.
Q. Which of the following is the branch of science dealing with the study of Algae?
Read the given passage and answer the question that follows.
Over the last few weeks, photographs of “red snow” around Ukraine’s Vernadsky Research Base, off the coast of Antarctica’s northernmost peninsula, have gone viral. “Red snow” or “watermelon” is a phenomenon that has been known since ancient times. Now, it raises concerns about climate change. (1) is believed to be one of the first to give a written account of red snow, over 2,000 years ago. In History of Animals, (1) wrote: “And, by the way, living animals are found in substances that are usually supposed to be incapable of putrefaction; for instance, worms are found in long-lying snow; and snow of this description gets reddish in colour, and the grub that is engendered in it is red, as might have been expected, and it is also hairy.” What (1) described as worms and grub, the scientific world today calls algae. The Greek philosopher was right: it is the algae that give the snow its red tinge. This alga species, Chlamydomonas Chlamydomonas nivalis, exists in snow in the polar and glacial regions, and carries a red pigment to keep itself warm. In turn, the red snow causes the surrounding ice to melt faster, a 2017 study from Alaska Pacific University said. The more the algae packed together, the redder the snow. And the darker the tinge, the more the heat absorbed by the snow. Subsequently, the ice melts faster. While the melt is good for the microbes that need the liquid water to survive and thrive, it’s bad for glaciers that are already melting from a myriad of other causes, the study said. These algae change the snow’s albedo — which refers to the amount of light or radiation the snow surface is able to reflect back. Changes in albedo lead to more melting. In the melting of snow in the Arctic, the key drivers have been snow and ice albedo, according to a 2016 study in the journal Nature.
Q. What is the first scientific base station of India situated in Antarctica?
Read the given passage and answer the question that follows.
Over the last few weeks, photographs of “red snow” around Ukraine’s Vernadsky Research Base, off the coast of Antarctica’s northernmost peninsula, have gone viral. “Red snow” or “watermelon” is a phenomenon that has been known since ancient times. Now, it raises concerns about climate change. (1) is believed to be one of the first to give a written account of red snow, over 2,000 years ago. In History of Animals, (1) wrote: “And, by the way, living animals are found in substances that are usually supposed to be incapable of putrefaction; for instance, worms are found in long-lying snow; and snow of this description gets reddish in colour, and the grub that is engendered in it is red, as might have been expected, and it is also hairy.” What (1) described as worms and grub, the scientific world today calls algae. The Greek philosopher was right: it is the algae that give the snow its red tinge. This alga species, Chlamydomonas Chlamydomonas nivalis, exists in snow in the polar and glacial regions, and carries a red pigment to keep itself warm. In turn, the red snow causes the surrounding ice to melt faster, a 2017 study from Alaska Pacific University said. The more the algae packed together, the redder the snow. And the darker the tinge, the more the heat absorbed by the snow. Subsequently, the ice melts faster. While the melt is good for the microbes that need the liquid water to survive and thrive, it’s bad for glaciers that are already melting from a myriad of other causes, the study said. These algae change the snow’s albedo — which refers to the amount of light or radiation the snow surface is able to reflect back. Changes in albedo lead to more melting. In the melting of snow in the Arctic, the key drivers have been snow and ice albedo, according to a 2016 study in the journal Nature.
Q. Which of the following is the highest peak in Antarctica?
Read the given passage and answer the question that follows.
Over the last few weeks, photographs of “red snow” around Ukraine’s Vernadsky Research Base, off the coast of Antarctica’s northernmost peninsula, have gone viral. “Red snow” or “watermelon” is a phenomenon that has been known since ancient times. Now, it raises concerns about climate change. (1) is believed to be one of the first to give a written account of red snow, over 2,000 years ago. In History of Animals, (1) wrote: “And, by the way, living animals are found in substances that are usually supposed to be incapable of putrefaction; for instance, worms are found in long-lying snow; and snow of this description gets reddish in colour, and the grub that is engendered in it is red, as might have been expected, and it is also hairy.” What (1) described as worms and grub, the scientific world today calls algae. The Greek philosopher was right: it is the algae that give the snow its red tinge. This alga species, Chlamydomonas Chlamydomonas nivalis, exists in snow in the polar and glacial regions, and carries a red pigment to keep itself warm. In turn, the red snow causes the surrounding ice to melt faster, a 2017 study from Alaska Pacific University said. The more the algae packed together, the redder the snow. And the darker the tinge, the more the heat absorbed by the snow. Subsequently, the ice melts faster. While the melt is good for the microbes that need the liquid water to survive and thrive, it’s bad for glaciers that are already melting from a myriad of other causes, the study said. These algae change the snow’s albedo — which refers to the amount of light or radiation the snow surface is able to reflect back. Changes in albedo lead to more melting. In the melting of snow in the Arctic, the key drivers have been snow and ice albedo, according to a 2016 study in the journal Nature.
Q. Which of the following is replaced by (1) in the passage?
Read the given passage and answer the question that follows.
Over the last few weeks, photographs of “red snow” around Ukraine’s Vernadsky Research Base, off the coast of Antarctica’s northernmost peninsula, have gone viral. “Red snow” or “watermelon” is a phenomenon that has been known since ancient times. Now, it raises concerns about climate change. (1) is believed to be one of the first to give a written account of red snow, over 2,000 years ago. In History of Animals, (1) wrote: “And, by the way, living animals are found in substances that are usually supposed to be incapable of putrefaction; for instance, worms are found in long-lying snow; and snow of this description gets reddish in colour, and the grub that is engendered in it is red, as might have been expected, and it is also hairy.” What (1) described as worms and grub, the scientific world today calls algae. The Greek philosopher was right: it is the algae that give the snow its red tinge. This alga species, Chlamydomonas Chlamydomonas nivalis, exists in snow in the polar and glacial regions, and carries a red pigment to keep itself warm. In turn, the red snow causes the surrounding ice to melt faster, a 2017 study from Alaska Pacific University said. The more the algae packed together, the redder the snow. And the darker the tinge, the more the heat absorbed by the snow. Subsequently, the ice melts faster. While the melt is good for the microbes that need the liquid water to survive and thrive, it’s bad for glaciers that are already melting from a myriad of other causes, the study said. These algae change the snow’s albedo — which refers to the amount of light or radiation the snow surface is able to reflect back. Changes in albedo lead to more melting. In the melting of snow in the Arctic, the key drivers have been snow and ice albedo, according to a 2016 study in the journal Nature.
Q. Red Sea got its name because of presence of similiar algae found in which country of the world?
Read the following passage and answer the question as directed.
On January 10, 2020, the Ministry of Steel launched the "__{X}__" programme, under which major PSUs like SAIL, IOCL and Coal India and concerned departments of five states; West Bengal, Andhra Pradesh, Chattisgarh, Jharkhand and Odisha will be working towards accelerating steel production in the region. For this purpose, the steel ministry is looking to invest $70 billion, that is likely to result in $35 billion addition to the GDP, and creation of 2.5 million jobs.
Kickstarting the programme here, the Minister of Petroleum and Natural Gas & Steel said that the idea was to promote the region as an integrated steel hub, which can foster growth towards a $5 trillion economy. Kolkata could be the epicentre of this growth, facilitated by Prime Minister Narendra Modi's visit to the city.
The steel ministry would anchor in coordinating various stake holders across central ministries, state governments and private investors, and a policy facilitating the creation of steel clusters has thus been put in place. Kalinganagar and Bokaro have been identified as pilot locations for steel clusters around integrated steel plants, and task forces and working groups with respective state governments have been formed for detailed planning for operationalisation of these clusters.
Prime Minister Narendra Modi, as part of his Kolkata visit, attended the 150th anniversary of Kolkata Port Trust. While Paradip Port Trust has been taken as a major stakeholder in the "__{X}__" initiative, Kolkata Port will play a vital role in developing inter-state connectivity through inland waterways.
Critical logistics and infrastructure projects were identified for expedition across the twelve major steel zones, which include rail, road and port capacity expansion projects. The twelve major steel zones identified are Kalinganagar, Angul, Rourkela, Jharsuguda, Nagarnar, Bhilai, Jamshedpur, Raipur, Bokaro, Durgapur, Kolkata and Vizag.
At a later stage, Bihar will be included within the cluster for driving growth in steel consumption, and that political differences between the centre and state governments, would not be a hindrance to uplift the 57 backward districts of the region.
According to the National Steel Policy announced in 2017, the government aims at a total production capacity of 300 million tonne by 2030-31 and out of which, around 200 million tonne is to be envisaged from the five eastern states.
Eastern region at present contributes 70% of the country's total steel production. This will go up to 87% with the integrated steel hub in place quotes by ___blank (i)___, IOCL Chairman and Managing Director. The oil and gas sector has been a major consumer of steel and the expansion would boost the steel industry thereby increasing the share of manufacturing to services.
Q. In the above passage, the name of the programme has been redacted with '__{X}__'. What is the name of the programme?
Purvodaya, an initiative in steel sector, is aimed at driving accelerated development of eastern India through establishment of integrated steel hub. Eastern states of India (Odisha, Jharkhand, Chhattisgarh, West Bengal) and the northern part of Andhra Pradesh collectively hold 80 per cent of the country's iron ore, 100 per cent of coking coal and significant portion of chromite, bauxite and dolomite reserves.
Read the following passage and answer the question as directed.
On January 10, 2020, the Ministry of Steel launched the "__{X}__" programme, under which major PSUs like SAIL, IOCL and Coal India and concerned departments of five states; West Bengal, Andhra Pradesh, Chattisgarh, Jharkhand and Odisha will be working towards accelerating steel production in the region. For this purpose, the steel ministry is looking to invest $70 billion, that is likely to result in $35 billion addition to the GDP, and creation of 2.5 million jobs.
Kickstarting the programme here, the Minister of Petroleum and Natural Gas & Steel said that the idea was to promote the region as an integrated steel hub, which can foster growth towards a $5 trillion economy. Kolkata could be the epicentre of this growth, facilitated by Prime Minister Narendra Modi's visit to the city.
The steel ministry would anchor in coordinating various stake holders across central ministries, state governments and private investors, and a policy facilitating the creation of steel clusters has thus been put in place. Kalinganagar and Bokaro have been identified as pilot locations for steel clusters around integrated steel plants, and task forces and working groups with respective state governments have been formed for detailed planning for operationalisation of these clusters.
Prime Minister Narendra Modi, as part of his Kolkata visit, attended the 150th anniversary of Kolkata Port Trust. While Paradip Port Trust has been taken as a major stakeholder in the "__{X}__" initiative, Kolkata Port will play a vital role in developing inter-state connectivity through inland waterways.
Critical logistics and infrastructure projects were identified for expedition across the twelve major steel zones, which include rail, road and port capacity expansion projects. The twelve major steel zones identified are Kalinganagar, Angul, Rourkela, Jharsuguda, Nagarnar, Bhilai, Jamshedpur, Raipur, Bokaro, Durgapur, Kolkata and Vizag.
At a later stage, Bihar will be included within the cluster for driving growth in steel consumption, and that political differences between the centre and state governments, would not be a hindrance to uplift the 57 backward districts of the region.
According to the National Steel Policy announced in 2017, the government aims at a total production capacity of 300 million tonne by 2030-31 and out of which, around 200 million tonne is to be envisaged from the five eastern states.
Eastern region at present contributes 70% of the country's total steel production. This will go up to 87% with the integrated steel hub in place quotes by ___blank (i)___, IOCL Chairman and Managing Director. The oil and gas sector has been a major consumer of steel and the expansion would boost the steel industry thereby increasing the share of manufacturing to services.
Q. In the above passage, whose name has been redacted with ___ blank (i)___?
Sanjiv Singh has taken over as Chairman of Indian Oil Corporation (Indian Oil) with effect from 1st June 2017. Prior to his elevation, he was Director (Refineries) of the Indian Oil Board since July 2014. A chemical engineer from IIT-Roorkee with a Diploma in Management, Mr. Sanjiv Singh, joined Indian Oil in 1981 and has served the Corporation for over 35 years.
Read the following passage and answer the question as directed.
On January 10, 2020, the Ministry of Steel launched the "__{X}__" programme, under which major PSUs like SAIL, IOCL and Coal India and concerned departments of five states; West Bengal, Andhra Pradesh, Chattisgarh, Jharkhand and Odisha will be working towards accelerating steel production in the region. For this purpose, the steel ministry is looking to invest $70 billion, that is likely to result in $35 billion addition to the GDP, and creation of 2.5 million jobs.
Kickstarting the programme here, the Minister of Petroleum and Natural Gas & Steel said that the idea was to promote the region as an integrated steel hub, which can foster growth towards a $5 trillion economy. Kolkata could be the epicentre of this growth, facilitated by Prime Minister Narendra Modi's visit to the city.
The steel ministry would anchor in coordinating various stake holders across central ministries, state governments and private investors, and a policy facilitating the creation of steel clusters has thus been put in place. Kalinganagar and Bokaro have been identified as pilot locations for steel clusters around integrated steel plants, and task forces and working groups with respective state governments have been formed for detailed planning for operationalisation of these clusters.
Prime Minister Narendra Modi, as part of his Kolkata visit, attended the 150th anniversary of Kolkata Port Trust. While Paradip Port Trust has been taken as a major stakeholder in the "__{X}__" initiative, Kolkata Port will play a vital role in developing inter-state connectivity through inland waterways.
Critical logistics and infrastructure projects were identified for expedition across the twelve major steel zones, which include rail, road and port capacity expansion projects. The twelve major steel zones identified are Kalinganagar, Angul, Rourkela, Jharsuguda, Nagarnar, Bhilai, Jamshedpur, Raipur, Bokaro, Durgapur, Kolkata and Vizag.
At a later stage, Bihar will be included within the cluster for driving growth in steel consumption, and that political differences between the centre and state governments, would not be a hindrance to uplift the 57 backward districts of the region.
According to the National Steel Policy announced in 2017, the government aims at a total production capacity of 300 million tonne by 2030-31 and out of which, around 200 million tonne is to be envisaged from the five eastern states.
Eastern region at present contributes 70% of the country's total steel production. This will go up to 87% with the integrated steel hub in place quotes by ___blank (i)___, IOCL Chairman and Managing Director. The oil and gas sector has been a major consumer of steel and the expansion would boost the steel industry thereby increasing the share of manufacturing to services.
Q. After reading the above passage, consider the following statements and choose the correct option.
Statement I: '__{X}__' aims at achieving India's steel policy target of 300 million tonne by 2030.
Statement II: Jharkhand is the largest producer of steel in India.
Statement III: '__{X}__' aims to promote Jharkhand as an integrated steel hub.
Statement IV: '__{X}__' would contribute towards the transformation of logistics and utilities infrastructure.
Odisha is the largest producer of steel in India.
Minister of Petroleum and Natural Gas & Steel Shri Dharmendra Pradhan launched Purvodaya for an accelerated development of eastern India through integrated steel hub in Kolkata, West Bengal.
The Integrated Steel Hub would focus on 3 key elements:
1. Capacity addition through easing the setup of Greenfield steel plants
2. Development of steel clusters near integrated steel plants as well as demand centres
3. Transformation of logistics and utilities infrastructure which would change the socio-economic landscape in the east
Read the following passage and answer the question as directed.
On January 10, 2020, the Ministry of Steel launched the "__{X}__" programme, under which major PSUs like SAIL, IOCL and Coal India and concerned departments of five states; West Bengal, Andhra Pradesh, Chattisgarh, Jharkhand and Odisha will be working towards accelerating steel production in the region. For this purpose, the steel ministry is looking to invest $70 billion, that is likely to result in $35 billion addition to the GDP, and creation of 2.5 million jobs.
Kickstarting the programme here, the Minister of Petroleum and Natural Gas & Steel said that the idea was to promote the region as an integrated steel hub, which can foster growth towards a $5 trillion economy. Kolkata could be the epicentre of this growth, facilitated by Prime Minister Narendra Modi's visit to the city.
The steel ministry would anchor in coordinating various stake holders across central ministries, state governments and private investors, and a policy facilitating the creation of steel clusters has thus been put in place. Kalinganagar and Bokaro have been identified as pilot locations for steel clusters around integrated steel plants, and task forces and working groups with respective state governments have been formed for detailed planning for operationalisation of these clusters.
Prime Minister Narendra Modi, as part of his Kolkata visit, attended the 150th anniversary of Kolkata Port Trust. While Paradip Port Trust has been taken as a major stakeholder in the "__{X}__" initiative, Kolkata Port will play a vital role in developing inter-state connectivity through inland waterways.
Critical logistics and infrastructure projects were identified for expedition across the twelve major steel zones, which include rail, road and port capacity expansion projects. The twelve major steel zones identified are Kalinganagar, Angul, Rourkela, Jharsuguda, Nagarnar, Bhilai, Jamshedpur, Raipur, Bokaro, Durgapur, Kolkata and Vizag.
At a later stage, Bihar will be included within the cluster for driving growth in steel consumption, and that political differences between the centre and state governments, would not be a hindrance to uplift the 57 backward districts of the region.
According to the National Steel Policy announced in 2017, the government aims at a total production capacity of 300 million tonne by 2030-31 and out of which, around 200 million tonne is to be envisaged from the five eastern states.
Eastern region at present contributes 70% of the country's total steel production. This will go up to 87% with the integrated steel hub in place quotes by ___blank (i)___, IOCL Chairman and Managing Director. The oil and gas sector has been a major consumer of steel and the expansion would boost the steel industry thereby increasing the share of manufacturing to services.
Q. Who holds the chair of the Minister of Petroleum and Natural Gas & Steel as in 2020?
Dharmendra Pradhan is the Cabinet Minister for Petroleum and Natural Gas & Steel as in 2020. On 31st May 2019, Shri Pradhan began his second consecutive tenure at the Ministry of Petroleum and Natural Gas & Steel becoming the first such incumbent in the history of independent India. Shri Pradhan was promoted as a Cabinet Minister on September 3, 2017. As a member of Parliament, Shri Pradhan represents Madhya Pradesh in the Rajya Sabha and was earlier a member of the 14th Lok Sabha. Born on 26th June 1969, he hails from the city of Talcher in Odisha.
Read the following passage and answer the question as directed.
On January 10, 2020, the Ministry of Steel launched the "__{X}__" programme, under which major PSUs like SAIL, IOCL and Coal India and concerned departments of five states; West Bengal, Andhra Pradesh, Chattisgarh, Jharkhand and Odisha will be working towards accelerating steel production in the region. For this purpose, the steel ministry is looking to invest $70 billion, that is likely to result in $35 billion addition to the GDP, and creation of 2.5 million jobs.
Kickstarting the programme here, the Minister of Petroleum and Natural Gas & Steel said that the idea was to promote the region as an integrated steel hub, which can foster growth towards a $5 trillion economy. Kolkata could be the epicentre of this growth, facilitated by Prime Minister Narendra Modi's visit to the city.
The steel ministry would anchor in coordinating various stake holders across central ministries, state governments and private investors, and a policy facilitating the creation of steel clusters has thus been put in place. Kalinganagar and Bokaro have been identified as pilot locations for steel clusters around integrated steel plants, and task forces and working groups with respective state governments have been formed for detailed planning for operationalisation of these clusters.
Prime Minister Narendra Modi, as part of his Kolkata visit, attended the 150th anniversary of Kolkata Port Trust. While Paradip Port Trust has been taken as a major stakeholder in the "__{X}__" initiative, Kolkata Port will play a vital role in developing inter-state connectivity through inland waterways.
Critical logistics and infrastructure projects were identified for expedition across the twelve major steel zones, which include rail, road and port capacity expansion projects. The twelve major steel zones identified are Kalinganagar, Angul, Rourkela, Jharsuguda, Nagarnar, Bhilai, Jamshedpur, Raipur, Bokaro, Durgapur, Kolkata and Vizag.
At a later stage, Bihar will be included within the cluster for driving growth in steel consumption, and that political differences between the centre and state governments, would not be a hindrance to uplift the 57 backward districts of the region.
According to the National Steel Policy announced in 2017, the government aims at a total production capacity of 300 million tonne by 2030-31 and out of which, around 200 million tonne is to be envisaged from the five eastern states.
Eastern region at present contributes 70% of the country's total steel production. This will go up to 87% with the integrated steel hub in place quotes by ___blank (i)___, IOCL Chairman and Managing Director. The oil and gas sector has been a major consumer of steel and the expansion would boost the steel industry thereby increasing the share of manufacturing to services.
Q. Where is Indian Oil Corporation Limited headquartered?
Indian Oil Corporation Limited, commonly known as Indian Oil, is an Indian government owned oil and gas company headquartered in New Delhi. It is the largest commercial oil company in the country. It has subsidiaries in Sri Lanka (Lanka IOC), Mauritius (Mauritius Ltd.) and the Middle East (IOC Middle East FZE).
Read the given passage and answer the question that follows.
World leaders made a fresh push for peace in Libya at a summit in (1) on Sunday, in a desperate bid to stop the conflict-wracked nation from turning into a “second Syria”. The Presidents of Russia, Turkey and France joined other global chiefs at the talks hosted by Chancellor Angela Merkel and held under the auspices of the UN. The summit’s main goal is to get foreign powers wielding influence in the region to stop interfering in the war — be it through weapons, troops or financing. Haftar in Leaders of both warring factions — strongman Khalifa Haftar and the head of Tripoli’s UN-recognised government Fayez-al-Sarraj — were also in (1) for the first such gathering since 2018. But pro-Haftar forces upped the ante ahead of the talks by blocking oil exports at Libya’s key ports, crippling the country’s main income source in protest at Turkey’s decision to send troops to shore up Mr. Sarraj’s Tripoli-based Government of National Accord (GNA). Ahead of the talks, Turkish President (2) lashed out at Mr. Haftar, saying he needed to drop his “hostile attitude” if Libya is to have any chance at winning peace. The flaring oil crisis underlined the devastating impact of foreign influence in the conflict, in which Mr. Sarraj’s GNA is backed by Turkey and Qatar while Mr. Haftar has the support of Russia, (3) and the United Arab Emirates. The UN hopes all sides will sign up to a plan to refrain from interference, and commit to a truce.
Q. In which year colonel Muammar Gaddafi toppled following an armed rebellion assisted by Western military intervention in Libya.?
Read the given passage and answer the question that follows.
World leaders made a fresh push for peace in Libya at a summit in (1) on Sunday, in a desperate bid to stop the conflict-wracked nation from turning into a “second Syria”. The Presidents of Russia, Turkey and France joined other global chiefs at the talks hosted by Chancellor Angela Merkel and held under the auspices of the UN. The summit’s main goal is to get foreign powers wielding influence in the region to stop interfering in the war — be it through weapons, troops or financing. Haftar in Leaders of both warring factions — strongman Khalifa Haftar and the head of Tripoli’s UN-recognised government Fayez-al-Sarraj — were also in (1) for the first such gathering since 2018. But pro-Haftar forces upped the ante ahead of the talks by blocking oil exports at Libya’s key ports, crippling the country’s main income source in protest at Turkey’s decision to send troops to shore up Mr. Sarraj’s Tripoli-based Government of National Accord (GNA). Ahead of the talks, Turkish President (2) lashed out at Mr. Haftar, saying he needed to drop his “hostile attitude” if Libya is to have any chance at winning peace. The flaring oil crisis underlined the devastating impact of foreign influence in the conflict, in which Mr. Sarraj’s GNA is backed by Turkey and Qatar while Mr. Haftar has the support of Russia, (3) and the United Arab Emirates. The UN hopes all sides will sign up to a plan to refrain from interference, and commit to a truce.
Q. Which of the following is replaced by (2) in the above passage?
Read the given passage and answer the question that follows.
World leaders made a fresh push for peace in Libya at a summit in (1) on Sunday, in a desperate bid to stop the conflict-wracked nation from turning into a “second Syria”. The Presidents of Russia, Turkey and France joined other global chiefs at the talks hosted by Chancellor Angela Merkel and held under the auspices of the UN. The summit’s main goal is to get foreign powers wielding influence in the region to stop interfering in the war — be it through weapons, troops or financing. Haftar in Leaders of both warring factions — strongman Khalifa Haftar and the head of Tripoli’s UN-recognised government Fayez-al-Sarraj — were also in (1) for the first such gathering since 2018. But pro-Haftar forces upped the ante ahead of the talks by blocking oil exports at Libya’s key ports, crippling the country’s main income source in protest at Turkey’s decision to send troops to shore up Mr. Sarraj’s Tripoli-based Government of National Accord (GNA). Ahead of the talks, Turkish President (2) lashed out at Mr. Haftar, saying he needed to drop his “hostile attitude” if Libya is to have any chance at winning peace. The flaring oil crisis underlined the devastating impact of foreign influence in the conflict, in which Mr. Sarraj’s GNA is backed by Turkey and Qatar while Mr. Haftar has the support of Russia, (3) and the United Arab Emirates. The UN hopes all sides will sign up to a plan to refrain from interference, and commit to a truce.
Q. Which of the following is replaced by (1) in the above passage?
Read the given passage and answer the question that follows.
World leaders made a fresh push for peace in Libya at a summit in (1) on Sunday, in a desperate bid to stop the conflict-wracked nation from turning into a “second Syria”. The Presidents of Russia, Turkey and France joined other global chiefs at the talks hosted by Chancellor Angela Merkel and held under the auspices of the UN. The summit’s main goal is to get foreign powers wielding influence in the region to stop interfering in the war — be it through weapons, troops or financing. Haftar in Leaders of both warring factions — strongman Khalifa Haftar and the head of Tripoli’s UN-recognised government Fayez-al-Sarraj — were also in (1) for the first such gathering since 2018. But pro-Haftar forces upped the ante ahead of the talks by blocking oil exports at Libya’s key ports, crippling the country’s main income source in protest at Turkey’s decision to send troops to shore up Mr. Sarraj’s Tripoli-based Government of National Accord (GNA). Ahead of the talks, Turkish President (2) lashed out at Mr. Haftar, saying he needed to drop his “hostile attitude” if Libya is to have any chance at winning peace. The flaring oil crisis underlined the devastating impact of foreign influence in the conflict, in which Mr. Sarraj’s GNA is backed by Turkey and Qatar while Mr. Haftar has the support of Russia, (3) and the United Arab Emirates. The UN hopes all sides will sign up to a plan to refrain from interference, and commit to a truce.
Q. Which of the following is replaced by (3) in the passage?
Read the given passage and answer the question that follows.
World leaders made a fresh push for peace in Libya at a summit in (1) on Sunday, in a desperate bid to stop the conflict-wracked nation from turning into a “second Syria”. The Presidents of Russia, Turkey and France joined other global chiefs at the talks hosted by Chancellor Angela Merkel and held under the auspices of the UN. The summit’s main goal is to get foreign powers wielding influence in the region to stop interfering in the war — be it through weapons, troops or financing. Haftar in Leaders of both warring factions — strongman Khalifa Haftar and the head of Tripoli’s UN-recognised government Fayez-al-Sarraj — were also in (1) for the first such gathering since 2018. But pro-Haftar forces upped the ante ahead of the talks by blocking oil exports at Libya’s key ports, crippling the country’s main income source in protest at Turkey’s decision to send troops to shore up Mr. Sarraj’s Tripoli-based Government of National Accord (GNA). Ahead of the talks, Turkish President (2) lashed out at Mr. Haftar, saying he needed to drop his “hostile attitude” if Libya is to have any chance at winning peace. The flaring oil crisis underlined the devastating impact of foreign influence in the conflict, in which Mr. Sarraj’s GNA is backed by Turkey and Qatar while Mr. Haftar has the support of Russia, (3) and the United Arab Emirates. The UN hopes all sides will sign up to a plan to refrain from interference, and commit to a truce.
Q. Which of the following countries doesn’t have land border with Libya?
Read the following passage and answer the question as directed.
{X} became the first state to challenge the Citizenship (Amendment) Act (CAA) before the Supreme Court. However, the legal route adopted by the state is different from the 60 petitions already pending before the court. The {X} government has moved the apex court under {Y} of the Constitution, the provision under which the Supreme Court has original jurisdiction to deal with any dispute between the Centre and a state; the Centre and a state on the one side and another state on the other side; and two or more states.
The Chhattisgarh government filed a suit in the Supreme Court under {Y}, challenging the National Investigation Agency (NIA) Act on the ground that it encroaches upon the state's powers to maintain law and order.
For a dispute to qualify as a dispute under {Y}, it has to be necessarily between states and the Centre, and must involve a question of law or fact on which the existence of a legal right of the state or the Centre depends.
{Y} cannot be used to settle political differences between state and central governments headed by different parties.
The Centre has other powers to ensure that its laws are implemented. The Centre can issue directions to a state to implement the laws made by Parliament. If states do not comply with the directions, the Centre can move the court seeking a permanent injunction against the states to force them to comply with the law. Non-compliance of court orders can result in contempt of court, and the court usually hauls up the chief secretaries of the states responsible for implementing laws.
The other petitions challenging the CAA have been filed under Article 32 of the Constitution. A state government cannot move the court under this provision because only people and citizens can claim fundamental rights.
Under {Y}, the challenge is made when the rights and power of a state or the Centre are in question.
However, the relief that the state (under {Y}) and petitioners under Article 32 have sought in the challenge to the CAA is the same — declaration of the law as being unconstitutional.
Q. In the above passage, what has been redacted with {X}?
Kerala became the first state to challenge the Contentious Citizenship (Amendment) Act or CAA after it moved to the Supreme Court. The plea has been filed as an original suit under Article 131 of the Constitution which empowers Supreme Court to hear disputes between the Government of India and one or more states.
Read the following passage and answer the question as directed.
{X} became the first state to challenge the Citizenship (Amendment) Act (CAA) before the Supreme Court. However, the legal route adopted by the state is different from the 60 petitions already pending before the court. The {X} government has moved the apex court under {Y} of the Constitution, the provision under which the Supreme Court has original jurisdiction to deal with any dispute between the Centre and a state; the Centre and a state on the one side and another state on the other side; and two or more states.
The Chhattisgarh government filed a suit in the Supreme Court under {Y}, challenging the National Investigation Agency (NIA) Act on the ground that it encroaches upon the state's powers to maintain law and order.
For a dispute to qualify as a dispute under {Y}, it has to be necessarily between states and the Centre, and must involve a question of law or fact on which the existence of a legal right of the state or the Centre depends.
{Y} cannot be used to settle political differences between state and central governments headed by different parties.
The Centre has other powers to ensure that its laws are implemented. The Centre can issue directions to a state to implement the laws made by Parliament. If states do not comply with the directions, the Centre can move the court seeking a permanent injunction against the states to force them to comply with the law. Non-compliance of court orders can result in contempt of court, and the court usually hauls up the chief secretaries of the states responsible for implementing laws.
The other petitions challenging the CAA have been filed under Article 32 of the Constitution. A state government cannot move the court under this provision because only people and citizens can claim fundamental rights.
Under {Y}, the challenge is made when the rights and power of a state or the Centre are in question.
However, the relief that the state (under {Y}) and petitioners under Article 32 have sought in the challenge to the CAA is the same — declaration of the law as being unconstitutional.
Q. In the above passage, which article has been redacted with {Y}?
In the above passage, Article 131 has been redacted with {Y}. It states that:
Original jurisdiction of the Supreme Court: Subject to the provisions of this Constitution the Supreme Court shall, to the exclusion of any other court, have original jurisdiction in any dispute: (a) between the Government of India and one or more states (b) between the Government of India and any state or states on one side and one or more other states on the other or (c) between two or more states.
Read the following passage and answer the question as directed.
{X} became the first state to challenge the Citizenship (Amendment) Act (CAA) before the Supreme Court. However, the legal route adopted by the state is different from the 60 petitions already pending before the court. The {X} government has moved the apex court under {Y} of the Constitution, the provision under which the Supreme Court has original jurisdiction to deal with any dispute between the Centre and a state; the Centre and a state on the one side and another state on the other side; and two or more states.
The Chhattisgarh government filed a suit in the Supreme Court under {Y}, challenging the National Investigation Agency (NIA) Act on the ground that it encroaches upon the state's powers to maintain law and order.
For a dispute to qualify as a dispute under {Y}, it has to be necessarily between states and the Centre, and must involve a question of law or fact on which the existence of a legal right of the state or the Centre depends.
{Y} cannot be used to settle political differences between state and central governments headed by different parties.
The Centre has other powers to ensure that its laws are implemented. The Centre can issue directions to a state to implement the laws made by Parliament. If states do not comply with the directions, the Centre can move the court seeking a permanent injunction against the states to force them to comply with the law. Non-compliance of court orders can result in contempt of court, and the court usually hauls up the chief secretaries of the states responsible for implementing laws.
The other petitions challenging the CAA have been filed under Article 32 of the Constitution. A state government cannot move the court under this provision because only people and citizens can claim fundamental rights.
Under {Y}, the challenge is made when the rights and power of a state or the Centre are in question.
However, the relief that the state (under {Y}) and petitioners under Article 32 have sought in the challenge to the CAA is the same — declaration of the law as being unconstitutional.
Q. Statement I: Article 32 of the Constitution gives court the power to issue writs when fundamental rights are violated.
Statement II: {Y} cannot be used to settle political differences between state and central governments headed by different parties.
Statement I: Article 32 of the Constitution of India confers power on the Supreme Court to issue direction or order or writ, including writs in the nature of habeas corpus, mandamus, prohibition, quo warranto and certiorari, whichever may be appropriate, for the enforcement of any of the rights conferred by Part III of the Constitution.
Statement II: Article 131 can be used to settle political differences between state and central governments headed by different parties.
Read the following passage and answer the question as directed.
{X} became the first state to challenge the Citizenship (Amendment) Act (CAA) before the Supreme Court. However, the legal route adopted by the state is different from the 60 petitions already pending before the court. The {X} government has moved the apex court under {Y} of the Constitution, the provision under which the Supreme Court has original jurisdiction to deal with any dispute between the Centre and a state; the Centre and a state on the one side and another state on the other side; and two or more states.
The Chhattisgarh government filed a suit in the Supreme Court under {Y}, challenging the National Investigation Agency (NIA) Act on the ground that it encroaches upon the state's powers to maintain law and order.
For a dispute to qualify as a dispute under {Y}, it has to be necessarily between states and the Centre, and must involve a question of law or fact on which the existence of a legal right of the state or the Centre depends.
{Y} cannot be used to settle political differences between state and central governments headed by different parties.
The Centre has other powers to ensure that its laws are implemented. The Centre can issue directions to a state to implement the laws made by Parliament. If states do not comply with the directions, the Centre can move the court seeking a permanent injunction against the states to force them to comply with the law. Non-compliance of court orders can result in contempt of court, and the court usually hauls up the chief secretaries of the states responsible for implementing laws.
The other petitions challenging the CAA have been filed under Article 32 of the Constitution. A state government cannot move the court under this provision because only people and citizens can claim fundamental rights.
Under {Y}, the challenge is made when the rights and power of a state or the Centre are in question.
However, the relief that the state (under {Y}) and petitioners under Article 32 have sought in the challenge to the CAA is the same — declaration of the law as being unconstitutional.
Q. How many kinds of jurisdictions does Supreme Court have?
The Supreme Court has three kinds of jurisdictions: original, appellate and advisory.
Under its advisory jurisdiction, the President has the power to seek an opinion from the apex court under Article 143 of the Constitution.
Under its appellate jurisdiction, the Supreme Court hears appeals from lower courts.
In its extraordinary original jurisdiction, the Supreme Court has exclusive power to adjudicate upon disputes involving elections of the President and the Vice President, those that involve states and the Centre, and cases involving the violation of fundamental rights.
Read the following passage and answer the question as directed.
{X} became the first state to challenge the Citizenship (Amendment) Act (CAA) before the Supreme Court. However, the legal route adopted by the state is different from the 60 petitions already pending before the court. The {X} government has moved the apex court under {Y} of the Constitution, the provision under which the Supreme Court has original jurisdiction to deal with any dispute between the Centre and a state; the Centre and a state on the one side and another state on the other side; and two or more states.
The Chhattisgarh government filed a suit in the Supreme Court under {Y}, challenging the National Investigation Agency (NIA) Act on the ground that it encroaches upon the state's powers to maintain law and order.
For a dispute to qualify as a dispute under {Y}, it has to be necessarily between states and the Centre, and must involve a question of law or fact on which the existence of a legal right of the state or the Centre depends.
{Y} cannot be used to settle political differences between state and central governments headed by different parties.
The Centre has other powers to ensure that its laws are implemented. The Centre can issue directions to a state to implement the laws made by Parliament. If states do not comply with the directions, the Centre can move the court seeking a permanent injunction against the states to force them to comply with the law. Non-compliance of court orders can result in contempt of court, and the court usually hauls up the chief secretaries of the states responsible for implementing laws.
The other petitions challenging the CAA have been filed under Article 32 of the Constitution. A state government cannot move the court under this provision because only people and citizens can claim fundamental rights.
Under {Y}, the challenge is made when the rights and power of a state or the Centre are in question.
However, the relief that the state (under {Y}) and petitioners under Article 32 have sought in the challenge to the CAA is the same — declaration of the law as being unconstitutional.
Q. How many kinds of writs are there in the Indian Constitution?
Supreme Court under Article 32 and High Courts under Article 226 are empowered to issue writs for the enforcement of Fundamental Rights. There are five types of writs in the Indian Constitution:
Habeas Corpus, Certiorari, Quo-Warranto, Mandamus and Prohibition
Read the given passage and answer the question that follows.
The United States will be signing the peace deal with the Taliban after nearly two years of negotiations in (1) The United States is all set to sign a peace agreement with the Taliban in Afghanistan, bringing an end to America’s longest war after nearly two years of negotiations. US President Donald Trump made the announcement. US Secretary of State (2) will witness the signing of the historic peace agreement with the Taliban representatives. Trump stated that the peace deal is being signed as a part of America’s efforts to bring lasting peace in war-torn Afghanistan. He further stated that if the Taliban and the Afghanistan government live upto their commitments, then it will pave the way forward for the US to bring their troops back home. US Secretary of Defence Mark Esper will be issuing a joint declaration with the Afghan government after signing of the peace agreement. The United States will be signing the peace deal with the Taliban after nearly two years of negotiations in Qatar’s capital city, Doha. Diplomats from nations including the US, Afghanistan, India, Pakistan and other UN member states have already begun arriving in (1). Following the signing, the US is expected to withdraw some of its troops, reducing its troop strength in the nation to about 8600 from earlier 13000. Further call backs will be issued depending upon compliance from the Taliban. The US-Taliban peace deal will also bring an end to the massive investment of money and lives, as the US has spent over $750 billion and tens of thousands human lives have been lost or permanently scarred and interrupted on all sides in the decades-long war.
Q. Which of the following is replaced by (1) in the passage?
Read the given passage and answer the question that follows.
The United States will be signing the peace deal with the Taliban after nearly two years of negotiations in (1) The United States is all set to sign a peace agreement with the Taliban in Afghanistan, bringing an end to America’s longest war after nearly two years of negotiations. US President Donald Trump made the announcement. US Secretary of State (2) will witness the signing of the historic peace agreement with the Taliban representatives. Trump stated that the peace deal is being signed as a part of America’s efforts to bring lasting peace in war-torn Afghanistan. He further stated that if the Taliban and the Afghanistan government live upto their commitments, then it will pave the way forward for the US to bring their troops back home. US Secretary of Defence Mark Esper will be issuing a joint declaration with the Afghan government after signing of the peace agreement. The United States will be signing the peace deal with the Taliban after nearly two years of negotiations in Qatar’s capital city, Doha. Diplomats from nations including the US, Afghanistan, India, Pakistan and other UN member states have already begun arriving in (1). Following the signing, the US is expected to withdraw some of its troops, reducing its troop strength in the nation to about 8600 from earlier 13000. Further call backs will be issued depending upon compliance from the Taliban. The US-Taliban peace deal will also bring an end to the massive investment of money and lives, as the US has spent over $750 billion and tens of thousands human lives have been lost or permanently scarred and interrupted on all sides in the decades-long war.
Q. Who is the present president of Afghanistan?
Read the given passage and answer the question that follows.
The United States will be signing the peace deal with the Taliban after nearly two years of negotiations in (1) The United States is all set to sign a peace agreement with the Taliban in Afghanistan, bringing an end to America’s longest war after nearly two years of negotiations. US President Donald Trump made the announcement. US Secretary of State (2) will witness the signing of the historic peace agreement with the Taliban representatives. Trump stated that the peace deal is being signed as a part of America’s efforts to bring lasting peace in war-torn Afghanistan. He further stated that if the Taliban and the Afghanistan government live upto their commitments, then it will pave the way forward for the US to bring their troops back home. US Secretary of Defence Mark Esper will be issuing a joint declaration with the Afghan government after signing of the peace agreement. The United States will be signing the peace deal with the Taliban after nearly two years of negotiations in Qatar’s capital city, Doha. Diplomats from nations including the US, Afghanistan, India, Pakistan and other UN member states have already begun arriving in (1). Following the signing, the US is expected to withdraw some of its troops, reducing its troop strength in the nation to about 8600 from earlier 13000. Further call backs will be issued depending upon compliance from the Taliban. The US-Taliban peace deal will also bring an end to the massive investment of money and lives, as the US has spent over $750 billion and tens of thousands human lives have been lost or permanently scarred and interrupted on all sides in the decades-long war.
Q. In which year US had started Afghanistan war?
Read the given passage and answer the question that follows.
The United States will be signing the peace deal with the Taliban after nearly two years of negotiations in (1) The United States is all set to sign a peace agreement with the Taliban in Afghanistan, bringing an end to America’s longest war after nearly two years of negotiations. US President Donald Trump made the announcement. US Secretary of State (2) will witness the signing of the historic peace agreement with the Taliban representatives. Trump stated that the peace deal is being signed as a part of America’s efforts to bring lasting peace in war-torn Afghanistan. He further stated that if the Taliban and the Afghanistan government live upto their commitments, then it will pave the way forward for the US to bring their troops back home. US Secretary of Defence Mark Esper will be issuing a joint declaration with the Afghan government after signing of the peace agreement. The United States will be signing the peace deal with the Taliban after nearly two years of negotiations in Qatar’s capital city, Doha. Diplomats from nations including the US, Afghanistan, India, Pakistan and other UN member states have already begun arriving in (1). Following the signing, the US is expected to withdraw some of its troops, reducing its troop strength in the nation to about 8600 from earlier 13000. Further call backs will be issued depending upon compliance from the Taliban. The US-Taliban peace deal will also bring an end to the massive investment of money and lives, as the US has spent over $750 billion and tens of thousands human lives have been lost or permanently scarred and interrupted on all sides in the decades-long war.
Q. Which of the following is replaced by (2) in the passage?
Read the given passage and answer the question that follows.
The United States will be signing the peace deal with the Taliban after nearly two years of negotiations in (1) The United States is all set to sign a peace agreement with the Taliban in Afghanistan, bringing an end to America’s longest war after nearly two years of negotiations. US President Donald Trump made the announcement. US Secretary of State (2) will witness the signing of the historic peace agreement with the Taliban representatives. Trump stated that the peace deal is being signed as a part of America’s efforts to bring lasting peace in war-torn Afghanistan. He further stated that if the Taliban and the Afghanistan government live upto their commitments, then it will pave the way forward for the US to bring their troops back home. US Secretary of Defence Mark Esper will be issuing a joint declaration with the Afghan government after signing of the peace agreement. The United States will be signing the peace deal with the Taliban after nearly two years of negotiations in Qatar’s capital city, Doha. Diplomats from nations including the US, Afghanistan, India, Pakistan and other UN member states have already begun arriving in (1). Following the signing, the US is expected to withdraw some of its troops, reducing its troop strength in the nation to about 8600 from earlier 13000. Further call backs will be issued depending upon compliance from the Taliban. The US-Taliban peace deal will also bring an end to the massive investment of money and lives, as the US has spent over $750 billion and tens of thousands human lives have been lost or permanently scarred and interrupted on all sides in the decades-long war.
Q. What was the code name of operation for Taliban attacks by US?
Read the following passage and answer the question as directed.
Greta Thunberg, the teen activist from {X} who has urged immediate action to address a global climate crisis, was named Time magazine's person of the year for 2019. Thunberg, a 16 year old, was lauded by Time for starting an environmental campaign in August 2018 which became a global movement, initially from skipping school to camping out in front of the country parliament to demand action.
"In the 16 months, she addressed heads of state at the UN, met with the pope, sparred with the president of the United States and inspired 4 million people to join the global climate strike on September 20, 2019, which was the largest climate demonstration in human history," the magazine said.
"Margaret Atwood compared her to Joan of Arc. After noticing a hundredfold increase in its usage, lexicographers at Collins Dictionary named Thunberg's pioneering idea, climate strike, the word of the year," Time said.
Thunberg, who turned 17 in January, 2020 continues to beat the drum, saying in {Y} that the voices of climate strikers are being heard but politicians are still not taking action.
The activist addressed the latest round of the UN climate meeting in December 2019 held in {Y}, bluntly criticising world leaders for "negotiating loopholes" and using PR to make it appear they are achieving bold climate targets.
But the reluctant celebrity, who has been chased by cameras and attracted large crowds at the {Y} conference, has urged the press to hear from other activists and indigenous youth instead.
The former vice-president Al Gore, a longtime environmentalist, said the magazine made a "brilliant choice", which made Thunberg the youngest person to have ever received the accolade.
"Greta embodies the moral authority of the youth activist movement demanding that we act immediately to solve the climate crisis. She is an inspiration to me and to people across the world," Gore said.
Other nominees for person of the year included Donald Trump and the House speaker, Nancy Pelosi, the top Democrat in Congress.
Q. In the above passage, what has been redacted with {X}?
Sweden has been redacted with {X}. Greta Tintin Eleonora Ernman Thunberg, born on 3rd January, 2003, is a Swedish environmental activist on climate change whose campaigning has gained international recognition. Thunberg is known for her straightforward speaking manner, both in public and to political leaders and assemblies, in which she urges immediate action to address the climate crisis.
Read the following passage and answer the question as directed.
Greta Thunberg, the teen activist from {X} who has urged immediate action to address a global climate crisis, was named Time magazine's person of the year for 2019. Thunberg, a 16 year old, was lauded by Time for starting an environmental campaign in August 2018 which became a global movement, initially from skipping school to camping out in front of the country parliament to demand action.
"In the 16 months, she addressed heads of state at the UN, met with the pope, sparred with the president of the United States and inspired 4 million people to join the global climate strike on September 20, 2019, which was the largest climate demonstration in human history," the magazine said.
"Margaret Atwood compared her to Joan of Arc. After noticing a hundredfold increase in its usage, lexicographers at Collins Dictionary named Thunberg's pioneering idea, climate strike, the word of the year," Time said.
Thunberg, who turned 17 in January, 2020 continues to beat the drum, saying in {Y} that the voices of climate strikers are being heard but politicians are still not taking action.
The activist addressed the latest round of the UN climate meeting in December 2019 held in {Y}, bluntly criticising world leaders for "negotiating loopholes" and using PR to make it appear they are achieving bold climate targets.
But the reluctant celebrity, who has been chased by cameras and attracted large crowds at the {Y} conference, has urged the press to hear from other activists and indigenous youth instead.
The former vice-president Al Gore, a longtime environmentalist, said the magazine made a "brilliant choice", which made Thunberg the youngest person to have ever received the accolade.
"Greta embodies the moral authority of the youth activist movement demanding that we act immediately to solve the climate crisis. She is an inspiration to me and to people across the world," Gore said.
Other nominees for person of the year included Donald Trump and the House speaker, Nancy Pelosi, the top Democrat in Congress.
Q. In the above passage, what has been redacted with {Y}?
Madrid, Spain has been redacted with {Y}. The 25th UN Climate Change Conference took place in Madrid, Spain, and it convened from 2-13 December, 2019. It featured the 25th session of the Conference of the parties (COP 25) to the United Nations Framework Convention to Combat Climate Change (UNFCCC) and the 51st meetings of the UNFCCC subsidiary bodies; the Subsidiary Body for Implementation (SBI 51) and the Subsidiary Body for Scientific and Technological Advice (SBSTA 51).
Read the given passage and answer the question that follows.
Greta Thunberg, the teen activist from {X} who has urged immediate action to address a global climate crisis, was named Time magazine's person of the year for 2019. Thunberg, a 16 year old, was lauded by Time for starting an environmental campaign in August 2018 which became a global movement, initially from skipping school to camping out in front of the country parliament to demand action.
"In the 16 months, she addressed heads of state at the UN, met with the pope, sparred with the president of the United States and inspired 4 million people to join the global climate strike on September 20, 2019, which was the largest climate demonstration in human history," the magazine said.
"Margaret Atwood compared her to Joan of Arc. After noticing a hundredfold increase in its usage, lexicographers at Collins Dictionary named Thunberg's pioneering idea, climate strike, the word of the year," Time said.
Thunberg, who turned 17 in January, 2020 continues to beat the drum, saying in {Y} that the voices of climate strikers are being heard but politicians are still not taking action.
The activist addressed the latest round of the UN climate meeting in December 2019 held in {Y}, bluntly criticising world leaders for "negotiating loopholes" and using PR to make it appear they are achieving bold climate targets.
But the reluctant celebrity, who has been chased by cameras and attracted large crowds at the {Y} conference, has urged the press to hear from other activists and indigenous youth instead.
The former vice-president Al Gore, a longtime environmentalist, said the magazine made a "brilliant choice", which made Thunberg the youngest person to have ever received the accolade.
"Greta embodies the moral authority of the youth activist movement demanding that we act immediately to solve the climate crisis. She is an inspiration to me and to people across the world," Gore said.
Other nominees for person of the year included Donald Trump and the House speaker, Nancy Pelosi, the top Democrat in Congress.
Q. Consider the following statements and mark the correct option.
Statement I: Malala Yousafzai is the youngest person to have received the TIME person of the year award till 2020.
Statement II: Mahatma Gandhi is the only Indian to have won TIME person of the year award till 2020.
Statement I is incorrect as Greta Thunberg, is the youngest person to have received the TIME person of the year award till 2020.
Statement II is correct.
Mahatma Gandhi is the only Indian to be named TIME Person of the Year, having been given the honour in 1930. The magazine described him as 'Saint Gandhi' and later named him as one of the 25 Political Icons of all time.
Read the given passage and answer the question that follows.
Greta Thunberg, the teen activist from {X} who has urged immediate action to address a global climate crisis, was named Time magazine's person of the year for 2019. Thunberg, a 16 year old, was lauded by Time for starting an environmental campaign in August 2018 which became a global movement, initially from skipping school to camping out in front of the country parliament to demand action.
"In the 16 months, she addressed heads of state at the UN, met with the pope, sparred with the president of the United States and inspired 4 million people to join the global climate strike on September 20, 2019, which was the largest climate demonstration in human history," the magazine said.
"Margaret Atwood compared her to Joan of Arc. After noticing a hundredfold increase in its usage, lexicographers at Collins Dictionary named Thunberg's pioneering idea, climate strike, the word of the year," Time said.
Thunberg, who turned 17 in January, 2020 continues to beat the drum, saying in {Y} that the voices of climate strikers are being heard but politicians are still not taking action.
The activist addressed the latest round of the UN climate meeting in December 2019 held in {Y}, bluntly criticising world leaders for "negotiating loopholes" and using PR to make it appear they are achieving bold climate targets.
But the reluctant celebrity, who has been chased by cameras and attracted large crowds at the {Y} conference, has urged the press to hear from other activists and indigenous youth instead.
The former vice-president Al Gore, a longtime environmentalist, said the magazine made a "brilliant choice", which made Thunberg the youngest person to have ever received the accolade.
"Greta embodies the moral authority of the youth activist movement demanding that we act immediately to solve the climate crisis. She is an inspiration to me and to people across the world," Gore said.
Other nominees for person of the year included Donald Trump and the House speaker, Nancy Pelosi, the top Democrat in Congress.
Q. Activist Greta Thunberg has been awarded with which organisation's Ambassador of Conscience Award 2019?
Climate change activist Greta Thunberg and the Fridays for Future movement of school children have been honoured with Amnesty International's Ambassador of Conscience Award for 2019. Amnesty International is a non-governmental organisation with its headquarters in the United Kingdom focused on human rights. The organisation claims it has more than eight million members and supporters around the world.
Read the given passage and answer the question that follows.
Greta Thunberg, the teen activist from {X} who has urged immediate action to address a global climate crisis, was named Time magazine's person of the year for 2019. Thunberg, a 16 year old, was lauded by Time for starting an environmental campaign in August 2018 which became a global movement, initially from skipping school to camping out in front of the country parliament to demand action.
"In the 16 months, she addressed heads of state at the UN, met with the pope, sparred with the president of the United States and inspired 4 million people to join the global climate strike on September 20, 2019, which was the largest climate demonstration in human history," the magazine said.
"Margaret Atwood compared her to Joan of Arc. After noticing a hundredfold increase in its usage, lexicographers at Collins Dictionary named Thunberg's pioneering idea, climate strike, the word of the year," Time said.
Thunberg, who turned 17 in January, 2020 continues to beat the drum, saying in {Y} that the voices of climate strikers are being heard but politicians are still not taking action.
The activist addressed the latest round of the UN climate meeting in December 2019 held in {Y}, bluntly criticising world leaders for "negotiating loopholes" and using PR to make it appear they are achieving bold climate targets.
But the reluctant celebrity, who has been chased by cameras and attracted large crowds at the {Y} conference, has urged the press to hear from other activists and indigenous youth instead.
The former vice-president Al Gore, a longtime environmentalist, said the magazine made a "brilliant choice", which made Thunberg the youngest person to have ever received the accolade.
"Greta embodies the moral authority of the youth activist movement demanding that we act immediately to solve the climate crisis. She is an inspiration to me and to people across the world," Gore said.
Other nominees for person of the year included Donald Trump and the House speaker, Nancy Pelosi, the top Democrat in Congress.
Q. For which movement was Greta Thunberg honoured with the Ambassador of Conscience Award 2019?
Fridays for Future movement of school children has been honoured with Amnesty International's Ambassador of Conscience Award 2019. The school strike for the climate also known variously as Fridays for Future (FFF), Youth for Climate, Climate Strike or Youth Strike for Climate, is an international movement of school students who take time off from class on Fridays to participate in demonstrations to demand action from political leaders to take action to prevent climate change and for the fossil fuel industry to transition to renewable energy.
Read the given passage and answer the question that follows.
The [1] recently confirmed that malware was detected at state-run [2] in September. Investigations were carried out by the Computer & Information Security Advisory Group (CISAG) – DAE along with the national agency, Indian Computer Emergency Response Team (CERT-In). The investigation concluded that the malware infection was limited to the administrative network of KKNPP. The affected system contains data related to day to day administrative functions. The malware infection was not able to get access to the controls of the Nuclear Power Plant. As plant control and instrumentation systems are not connected to any external network such as Intranet, Internet and administrative system.
Q. What should replace [1]?
Read the given passage and answer the question that follows.
The [1] recently confirmed that malware was detected at state-run [2] in September. Investigations were carried out by the Computer & Information Security Advisory Group (CISAG) – DAE along with the national agency, Indian Computer Emergency Response Team (CERT-In). The investigation concluded that the malware infection was limited to the administrative network of KKNPP. The affected system contains data related to day to day administrative functions. The malware infection was not able to get access to the controls of the Nuclear Power Plant. As plant control and instrumentation systems are not connected to any external network such as Intranet, Internet and administrative system.
Q. What should replace [2]?
Read the given passage and answer the question that follows.
The [1] recently confirmed that malware was detected at state-run [2] in September. Investigations were carried out by the Computer & Information Security Advisory Group (CISAG) – DAE along with the national agency, Indian Computer Emergency Response Team (CERT-In). The investigation concluded that the malware infection was limited to the administrative network of KKNPP. The affected system contains data related to day to day administrative functions. The malware infection was not able to get access to the controls of the Nuclear Power Plant. As plant control and instrumentation systems are not connected to any external network such as Intranet, Internet and administrative system.
Q. Which country has helped India to develop a nuclear power plant as mentioned in [2]?
Read the given passage and answer the question that follows.
The [1] recently confirmed that malware was detected at state-run [2] in September. Investigations were carried out by the Computer & Information Security Advisory Group (CISAG) – DAE along with the national agency, Indian Computer Emergency Response Team (CERT-In). The investigation concluded that the malware infection was limited to the administrative network of KKNPP. The affected system contains data related to day to day administrative functions. The malware infection was not able to get access to the controls of the Nuclear Power Plant. As plant control and instrumentation systems are not connected to any external network such as Intranet, Internet and administrative system.
Q. Which is the largest nuclear power plant in India?
Read the given passage and answer the question that follows.
The [1] recently confirmed that malware was detected at state-run [2] in September. Investigations were carried out by the Computer & Information Security Advisory Group (CISAG) – DAE along with the national agency, Indian Computer Emergency Response Team (CERT-In). The investigation concluded that the malware infection was limited to the administrative network of KKNPP. The affected system contains data related to day to day administrative functions. The malware infection was not able to get access to the controls of the Nuclear Power Plant. As plant control and instrumentation systems are not connected to any external network such as Intranet, Internet and administrative system.
Q. Which of the following is used as moderator in Nuclear reactors?
A is the correct option.A graphite reactor is a nuclear reactor that uses carbon as a neutron moderator, which allows un-enriched uranium to be used as nuclear fuel. The first artificial nuclear reactor, the Chicago Pile-1, used graphite as a moderator.
Read the given passage and answer the question that follows.
The current trend in crude oil prices gives a serious cause for concern and if this persists, many of the calculations indicating further recovery and improved growth for the economy can be nullified. This year, the oil prices have risen from $70 per barrel to briefly touch $87, falling back somewhat thereafter. Such levels have not been seen since October 2008. That year, oil crossed a historic $140 per barrel. It is the continued upward trend (except for the period of recession in the global economy) through the later part of the last year, which shows no signs of abating. High oil prices, which rein in growth, will be a bad news for India in more than one way. Slower growth will take away some of the buoyancy that revenue collection is now displaying. But India's problem is compounded by the fact that oil prices are not fully passed on and thus result in under-recoveries for the oil-marketing companies.
Q. Which of the following is not in consonance with the contents of the passage?
If the prices increases are not fully passed on, option C may be inevitable. Refer to “But India's problem …………….for the oil-marketing companies”. Hence option A is in consonance with the text. If Oil companies have under recoveries that will lead to financial ruin, they will definitely not be buoyant. Hence option B is also in consonance with the text. The option D is not in consonance with the text because it states that the oil prices are not fully passed on.
Read the given passage and answer the question that follows.
The current trend in crude oil prices gives a serious cause for concern and if this persists, many of the calculations indicating further recovery and improved growth for the economy can be nullified. This year, the oil prices have risen from $70 per barrel to briefly touch $87, falling back somewhat thereafter. Such levels have not been seen since October 2008. That year, oil crossed a historic $140 per barrel. It is the continued upward trend (except for the period of recession in the global economy) through the later part of the last year, which shows no signs of abating. High oil prices, which rein in growth, will be a bad news for India in more than one way. Slower growth will take away some of the buoyancy that revenue collection is now displaying. But India's problem is compounded by the fact that oil prices are not fully passed on and thus result in under-recoveries for the oil-marketing companies.
Q. Which of the following best captures the main idea of the para?
Option (A) leaves out author’s concern for growth of economy and talks just about crude oil and OMCs.
Option (C) is incorrect as it states that growth of economy is already being nullified while author talks about “The current trend in crude oil prices gives a serious cause for concern and if this persists, many of the calculations indicating further recovery and improved growth for the economy can be nullified”
Above point also helps us reach option (C) as the best option. Option (D) is incorrect as it leaves out current crude oil price increase which author takes as basis
Read the given passage and answer the question that follows.
The current trend in crude oil prices gives a serious cause for concern and if this persists, many of the calculations indicating further recovery and improved growth for the economy can be nullified. This year, the oil prices have risen from $70 per barrel to briefly touch $87, falling back somewhat thereafter. Such levels have not been seen since October 2008. That year, oil crossed a historic $140 per barrel. It is the continued upward trend (except for the period of recession in the global economy) through the later part of the last year, which shows no signs of abating. High oil prices, which rein in growth, will be a bad news for India in more than one way. Slower growth will take away some of the buoyancy that revenue collection is now displaying. But India's problem is compounded by the fact that oil prices are not fully passed on and thus result in under-recoveries for the oil-marketing companies.
Q. Which of the following refutes the main idea of para?
Main idea of para is Current trend in crude oil price, if persists, can be detrimental for the economy. Option (A) is incorrect because it’s incomplete, what will be impact on economy is left out. Option (B) reverses causation. It gives oil price as effect instead of cause of slowdown. Option (C) weakens the main idea as it attacks the basis (crude oil price increase) and states global meltdown as reason for slowdown of Indian economy. Option (D) gives an instance, an example supporting main idea Hence is incorrect answer here.
Read the given passage and answer the question that follows.
The current trend in crude oil prices gives a serious cause for concern and if this persists, many of the calculations indicating further recovery and improved growth for the economy can be nullified. This year, the oil prices have risen from $70 per barrel to briefly touch $87, falling back somewhat thereafter. Such levels have not been seen since October 2008. That year, oil crossed a historic $140 per barrel. It is the continued upward trend (except for the period of recession in the global economy) through the later part of the last year, which shows no signs of abating. High oil prices, which rein in growth, will be a bad news for India in more than one way. Slower growth will take away some of the buoyancy that revenue collection is now displaying. But India's problem is compounded by the fact that oil prices are not fully passed on and thus result in under-recoveries for the oil-marketing companies.
Q. Which of the following options best captures the purpose served, in the para, by the first line “The current trend in crude oil prices gives a serious cause for concern and if this persists, many of the calculations indicating further recovery and improved growth for the economy can be nullified”?
Option (A) though correct is incomplete as option talks just about crude oil price while line 1 of para talks about economy too. Option (B) is incorrect as it implies that line 2 is the main thing and line 1 just forms the basis for it. It isn’t that line 1 of para forms basis of line 2, rather line 2 helps elucidate what the author is talking about in line 1. Option (D) is incorrect as it states that growth of economy is already being nullified while author talks about “The current trend in crude oil prices gives a serious cause for concern and if this persists, many of the calculations indicating further recovery and improved growth for the economy can be nullified”
Read the passage below and answer the question.
"Fake News" is a real phenomenon. There are sleazy predators who cook up completely apocryphal news stories and try to get them circulated on social media and on the internet. But it's a terrible pollution of the language for the president to call any article that criticizes him "fake news." If you compare the credibility of the president's statement with those of the mainstream media, there's just no comparison. The president's been called out on lie after lie after lie. And as someone who's often written for the so-called "mainstream media," for all their flaws, they got fact-checkers. They won't let me just write anything that comes into my head, I've got to prove it. So that's the kind of standard that has established the credibility of many of the news media.
And the basis of democracy is that everyone can be criticized, particularly the leaders. We don't have a monarch, a Supreme Leader, or a dictator for life. We've got a person who is temporarily in charge of the government and when he makes an error, it is mandatory for the free press to call it out. To try to delegitimize the press whenever it criticizes the president, it's really the reflex of an autocrat, of a tin-pot dictator in some banana republic and not worthy of a democracy like the United States where the president serves in our pleasure and can be criticized just like anyone else.
Q. Why does the author say that he cannot write anything in the media that "comes into his head"?
The author states a reason why he cannot write just about anything in the media in these lines: 'And as someone who's often written for the so-called "mainstream media," for all their flaws, they got fact-checkers. They won't let me just write anything that comes into my head, I've got to prove it. So that's the kind of standard that has established the credibility of many of the news media.'
Read the passage below and answer the question.
"Fake News" is a real phenomenon. There are sleazy predators who cook up completely apocryphal news stories and try to get them circulated on social media and on the internet. But it's a terrible pollution of the language for the president to call any article that criticizes him "fake news." If you compare the credibility of the president's statement with those of the mainstream media, there's just no comparison. The president's been called out on lie after lie after lie. And as someone who's often written for the so-called "mainstream media," for all their flaws, they got fact-checkers. They won't let me just write anything that comes into my head, I've got to prove it. So that's the kind of standard that has established the credibility of many of the news media.
And the basis of democracy is that everyone can be criticized, particularly the leaders. We don't have a monarch, a Supreme Leader, or a dictator for life. We've got a person who is temporarily in charge of the government and when he makes an error, it is mandatory for the free press to call it out. To try to delegitimize the press whenever it criticizes the president, it's really the reflex of an autocrat, of a tin-pot dictator in some banana republic and not worthy of a democracy like the United States where the president serves in our pleasure and can be criticized just like anyone else.
Q. Which of the following is similar to the way that the author states the president corrupts the use of the term 'fake news'?
The author states that the president calls out anything that criticizes him as fake news. He states this use to be incorrect as fake news is some wrong information published and circulated in the social media which has no true basis. A student labeling his teacher biased just because he thinks he didn't get the marks he deserves is very similar to the president calling out the news that criticizes him as fake. Option 2 is incorrect because there is nothing wrong in a coach calling a referee impartial as a referee is supposed to be impartial. Options 3 and 4 are incorrect because in both the cases the mentor and the publishing house respectively are wrong in their actions so the researcher and novelist are right in calling them out.
Read the passage below and answer the question.
"Fake News" is a real phenomenon. There are sleazy predators who cook up completely apocryphal news stories and try to get them circulated on social media and on the internet. But it's a terrible pollution of the language for the president to call any article that criticizes him "fake news." If you compare the credibility of the president's statement with those of the mainstream media, there's just no comparison. The president's been called out on lie after lie after lie. And as someone who's often written for the so-called "mainstream media," for all their flaws, they got fact-checkers. They won't let me just write anything that comes into my head, I've got to prove it. So that's the kind of standard that has established the credibility of many of the news media.
And the basis of democracy is that everyone can be criticized, particularly the leaders. We don't have a monarch, a Supreme Leader, or a dictator for life. We've got a person who is temporarily in charge of the government and when he makes an error, it is mandatory for the free press to call it out. To try to delegitimize the press whenever it criticizes the president, it's really the reflex of an autocrat, of a tin-pot dictator in some banana republic and not worthy of a democracy like the United States where the president serves in our pleasure and can be criticized just like anyone else.
Q. Which of the following can be inferred from the author's description of the president?
Both options 1 and 2 can be derived from the author's description of the president. When the author states that the press has the right to call him out when he makes an error, he implies that the president just like any other human can make an error. Option 2 is also correct as he states that there is 'no comparison' between the president's statements and the mainstream media. This implies that the president is less credible than the news media.
Read the passage below and answer the question.
"Fake News" is a real phenomenon. There are sleazy predators who cook up completely apocryphal news stories and try to get them circulated on social media and on the internet. But it's a terrible pollution of the language for the president to call any article that criticizes him "fake news." If you compare the credibility of the president's statement with those of the mainstream media, there's just no comparison. The president's been called out on lie after lie after lie. And as someone who's often written for the so-called "mainstream media," for all their flaws, they got fact-checkers. They won't let me just write anything that comes into my head, I've got to prove it. So that's the kind of standard that has established the credibility of many of the news media.
And the basis of democracy is that everyone can be criticized, particularly the leaders. We don't have a monarch, a Supreme Leader, or a dictator for life. We've got a person who is temporarily in charge of the government and when he makes an error, it is mandatory for the free press to call it out. To try to delegitimize the press whenever it criticizes the president, it's really the reflex of an autocrat, of a tin-pot dictator in some banana republic and not worthy of a democracy like the United States where the president serves in our pleasure and can be criticized just like anyone else.
Q. What role does the author's claim that the even president can be criticised play in the argument in the passage?
The author's claim that the president is no 'monarch, a Supreme Leader, or a dictator' that he cannot be criticized. This supports his original claim as to why the president is wrong to claim any news that criticizes him as fake news. Option 1 is incorrect because there is no 'findings of the news media'.
Read the passage below and answer the question.
"Fake News" is a real phenomenon. There are sleazy predators who cook up completely apocryphal news stories and try to get them circulated on social media and on the internet. But it's a terrible pollution of the language for the president to call any article that criticizes him "fake news." If you compare the credibility of the president's statement with those of the mainstream media, there's just no comparison. The president's been called out on lie after lie after lie. And as someone who's often written for the so-called "mainstream media," for all their flaws, they got fact-checkers. They won't let me just write anything that comes into my head, I've got to prove it. So that's the kind of standard that has established the credibility of many of the news media.
And the basis of democracy is that everyone can be criticized, particularly the leaders. We don't have a monarch, a Supreme Leader, or a dictator for life. We've got a person who is temporarily in charge of the government and when he makes an error, it is mandatory for the free press to call it out. To try to delegitimize the press whenever it criticizes the president, it's really the reflex of an autocrat, of a tin-pot dictator in some banana republic and not worthy of a democracy like the United States where the president serves in our pleasure and can be criticized just like anyone else.
Q. Based on the information in the given passage, which of the following is most likely to be true?
The correct option is 3. All other options run contradictory to what is stated in the text. The author says that the president cannot delegitimize the media when it criticizes him; so option 1 is incorrect. Option 2 is illogical. One cannot 'accuse' someone to be more credible than him. The author states that the president 'can be criticized just like anyone else'; so option 4 is incorrect.
Read the given passage and answer the question that follows.
In each questions given below a statements followed by three courses of action numbered I, II and III. A course of action is a step or administrative decision to be taken for improvement, follow-up or further action in regard to the problem, policy, etc. On the basis of the information given in the statement, you have to assume everything in the statement to be true, then decide which of the three given/suggested courses of action logically follows for pursuing and decide the answer.
Q. Statement: The chairman of the car company announced in the meeting that all trail of its first product the new car model ‘M’ are over and company plans to launch its car in the marked after six months.
Courses of action: I. The network of dealers is to be finalised and all legal, financial and other matters in this connection will have to be finalised shortly.
II. The company will have to make plan for product other than car.
III. Material, managerial and other resources will have to be in fine tune to maintain production schedule.
Once the trials are over, the best availability of material, managerial and other resources in necessary to maintain production schedule. Hence, III follows. As mentioned in the statement, ‘model M is its first product’, so it is necessary to finalise the network of dealers and all matters regarding the sale of the product. Hence, I follows. II has no connection with the statement.
Read the given passage and answer the question that follows.
In each questions given below a statements followed by three courses of action numbered I, II and III. A course of action is a step or administrative decision to be taken for improvement, follow-up or further action in regard to the problem, policy, etc. On the basis of the information given in the statement, you have to assume everything in the statement to be true, then decide which of the three given/suggested courses of action logically follows for pursuing and decide the answer.
Q. Statement: India’s national bird does not have a safe sanctuary in the nation’s capital itself. Last week about a dozen of them were brutally killed in some parts of the city and such killing have been going on for last couple of months.
Course of action: I. Killing of national bird should be declared as treason.
II. Poachers should be banned from entering forests.
III. Nation’s interest should be kept above all and no action should be taken towards such trivial issues.
Action I does not follow became it is an extreme step. It is very difficult to identify who is poacher or not, so action II does not follow. Action III is a negative action which is not admirable.
Read the given passage and answer the question that follows.
In each questions given below a statements followed by three courses of action numbered I, II and III. A course of action is a step or administrative decision to be taken for improvement, follow-up or further action in regard to the problem, policy, etc. On the basis of the information given in the statement, you have to assume everything in the statement to be true, then decide which of the three given/suggested courses of action logically follows for pursuing and decide the answer.
Q. Statement: A blast was triggered off, injuring many, when the night shift workers at an ordinance factory were handling ‘fox signalling explosive’.
Course of action: I. The factory management should train its staff as regards to the safety aspects of handing such explosive materials.
II. The service of the supervisor in change of the night shift should be terminated.
III. The factory should immediately stop carrying out such exercises at night.
Action I follows because training to staff on the safety aspects of handling explosive materials will reduce the chance of such accidents in further. Termination of supervisor incharge is an extreme step. An enquiry should be done against him and if found guilty then he can be terminated. Action III is not advisable as it is a negative action.
Read the passage below and answer the question.
The Supreme Court has taken a timely decision by agreeing to hear a plea from the Election Commission of India (ECI) to direct political parties to not field candidates with criminal antecedents. The immediate provocation is the finding that 46% of Members of Parliament have criminal records. While the number might be inflated as many politicians tend to be charged with relatively minor offences — "unlawful assembly" and "defamation" — the real worry is that the current cohort of Lok Sabha MPs has the highest (29%) proportion of those with serious declared criminal cases compared to its recent predecessors. Researchers have found that such candidates with serious records seem to do well despite their public image, largely due to their ability to finance their own elections and bring substantive resources to their respective parties. Some voters tend to view such candidates through a narrow prism: of being able to represent their interests by hook or by crook. Others do not seek to punish these candidates in instances where they are in contest with other candidates with similar records. Either way, these unhealthy tendencies in the democratic system reflect a poor image of the nature of India's state institutions and the quality of its elected representatives.
Q. If the author's arguments in the given passage are true, which of the following must also be true?
If the author's argument regarding not fielding those candidates with serious criminal charges is true, then the parties should become selective in providing tickets to them. So the correct options is 2. Option 1 is incorrect because 46% is not a majority. Option 3 is incorrect because the voters are already doing that. Option 4 is incorrect as the Supreme Court has already agreed to hear the plea.
Read the passage below and answer the question.
The Supreme Court has taken a timely decision by agreeing to hear a plea from the Election Commission of India (ECI) to direct political parties to not field candidates with criminal antecedents. The immediate provocation is the finding that 46% of Members of Parliament have criminal records. While the number might be inflated as many politicians tend to be charged with relatively minor offences — "unlawful assembly" and "defamation" — the real worry is that the current cohort of Lok Sabha MPs has the highest (29%) proportion of those with serious declared criminal cases compared to its recent predecessors. Researchers have found that such candidates with serious records seem to do well despite their public image, largely due to their ability to finance their own elections and bring substantive resources to their respective parties. Some voters tend to view such candidates through a narrow prism: of being able to represent their interests by hook or by crook. Others do not seek to punish these candidates in instances where they are in contest with other candidates with similar records. Either way, these unhealthy tendencies in the democratic system reflect a poor image of the nature of India's state institutions and the quality of its elected representatives.
Q. Which of the following, if true, most weakens the author's conclusion?
Only option 1 weakens the argument of the author by stating that the ones who are charged with serious criminal records are not guilty until proven in a court of law. 'Appointing special courts', 'Lower house...divided on the issue' and 'Voters...not elect[ing] candidates with dubious credentials' do not weaken the argument.
Read the passage below and answer the question.
The Supreme Court has taken a timely decision by agreeing to hear a plea from the Election Commission of India (ECI) to direct political parties to not field candidates with criminal antecedents. The immediate provocation is the finding that 46% of Members of Parliament have criminal records. While the number might be inflated as many politicians tend to be charged with relatively minor offences — "unlawful assembly" and "defamation" — the real worry is that the current cohort of Lok Sabha MPs has the highest (29%) proportion of those with serious declared criminal cases compared to its recent predecessors. Researchers have found that such candidates with serious records seem to do well despite their public image, largely due to their ability to finance their own elections and bring substantive resources to their respective parties. Some voters tend to view such candidates through a narrow prism: of being able to represent their interests by hook or by crook. Others do not seek to punish these candidates in instances where they are in contest with other candidates with similar records. Either way, these unhealthy tendencies in the democratic system reflect a poor image of the nature of India's state institutions and the quality of its elected representatives.
Q. Which of the following situations is similar in nature to fielding of 'criminal' politicians that the author describes in the passage?
The question is about finding a situation which is similar to parties letting 'criminal' politicians contest in elections. Letting these criminals contest would be a concern which the parties tend to ignore because such candidates 'seem to do well despite their public image, largely due to their ability to finance their own elections and bring substantive resources to their respective parties'. A similar concern is shared in option 4 where the threat to data security does not discourage users from sharing their information online. In other options, this analogy has not been brought out.
Read the passage below and answer the question.
The Supreme Court has taken a timely decision by agreeing to hear a plea from the Election Commission of India (ECI) to direct political parties to not field candidates with criminal antecedents. The immediate provocation is the finding that 46% of Members of Parliament have criminal records. While the number might be inflated as many politicians tend to be charged with relatively minor offences — "unlawful assembly" and "defamation" — the real worry is that the current cohort of Lok Sabha MPs has the highest (29%) proportion of those with serious declared criminal cases compared to its recent predecessors. Researchers have found that such candidates with serious records seem to do well despite their public image, largely due to their ability to finance their own elections and bring substantive resources to their respective parties. Some voters tend to view such candidates through a narrow prism: of being able to represent their interests by hook or by crook. Others do not seek to punish these candidates in instances where they are in contest with other candidates with similar records. Either way, these unhealthy tendencies in the democratic system reflect a poor image of the nature of India's state institutions and the quality of its elected representatives.
Q. Which of the following can we infer from the given passage?
Only option 1 can be inferred from the passage. The author's comment on the Supreme Court's hearing of the ECI's plea as a 'timely decision' suggests that the criminal candidates will not be allowed to contest by their respective parties. It is questionable whether the parties will publicise their candidates' criminal cases. So option 2 is incorrect
Read the given passage and answer the question that follows.
Poverty measurement is an unsettled issue, both conceptually and methodologically. Since poverty is a process as well as an outcome; many come out of it while others may be falling into it. The net effect of these two parallel processes Is a proportion commonly identified as the ‘head count ratio’, but these ratios hide the fundamental dynamism that characterises poverty in practice. The most recent poverty reestimates by an expert group has also missed the crucial dynamism. In a study conducted on 13.000 households which represented the entire country in 1993-94 and again on 2004-05. it was found that in the ten-year period 18.2% rural population moved out of poverty whereas another 22.1% fell into it over this period. This net increase of about four percentage points was seen to have a considerable variation across states and regions.
Q. Which of the following is a conclusion which can be drawn from the facts slated in the above paragraph?
Process of poverty measurement needs to take into account various factors to tackle its dynamic nature.
Read the given passage and answer the question that follows.
Poverty measurement is an unsettled issue, both conceptually and methodologically. Since poverty is a process as well as an outcome; many come out of it while others may be falling into it. The net effect of these two parallel processes Is a proportion commonly identified as the ‘head count ratio’, but these ratios hide the fundamental dynamism that characterises poverty in practice. The most recent poverty reestimates by an expert group has also missed the crucial dynamism. In a study conducted on 13.000 households which represented the entire country in 1993-94 and again on 2004-05. it was found that in the ten-year period 18.2% rural population moved out of poverty whereas another 22.1% fell into it over this period. This net increase of about four percentage points was seen to have a considerable variation across states and regions.
Q. Which of following is an assumption which is implicit in the facts stated in the above paragraph?
It may not be possible to have an accurate poverty measurement in India.
Read the given passage and answer the question that follows.
Poverty measurement is an unsettled issue, both conceptually and methodologically. Since poverty is a process as well as an outcome; many come out of it while others may be falling into it. The net effect of these two parallel processes Is a proportion commonly identified as the ‘head count ratio’, but these ratios hide the fundamental dynamism that characterises poverty in practice. The most recent poverty reestimates by an expert group has also missed the crucial dynamism. In a study conducted on 13.000 households which represented the entire country in 1993-94 and again on 2004-05. it was found that in the ten-year period 18.2% rural population moved out of poverty whereas another 22.1% fell into it over this period. This net increase of about four percentage points was seen to have a considerable variation across states and regions.
Q. Which of the following is an inference which can be made from the facts stated in the above paragraph?
Increase in number of persons falling into poverty varies considerably across the country over a period of time.
Read the passage below and answer the question.
Mars is at the top of the list for two reasons. First, it is relatively close to Earth compared to the moons of Saturn and Jupiter (which are also considered good candidates for discovering life beyond Earth in the solar system, and are targeted for exploration in the coming decade). Second, Mars is extremely observable because it lacks a thick atmosphere like Venus, and so far, there are pretty good evidence that Mars' surface temperature and pressure hovers around the point liquid water -- considered essential for life -- can exist. Further, there is good evidence in the form of observable river deltas, and more recent measurements made on Mars' surface, that liquid water did in fact flow on Mars billions of years ago.
Scientists are becoming increasingly convinced that billions of years Mars was habitable. Whether it was in fact inhabited, or is still inhabited, remains hotly debated. To better constrain these questions, scientists are trying to understand the kinds of water chemistry that could have generated the minerals observed on Mars today, which were produced billions of years ago.
Recent remote measurements on Mars suggest its ancient environments may provide clues about Mars' early habitability. Earth's oceans are of course host to myriad forms of life, thus it seems compelling that Mars' early surface environment was a place contemporary Earth life could have lived, but it remains a mystery as to why evidence of life on Mars is so hard to find.
Q. Which of the following is most consistent with the scientists' reasoning that Mars has been habitable since billions of years?
The text mentions "... scientists are trying to understand the kinds of water chemistry that could have generated the minerals observed on Mars today, which were produced billions of years ago ... Earth's oceans are of course host to myriad forms of life, thus it seems compelling that Mars' early surface environment was a place contemporary Earth life could have lived ..." to suggest the importance of water in establishing the presence of life forms on other planets like Mars. Thus, option 1 is correct.
Read the passage below and answer the question.
Mars is at the top of the list for two reasons. First, it is relatively close to Earth compared to the moons of Saturn and Jupiter (which are also considered good candidates for discovering life beyond Earth in the solar system, and are targeted for exploration in the coming decade). Second, Mars is extremely observable because it lacks a thick atmosphere like Venus, and so far, there are pretty good evidence that Mars' surface temperature and pressure hovers around the point liquid water -- considered essential for life -- can exist. Further, there is good evidence in the form of observable river deltas, and more recent measurements made on Mars' surface, that liquid water did in fact flow on Mars billions of years ago.
Scientists are becoming increasingly convinced that billions of years Mars was habitable. Whether it was in fact inhabited, or is still inhabited, remains hotly debated. To better constrain these questions, scientists are trying to understand the kinds of water chemistry that could have generated the minerals observed on Mars today, which were produced billions of years ago.
Recent remote measurements on Mars suggest its ancient environments may provide clues about Mars' early habitability. Earth's oceans are of course host to myriad forms of life, thus it seems compelling that Mars' early surface environment was a place contemporary Earth life could have lived, but it remains a mystery as to why evidence of life on Mars is so hard to find.
Q. Which of the following regarding Mars can be inferred from the passage?
The context mentions "Whether it was in fact inhabited, or is still inhabited, remains hotly debated ... but it remains a mystery as to why evidence of life on Mars is so hard to find." Thus, the most appropriate answer is option 4.
Option 1 is incorrect because of the above statement.
Option 2 is incorrect because it affirms that the planet is habitable while the passage speculates this.
Option 3 is incorrect because the text uses the term 'relatively close', not 'closest'.
Read the passage below and answer the question.
Mars is at the top of the list for two reasons. First, it is relatively close to Earth compared to the moons of Saturn and Jupiter (which are also considered good candidates for discovering life beyond Earth in the solar system, and are targeted for exploration in the coming decade). Second, Mars is extremely observable because it lacks a thick atmosphere like Venus, and so far, there are pretty good evidence that Mars' surface temperature and pressure hovers around the point liquid water -- considered essential for life -- can exist. Further, there is good evidence in the form of observable river deltas, and more recent measurements made on Mars' surface, that liquid water did in fact flow on Mars billions of years ago.
Scientists are becoming increasingly convinced that billions of years Mars was habitable. Whether it was in fact inhabited, or is still inhabited, remains hotly debated. To better constrain these questions, scientists are trying to understand the kinds of water chemistry that could have generated the minerals observed on Mars today, which were produced billions of years ago.
Recent remote measurements on Mars suggest its ancient environments may provide clues about Mars' early habitability. Earth's oceans are of course host to myriad forms of life, thus it seems compelling that Mars' early surface environment was a place contemporary Earth life could have lived, but it remains a mystery as to why evidence of life on Mars is so hard to find.
Q. The author uses which of the following strategies to present his argument?
The author mentions about Mars and the possibility of life there by stating it to be a popular notion among scientists: "Scientists are becoming increasingly convinced that billions of years Mars was habitable". This is supported by mentioning facts that lead to the conclusion. The most appropriate answer, therefore, is option 3
Read the passage below and answer the question.
Mars is at the top of the list for two reasons. First, it is relatively close to Earth compared to the moons of Saturn and Jupiter (which are also considered good candidates for discovering life beyond Earth in the solar system, and are targeted for exploration in the coming decade). Second, Mars is extremely observable because it lacks a thick atmosphere like Venus, and so far, there are pretty good evidence that Mars' surface temperature and pressure hovers around the point liquid water -- considered essential for life -- can exist. Further, there is good evidence in the form of observable river deltas, and more recent measurements made on Mars' surface, that liquid water did in fact flow on Mars billions of years ago.
Scientists are becoming increasingly convinced that billions of years Mars was habitable. Whether it was in fact inhabited, or is still inhabited, remains hotly debated. To better constrain these questions, scientists are trying to understand the kinds of water chemistry that could have generated the minerals observed on Mars today, which were produced billions of years ago.
Recent remote measurements on Mars suggest its ancient environments may provide clues about Mars' early habitability. Earth's oceans are of course host to myriad forms of life, thus it seems compelling that Mars' early surface environment was a place contemporary Earth life could have lived, but it remains a mystery as to why evidence of life on Mars is so hard to find.
Q. Which of the following, if true, most weakens the author's conclusion?
The text at several places is highly optimistic about the presence of life on Mars. Option 3 which states that there is a threat to life forms on Mars invalidates the author's argument for Mars being habitable. Other options are incorrect as they support the argument.
Read the passage below and answer the question.
Mars is at the top of the list for two reasons. First, it is relatively close to Earth compared to the moons of Saturn and Jupiter (which are also considered good candidates for discovering life beyond Earth in the solar system, and are targeted for exploration in the coming decade). Second, Mars is extremely observable because it lacks a thick atmosphere like Venus, and so far, there are pretty good evidence that Mars' surface temperature and pressure hovers around the point liquid water -- considered essential for life -- can exist. Further, there is good evidence in the form of observable river deltas, and more recent measurements made on Mars' surface, that liquid water did in fact flow on Mars billions of years ago.
Scientists are becoming increasingly convinced that billions of years Mars was habitable. Whether it was in fact inhabited, or is still inhabited, remains hotly debated. To better constrain these questions, scientists are trying to understand the kinds of water chemistry that could have generated the minerals observed on Mars today, which were produced billions of years ago.
Recent remote measurements on Mars suggest its ancient environments may provide clues about Mars' early habitability. Earth's oceans are of course host to myriad forms of life, thus it seems compelling that Mars' early surface environment was a place contemporary Earth life could have lived, but it remains a mystery as to why evidence of life on Mars is so hard to find.
Q. Which of the following must also be true, if the statement in boldface is true?
The boldface statement states that it is compelling for the scientists to believe that Mars has been habitable since billions of years. The reason for this is stated in the line: "Earth's oceans are of course host to myriad forms of life". Thus, the correct answer is option 2.
Read the given passage and answer the question that follows.
The government's version of the Lokpal Bill says that sitting prime ministers will not come under the ombudsman's lens. Now, the Lokpal will have jurisdiction over ministries, but not the prime minister. It's a mistake. He is the head of the Cabinet and the first among equals; as liable to scrutiny as his peers. Ministries and various departments are the functioning arms of the government which enforce rules. The root of corruption lies in how these rules and policies are administered.
Q. Which of the following is the conclusion of the above discussion?
This statement clearly shows the conclusion of the passage that PM can’t be out of the purview of the Lokpal.
Read the given passage and answer the question that follows.
The government's version of the Lokpal Bill says that sitting prime ministers will not come under the ombudsman's lens. Now, the Lokpal will have jurisdiction over ministries, but not the prime minister. It's a mistake. He is the head of the Cabinet and the first among equals; as liable to scrutiny as his peers. Ministries and various departments are the functioning arms of the government which enforce rules. The root of corruption lies in how these rules and policies are administered.
Q. Which of the following can be the assumption behind the above discussion?
This statement shows the hidden assumption being taken in this passage.
Read the given passage and answer the question that follows.
The government's version of the Lokpal Bill says that sitting prime ministers will not come under the ombudsman's lens. Now, the Lokpal will have jurisdiction over ministries, but not the prime minister. It's a mistake. He is the head of the Cabinet and the first among equals; as liable to scrutiny as his peers. Ministries and various departments are the functioning arms of the government which enforce rules. The root of corruption lies in how these rules and policies are administered.
Q. Which of the following is an inference of the above passage?
This statement gives the inference which comes from this passage as not including PM in lokpal is a flaw.
Read the passage below and answer the question.
The study comes as Vermont prepares to implement a mandatory law that makes it illegal to throw food items in the trash beginning July 1, 2020. Several large cities including San Francisco and Seattle have implemented similar policies, but Vermont is the first state to ban household food waste from landfills. The policy is the last phase of a universal state recycling law passed in 2012 that bans all food waste, "blue bin" recyclables and yard debris from landfills state-wide by 2020.
Previous research by Niles and other UVM colleagues showed Americans waste nearly a pound of food daily, roughly one third of a person's recommended daily calories. When disposed of in a landfill, food waste rots and produces methane, a greenhouse gas 25 times more powerful than carbon dioxide over a 100-year period, according to the U.S. Environmental Protection Agency. Conversely, composting can aid in carbon sequestration and creates a natural fertilizer for farms and gardens.
Research has shown the rates of home composting in Vermont are much higher than in other regions. One third of Vermonters indicated they are exclusively composting or feeding food scraps to pets or livestock, with no food scraps ending up in the trash. This research suggests that investing in education, outreach and infrastructure to help households manage their own food waste could have significant environmental and economic impacts in other rural regions seeking food waste management solutions
Q. What is the role played by the second paragraph in relation to the first paragraph?
The first paragraph states the decision taken by Vermont and the second paragraph mentions about "Previous research by Niles and other UVM colleagues showed ..." to provide reason in support of the decision. Thus, the most appropriate answer is option 2.
Read the passage below and answer the question.
The study comes as Vermont prepares to implement a mandatory law that makes it illegal to throw food items in the trash beginning July 1, 2020. Several large cities including San Francisco and Seattle have implemented similar policies, but Vermont is the first state to ban household food waste from landfills. The policy is the last phase of a universal state recycling law passed in 2012 that bans all food waste, "blue bin" recyclables and yard debris from landfills state-wide by 2020.
Previous research by Niles and other UVM colleagues showed Americans waste nearly a pound of food daily, roughly one third of a person's recommended daily calories. When disposed of in a landfill, food waste rots and produces methane, a greenhouse gas 25 times more powerful than carbon dioxide over a 100-year period, according to the U.S. Environmental Protection Agency. Conversely, composting can aid in carbon sequestration and creates a natural fertilizer for farms and gardens.
Research has shown the rates of home composting in Vermont are much higher than in other regions. One third of Vermonters indicated they are exclusively composting or feeding food scraps to pets or livestock, with no food scraps ending up in the trash. This research suggests that investing in education, outreach and infrastructure to help households manage their own food waste could have significant environmental and economic impacts in other rural regions seeking food waste management solutions
Q. Which of the following is most similar to the proposed implementation of a mandatory law against throwing food waste in trash?
The similar situation to the mentioned by researcher in the passage about "Research has shown the rates of home composting in Vermont are much higher than in other regions" which means that people are already taking environment friendly steps.
Read the passage below and answer the question.
The study comes as Vermont prepares to implement a mandatory law that makes it illegal to throw food items in the trash beginning July 1, 2020. Several large cities including San Francisco and Seattle have implemented similar policies, but Vermont is the first state to ban household food waste from landfills. The policy is the last phase of a universal state recycling law passed in 2012 that bans all food waste, "blue bin" recyclables and yard debris from landfills state-wide by 2020.
Previous research by Niles and other UVM colleagues showed Americans waste nearly a pound of food daily, roughly one third of a person's recommended daily calories. When disposed of in a landfill, food waste rots and produces methane, a greenhouse gas 25 times more powerful than carbon dioxide over a 100-year period, according to the U.S. Environmental Protection Agency. Conversely, composting can aid in carbon sequestration and creates a natural fertilizer for farms and gardens.
Research has shown the rates of home composting in Vermont are much higher than in other regions. One third of Vermonters indicated they are exclusively composting or feeding food scraps to pets or livestock, with no food scraps ending up in the trash. This research suggests that investing in education, outreach and infrastructure to help households manage their own food waste could have significant environmental and economic impacts in other rural regions seeking food waste management solutions
Q. Which of the following, if considered true, would suggest that the government's plan would be successful?
The government's plan is to ban dumping of food waste in the landfills. The plan, in order to be successful, needs complete support of the people of Vermont. Thus, it is necessary for the statement mentioned in option 2 to be true for the plan to be successful.
Read the passage below and answer the question.
The study comes as Vermont prepares to implement a mandatory law that makes it illegal to throw food items in the trash beginning July 1, 2020. Several large cities including San Francisco and Seattle have implemented similar policies, but Vermont is the first state to ban household food waste from landfills. The policy is the last phase of a universal state recycling law passed in 2012 that bans all food waste, "blue bin" recyclables and yard debris from landfills state-wide by 2020.
Previous research by Niles and other UVM colleagues showed Americans waste nearly a pound of food daily, roughly one third of a person's recommended daily calories. When disposed of in a landfill, food waste rots and produces methane, a greenhouse gas 25 times more powerful than carbon dioxide over a 100-year period, according to the U.S. Environmental Protection Agency. Conversely, composting can aid in carbon sequestration and creates a natural fertilizer for farms and gardens.
Research has shown the rates of home composting in Vermont are much higher than in other regions. One third of Vermonters indicated they are exclusively composting or feeding food scraps to pets or livestock, with no food scraps ending up in the trash. This research suggests that investing in education, outreach and infrastructure to help households manage their own food waste could have significant environmental and economic impacts in other rural regions seeking food waste management solutions
Q. Which of the following can be inferred from the author's description of the law to prevent dumping of food waste in Vermont?
Both options 1 and 2 can be inferred from the argument. Option 1 can be inferred from: "One third of Vermonters indicated they are exclusively composting or feeding food scraps to pets or livestock, with no food scraps ending up in the trash."
Option 2 can be inferred from author's statement: "research suggests that investing in education, outreach and infrastructure to help households manage their own food waste".
Read the passage below and answer the question.
The study comes as Vermont prepares to implement a mandatory law that makes it illegal to throw food items in the trash beginning July 1, 2020. Several large cities including San Francisco and Seattle have implemented similar policies, but Vermont is the first state to ban household food waste from landfills. The policy is the last phase of a universal state recycling law passed in 2012 that bans all food waste, "blue bin" recyclables and yard debris from landfills state-wide by 2020.
Previous research by Niles and other UVM colleagues showed Americans waste nearly a pound of food daily, roughly one third of a person's recommended daily calories. When disposed of in a landfill, food waste rots and produces methane, a greenhouse gas 25 times more powerful than carbon dioxide over a 100-year period, according to the U.S. Environmental Protection Agency. Conversely, composting can aid in carbon sequestration and creates a natural fertilizer for farms and gardens.
Research has shown the rates of home composting in Vermont are much higher than in other regions. One third of Vermonters indicated they are exclusively composting or feeding food scraps to pets or livestock, with no food scraps ending up in the trash. This research suggests that investing in education, outreach and infrastructure to help households manage their own food waste could have significant environmental and economic impacts in other rural regions seeking food waste management solutions
Q. Which of the following statements is most consistent with the author's description in the passage?
The text mentions "When disposed of in a landfill, food waste rots and produces methane, a greenhouse gas 25 times more powerful than carbon dioxide over a 100-year period ... Conversely, composting can aid in carbon sequestration and creates a natural fertilizer for farms and gardens."
Thus, it can be inferred that composting is more useful than dumping food waste in landfills. Therefore, option 2 is correct.
Read the given passage and answer the question that follows.
The Supreme Court has held that office of the Chief Justice of India is a public authority under the transparency law, the Right to Information Act.
"Transparency doesn’t undermine judicial independence," the Supreme Court said in a unanimous verdict on Wednesday as it upheld the Delhi High Court judgment which ruled that office of the Chief Justice comes under the purview of RTI.
The Supreme Court, however, said that confidentiality and right to privacy have to be maintained and added that RTI can’t be used for as a tool of surveillance. It also said only names of judges recommended by the collegium can be disclosed, not the reasons.
The bench, headed by the chief justice, had wrapped up the hearing, saying nobody wants a "system of opaqueness", but the judiciary cannot be destroyed in the name of transparency.
"Nobody wants to remain in the state of darkness or keep anybody in the state of darkness," it had said. "The question is drawing a line. In the name of transparency, you can't destroy the institution."
The case
The move to bring the office of the CJI under the transparency law was initiated by RTI activist SC Agrawal. His lawyer Prashant Bhushan had submitted in the top court that though the SC should not have been judging its own cause, it is hearing the appeals due to "doctrine of necessity".
The lawyer had described the reluctance of the judiciary in parting information under the Right To Information Act as "unfortunate" and "disturbing", asking: "Do judges inhabit different universe?"
He had submitted that the apex court has always stood for transparency in the functioning of other organs of State, but it develops cold feet when its own issues require attention.
Referring to the RTI provisions, Bhushan had said they also deal with exemptions and information that cannot be given to applicants, but the public interest should always "outweigh" personal interests if the person concerned is holding or about to hold a public office.
Dealing with "judicial independence", he said the National Judicial Accountability Commission Act was struck down for protecting the judiciary against interference from the executive, but this did not mean that judiciary is free from "public scrutiny".
"This is not the independence from accountability. Independence of judiciary means it has to be independent from the executive and not independent from common public. People are entitled to know as to what public authorities are doing," Bhushan had said.
The deliberations of the collegium in appointing and overlooking judges or lawyers should be made public and information can be parted with under RTI on case-to-case basis keeping in mind the larger public interest, the lawyer had said.
Q. What was the primary consideration taken into account by the Supreme Court in giving its decision?
The question asks for the primary consideration taken into account by the SC while deciding the case. As per the rubric of the question, the same has to be decided on the basis of the information in the passage itself, and not from external sources. The correct answer as per the passage is (b), expressed in the first few paragraphs of the extract. (a) is not correct as this was one of the arguments advanced by the petitioner, and not the primary consideration of the Supreme Court itself as per the passage. (c) is wrong as the expression ‘at all costs’ is wrong, considering the Supreme Court has advocated a balance between the right to privacy of judges and transparency. (d), while being a point held by the Court in the case, was not the primary consideration on which the judgment was based.
Read the given passage and answer the question that follows.
The Supreme Court has held that office of the Chief Justice of India is a public authority under the transparency law, the Right to Information Act.
"Transparency doesn’t undermine judicial independence," the Supreme Court said in a unanimous verdict on Wednesday as it upheld the Delhi High Court judgment which ruled that office of the Chief Justice comes under the purview of RTI.
The Supreme Court, however, said that confidentiality and right to privacy have to be maintained and added that RTI can’t be used for as a tool of surveillance. It also said only names of judges recommended by the collegium can be disclosed, not the reasons.
The bench, headed by the chief justice, had wrapped up the hearing, saying nobody wants a "system of opaqueness", but the judiciary cannot be destroyed in the name of transparency.
"Nobody wants to remain in the state of darkness or keep anybody in the state of darkness," it had said. "The question is drawing a line. In the name of transparency, you can't destroy the institution."
The case
The move to bring the office of the CJI under the transparency law was initiated by RTI activist SC Agrawal. His lawyer Prashant Bhushan had submitted in the top court that though the SC should not have been judging its own cause, it is hearing the appeals due to "doctrine of necessity".
The lawyer had described the reluctance of the judiciary in parting information under the Right To Information Act as "unfortunate" and "disturbing", asking: "Do judges inhabit different universe?"
He had submitted that the apex court has always stood for transparency in the functioning of other organs of State, but it develops cold feet when its own issues require attention.
Referring to the RTI provisions, Bhushan had said they also deal with exemptions and information that cannot be given to applicants, but the public interest should always "outweigh" personal interests if the person concerned is holding or about to hold a public office.
Dealing with "judicial independence", he said the National Judicial Accountability Commission Act was struck down for protecting the judiciary against interference from the executive, but this did not mean that judiciary is free from "public scrutiny".
"This is not the independence from accountability. Independence of judiciary means it has to be independent from the executive and not independent from common public. People are entitled to know as to what public authorities are doing," Bhushan had said.
The deliberations of the collegium in appointing and overlooking judges or lawyers should be made public and information can be parted with under RTI on case-to-case basis keeping in mind the larger public interest, the lawyer had said.
Q. With which of the following is the petitioner in the present case least likely to agree?
Based on the passage, the correct answer is (b). The petitioner does not argue that the extension of RTI to the CJI’s office has a negative impact on the independence of the judiciary from the executive. In fact, he impliedly disagrees with the same: he argues that independence from the executive does not imply independence from public scrutiny, thus implying that the latter is distinct from the former, and the former can be achieved without the latter. Thus, he is the least likely to agree with this statement. (a) is wrong as the petitioner argues that the Supreme Court should not have been the judge in this case, but has to be because of the doctrine of necessity. Thus, he agrees with (a). (c) is wrong as the petitioner agrees with it, with the caveat that larger public interest should be kept in mind while making such a disclosure, and it should be done on a case-by-case basis. (d) is wrong as the entire argument of the petitioner is based on this premise, and thus he is very likely to agree with this.
Read the given passage and answer the question that follows.
The Supreme Court has held that office of the Chief Justice of India is a public authority under the transparency law, the Right to Information Act.
"Transparency doesn’t undermine judicial independence," the Supreme Court said in a unanimous verdict on Wednesday as it upheld the Delhi High Court judgment which ruled that office of the Chief Justice comes under the purview of RTI.
The Supreme Court, however, said that confidentiality and right to privacy have to be maintained and added that RTI can’t be used for as a tool of surveillance. It also said only names of judges recommended by the collegium can be disclosed, not the reasons.
The bench, headed by the chief justice, had wrapped up the hearing, saying nobody wants a "system of opaqueness", but the judiciary cannot be destroyed in the name of transparency.
"Nobody wants to remain in the state of darkness or keep anybody in the state of darkness," it had said. "The question is drawing a line. In the name of transparency, you can't destroy the institution."
The case
The move to bring the office of the CJI under the transparency law was initiated by RTI activist SC Agrawal. His lawyer Prashant Bhushan had submitted in the top court that though the SC should not have been judging its own cause, it is hearing the appeals due to "doctrine of necessity".
The lawyer had described the reluctance of the judiciary in parting information under the Right To Information Act as "unfortunate" and "disturbing", asking: "Do judges inhabit different universe?"
He had submitted that the apex court has always stood for transparency in the functioning of other organs of State, but it develops cold feet when its own issues require attention.
Referring to the RTI provisions, Bhushan had said they also deal with exemptions and information that cannot be given to applicants, but the public interest should always "outweigh" personal interests if the person concerned is holding or about to hold a public office.
Dealing with "judicial independence", he said the National Judicial Accountability Commission Act was struck down for protecting the judiciary against interference from the executive, but this did not mean that judiciary is free from "public scrutiny".
"This is not the independence from accountability. Independence of judiciary means it has to be independent from the executive and not independent from common public. People are entitled to know as to what public authorities are doing," Bhushan had said.
The deliberations of the collegium in appointing and overlooking judges or lawyers should be made public and information can be parted with under RTI on case-to-case basis keeping in mind the larger public interest, the lawyer had said.
Q. Section 8(1)(j) of the RTI Act states- “Notwithstanding anything contained in this Act, there shall be no obligation to give any citizen… information which relates to personal information the disclosure of which has no relationship to any public activity or interest, or which would cause unwarranted invasion of the privacy of the individual unless the Central Public Information Officer or the State Public Information Officer or the appellate authority, as the case may be, is satisfied that the larger public interest justifies the disclosure of such information: Provided that the information which cannot be denied to the Parliament or a State Legislature shall not be denied to any person.”
As per this rule, in which of the following cases is the applicant most likely to get information under the RTI Act?
The correct answer is (d) as clear public interest is shown in this situation, on which ground information is likely to be provided. Other options either do not deal with public interest or do not deal with public authorities in the first place.
Read the given passage and answer the question that follows.
The Supreme Court has held that office of the Chief Justice of India is a public authority under the transparency law, the Right to Information Act.
"Transparency doesn’t undermine judicial independence," the Supreme Court said in a unanimous verdict on Wednesday as it upheld the Delhi High Court judgment which ruled that office of the Chief Justice comes under the purview of RTI.
The Supreme Court, however, said that confidentiality and right to privacy have to be maintained and added that RTI can’t be used for as a tool of surveillance. It also said only names of judges recommended by the collegium can be disclosed, not the reasons.
The bench, headed by the chief justice, had wrapped up the hearing, saying nobody wants a "system of opaqueness", but the judiciary cannot be destroyed in the name of transparency.
"Nobody wants to remain in the state of darkness or keep anybody in the state of darkness," it had said. "The question is drawing a line. In the name of transparency, you can't destroy the institution."
The case
The move to bring the office of the CJI under the transparency law was initiated by RTI activist SC Agrawal. His lawyer Prashant Bhushan had submitted in the top court that though the SC should not have been judging its own cause, it is hearing the appeals due to "doctrine of necessity".
The lawyer had described the reluctance of the judiciary in parting information under the Right To Information Act as "unfortunate" and "disturbing", asking: "Do judges inhabit different universe?"
He had submitted that the apex court has always stood for transparency in the functioning of other organs of State, but it develops cold feet when its own issues require attention.
Referring to the RTI provisions, Bhushan had said they also deal with exemptions and information that cannot be given to applicants, but the public interest should always "outweigh" personal interests if the person concerned is holding or about to hold a public office.
Dealing with "judicial independence", he said the National Judicial Accountability Commission Act was struck down for protecting the judiciary against interference from the executive, but this did not mean that judiciary is free from "public scrutiny".
"This is not the independence from accountability. Independence of judiciary means it has to be independent from the executive and not independent from common public. People are entitled to know as to what public authorities are doing," Bhushan had said.
The deliberations of the collegium in appointing and overlooking judges or lawyers should be made public and information can be parted with under RTI on case-to-case basis keeping in mind the larger public interest, the lawyer had said.
Q. As per the petitioner, when should information about the discussions regarding the appointment of judges by the collegium not be withheld?
The correct answer is (a) as stated explicitly in the passage. (b) and (d) go against the passage as the petitioner advocates the release of such information on a case-by-case basis. (c) is wrong as independence of the judiciary is not the ground advocated by the petitioner for judging if the information concerned should be withheld.
Read the given passage and answer the question that follows.
The Supreme Court has held that office of the Chief Justice of India is a public authority under the transparency law, the Right to Information Act.
"Transparency doesn’t undermine judicial independence," the Supreme Court said in a unanimous verdict on Wednesday as it upheld the Delhi High Court judgment which ruled that office of the Chief Justice comes under the purview of RTI.
The Supreme Court, however, said that confidentiality and right to privacy have to be maintained and added that RTI can’t be used for as a tool of surveillance. It also said only names of judges recommended by the collegium can be disclosed, not the reasons.
The bench, headed by the chief justice, had wrapped up the hearing, saying nobody wants a "system of opaqueness", but the judiciary cannot be destroyed in the name of transparency.
"Nobody wants to remain in the state of darkness or keep anybody in the state of darkness," it had said. "The question is drawing a line. In the name of transparency, you can't destroy the institution."
The case
The move to bring the office of the CJI under the transparency law was initiated by RTI activist SC Agrawal. His lawyer Prashant Bhushan had submitted in the top court that though the SC should not have been judging its own cause, it is hearing the appeals due to "doctrine of necessity".
The lawyer had described the reluctance of the judiciary in parting information under the Right To Information Act as "unfortunate" and "disturbing", asking: "Do judges inhabit different universe?"
He had submitted that the apex court has always stood for transparency in the functioning of other organs of State, but it develops cold feet when its own issues require attention.
Referring to the RTI provisions, Bhushan had said they also deal with exemptions and information that cannot be given to applicants, but the public interest should always "outweigh" personal interests if the person concerned is holding or about to hold a public office.
Dealing with "judicial independence", he said the National Judicial Accountability Commission Act was struck down for protecting the judiciary against interference from the executive, but this did not mean that judiciary is free from "public scrutiny".
"This is not the independence from accountability. Independence of judiciary means it has to be independent from the executive and not independent from common public. People are entitled to know as to what public authorities are doing," Bhushan had said.
The deliberations of the collegium in appointing and overlooking judges or lawyers should be made public and information can be parted with under RTI on case-to-case basis keeping in mind the larger public interest, the lawyer had said.
Q. Which was the primary concern of the bench regarding the extension of RTI to the office of the CJI, expressed by the Supreme Court during the hearing?
As per the passage, the Supreme Court earlier stated that the judiciary as an institution cannot be destroyed in the name of transparency. (a) is wrong as this concern of judges being pressurized into resigning has not been stated anywhere in the passage. The same logic is applicable to (b). (d) is wrong as a per the extract, fear of public censure has not been said to lead to ineffectiveness in the functioning of the collegium.
Read the given passage and answer the question that follows.
The question is based on the reasoning and arguments, or facts and principles set out in the passage. Some of these principles may not be true in the real or legal sense, yet you must conclusively assume that they are true for the purpose. Please answer the question on the basis of what is stated or implied in the passage. Do not rely on any principle of law other than the ones supplied to you, and do not assume any facts other than those supplied to you when answering the question. Please choose the option that most accurately and comprehensively answers the question.
Right to Equality is the first fundamental right assured to the people of India. Each citizen of India is guaranteed the Right to Equality by Articles 14 to 18 of the Constitution. Article 14 encapsulates the general standards of equality under the watchful eyes of the law and restricts nonsensical and baseless separation between people. The succeeding articles layout explicit utilisation of the general standards set down in Article 14. This article goes over the Right to Equality in India, covering every one of the articles that this right exemplifies. According to Article 14, it is an obligation of the state to not deny to any person equality before the law or equal protection of the laws within the territory of India. Equality is one of the magnificent corner-stones of the Indian democracy.
Equality before the law or equal protection of the laws does not mean a similar treatment to everybody. As no two individuals are equal in all regards, a similar treatment to them in each regard would bring about unequal treatment.
What Article 14 precludes is class-legislation; however, it doesn't prohibit reasonable classification. The classification, under all circumstances, must not be discretionary or fake or shifty but should be founded on some genuine and significant qualification bearing a fair and reasonable connection to the objective looked to be accomplished by the legislation. Article 14 applies where people who are equal are dealt with contrastingly on no reasonable grounds. In a situation where equals and unequals are treated differently, Article 14 does not come into the picture. As such, this right was considered to be a negative right of an individual not to be discriminated in access to public offices or places or in public matters generally. It did not take account of the existing inequalities arising even from public policies and exercise of public powers. The makers of the Indian Constitution were not satisfied with such type of undertaking. They knew of the widespread social and economic inequalities in the country sanctioned for thousands of years by public policies and exercise of public power supported by religion and other social norms and practices. They were of the opinion that only Article 14 would not be sufficient enough to deal with these inequalities, so they introduced Articles 15-18 which states that nothing shall prevent the State from making any special provision for women and children.
Classification to be sensible must satisfy two conditions, one being that the classification must be established on clear differentia which recognises people or things that are assembled from others which are not part of the group and that the differentiation must have a balanced connection to the article looked to be accomplished by the Act.
Q. The Parliament of India made a law stating that all those who have served imprisonment for 12 years in offences where punishment for life is the highest punishment, shall be entitled to file for mercy petition before the President of India. This was challenged by Ram, who was serving imprisonment for theft, as being discriminatory towards those where punishment for life is not the highest punishment. Decide.
The law is not violative as it makes a difference between a life convict and a convict who is not sentenced to life or has served imprisonment for more than 12 years, where the maximum punishment for the offence for which such person is serving imprisonment is life.
Read the given passage and answer the question that follows.
The question is based on the reasoning and arguments, or facts and principles set out in the passage. Some of these principles may not be true in the real or legal sense, yet you must conclusively assume that they are true for the purpose. Please answer the question on the basis of what is stated or implied in the passage. Do not rely on any principle of law other than the ones supplied to you, and do not assume any facts other than those supplied to you when answering the question. Please choose the option that most accurately and comprehensively answers the question.
Right to Equality is the first fundamental right assured to the people of India. Each citizen of India is guaranteed the Right to Equality by Articles 14 to 18 of the Constitution. Article 14 encapsulates the general standards of equality under the watchful eyes of the law and restricts nonsensical and baseless separation between people. The succeeding articles layout explicit utilisation of the general standards set down in Article 14. This article goes over the Right to Equality in India, covering every one of the articles that this right exemplifies. According to Article 14, it is an obligation of the state to not deny to any person equality before the law or equal protection of the laws within the territory of India. Equality is one of the magnificent corner-stones of the Indian democracy.
Equality before the law or equal protection of the laws does not mean a similar treatment to everybody. As no two individuals are equal in all regards, a similar treatment to them in each regard would bring about unequal treatment.
What Article 14 precludes is class-legislation; however, it doesn't prohibit reasonable classification. The classification, under all circumstances, must not be discretionary or fake or shifty but should be founded on some genuine and significant qualification bearing a fair and reasonable connection to the objective looked to be accomplished by the legislation. Article 14 applies where people who are equal are dealt with contrastingly on no reasonable grounds. In a situation where equals and unequals are treated differently, Article 14 does not come into the picture. As such, this right was considered to be a negative right of an individual not to be discriminated in access to public offices or places or in public matters generally. It did not take account of the existing inequalities arising even from public policies and exercise of public powers. The makers of the Indian Constitution were not satisfied with such type of undertaking. They knew of the widespread social and economic inequalities in the country sanctioned for thousands of years by public policies and exercise of public power supported by religion and other social norms and practices. They were of the opinion that only Article 14 would not be sufficient enough to deal with these inequalities, so they introduced Articles 15-18 which states that nothing shall prevent the State from making any special provision for women and children.
Classification to be sensible must satisfy two conditions, one being that the classification must be established on clear differentia which recognises people or things that are assembled from others which are not part of the group and that the differentiation must have a balanced connection to the article looked to be accomplished by the Act.
Q. The Parliament made a law relating to district-wise seat distribution in the state medical college entrance exam, according to the proportion of the population in a district to the total population of the state. This was challenged as violative of Article 14. Decide.
The provision is violative of the said Article 14 as any scheme of admission should be devised to select the best available talent for admission because it is considered discriminatory to select a less talented candidate against a talented candidate just on the basis of population. The district-wise seat distribution doesn't meet the objective as said in the passage.
Read the given passage and answer the question that follows.
The question is based on the reasoning and arguments, or facts and principles set out in the passage. Some of these principles may not be true in the real or legal sense, yet you must conclusively assume that they are true for the purpose. Please answer the question on the basis of what is stated or implied in the passage. Do not rely on any principle of law other than the ones supplied to you, and do not assume any facts other than those supplied to you when answering the question. Please choose the option that most accurately and comprehensively answers the question.
Right to Equality is the first fundamental right assured to the people of India. Each citizen of India is guaranteed the Right to Equality by Articles 14 to 18 of the Constitution. Article 14 encapsulates the general standards of equality under the watchful eyes of the law and restricts nonsensical and baseless separation between people. The succeeding articles layout explicit utilisation of the general standards set down in Article 14. This article goes over the Right to Equality in India, covering every one of the articles that this right exemplifies. According to Article 14, it is an obligation of the state to not deny to any person equality before the law or equal protection of the laws within the territory of India. Equality is one of the magnificent corner-stones of the Indian democracy.
Equality before the law or equal protection of the laws does not mean a similar treatment to everybody. As no two individuals are equal in all regards, a similar treatment to them in each regard would bring about unequal treatment.
What Article 14 precludes is class-legislation; however, it doesn't prohibit reasonable classification. The classification, under all circumstances, must not be discretionary or fake or shifty but should be founded on some genuine and significant qualification bearing a fair and reasonable connection to the objective looked to be accomplished by the legislation. Article 14 applies where people who are equal are dealt with contrastingly on no reasonable grounds. In a situation where equals and unequals are treated differently, Article 14 does not come into the picture. As such, this right was considered to be a negative right of an individual not to be discriminated in access to public offices or places or in public matters generally. It did not take account of the existing inequalities arising even from public policies and exercise of public powers. The makers of the Indian Constitution were not satisfied with such type of undertaking. They knew of the widespread social and economic inequalities in the country sanctioned for thousands of years by public policies and exercise of public power supported by religion and other social norms and practices. They were of the opinion that only Article 14 would not be sufficient enough to deal with these inequalities, so they introduced Articles 15-18 which states that nothing shall prevent the State from making any special provision for women and children.
Classification to be sensible must satisfy two conditions, one being that the classification must be established on clear differentia which recognises people or things that are assembled from others which are not part of the group and that the differentiation must have a balanced connection to the article looked to be accomplished by the Act.
Q. A law was passed stating that all historically oppressed classes shall have 10% of reservation in government jobs to uplift themselves. This was challenged as discriminatory to other classes of the population. Decide whether the law is a valid law.
The law is valid as it creates a reasonable classification between those who have been historically oppressed and those who have not been oppressed.
Read the given passage and answer the question that follows.
The question is based on the reasoning and arguments, or facts and principles set out in the passage. Some of these principles may not be true in the real or legal sense, yet you must conclusively assume that they are true for the purpose. Please answer the question on the basis of what is stated or implied in the passage. Do not rely on any principle of law other than the ones supplied to you, and do not assume any facts other than those supplied to you when answering the question. Please choose the option that most accurately and comprehensively answers the question.
Right to Equality is the first fundamental right assured to the people of India. Each citizen of India is guaranteed the Right to Equality by Articles 14 to 18 of the Constitution. Article 14 encapsulates the general standards of equality under the watchful eyes of the law and restricts nonsensical and baseless separation between people. The succeeding articles layout explicit utilisation of the general standards set down in Article 14. This article goes over the Right to Equality in India, covering every one of the articles that this right exemplifies. According to Article 14, it is an obligation of the state to not deny to any person equality before the law or equal protection of the laws within the territory of India. Equality is one of the magnificent corner-stones of the Indian democracy.
Equality before the law or equal protection of the laws does not mean a similar treatment to everybody. As no two individuals are equal in all regards, a similar treatment to them in each regard would bring about unequal treatment.
What Article 14 precludes is class-legislation; however, it doesn't prohibit reasonable classification. The classification, under all circumstances, must not be discretionary or fake or shifty but should be founded on some genuine and significant qualification bearing a fair and reasonable connection to the objective looked to be accomplished by the legislation. Article 14 applies where people who are equal are dealt with contrastingly on no reasonable grounds. In a situation where equals and unequals are treated differently, Article 14 does not come into the picture. As such, this right was considered to be a negative right of an individual not to be discriminated in access to public offices or places or in public matters generally. It did not take account of the existing inequalities arising even from public policies and exercise of public powers. The makers of the Indian Constitution were not satisfied with such type of undertaking. They knew of the widespread social and economic inequalities in the country sanctioned for thousands of years by public policies and exercise of public power supported by religion and other social norms and practices. They were of the opinion that only Article 14 would not be sufficient enough to deal with these inequalities, so they introduced Articles 15-18 which states that nothing shall prevent the State from making any special provision for women and children.
Classification to be sensible must satisfy two conditions, one being that the classification must be established on clear differentia which recognises people or things that are assembled from others which are not part of the group and that the differentiation must have a balanced connection to the article looked to be accomplished by the Act.
Q. The Parliament made a law stating that all the silk mills in Karnataka, which make surplus profit, shall pay an extra 10% tax as the state had suffered a severe flood in the previous year and the centre had to pay heavy relief amount to the flood-affected area people. This was challenged as arbitrary. Decide whether it is arbitrary.
The Act is invalid as it targets a particular group of associations without any justifiable reason. Also, levying such tax would discourage the mills from performing well.
Read the given passage and answer the question that follows.
The question is based on the reasoning and arguments, or facts and principles set out in the passage. Some of these principles may not be true in the real or legal sense, yet you must conclusively assume that they are true for the purpose. Please answer the question on the basis of what is stated or implied in the passage. Do not rely on any principle of law other than the ones supplied to you, and do not assume any facts other than those supplied to you when answering the question. Please choose the option that most accurately and comprehensively answers the question.
Right to Equality is the first fundamental right assured to the people of India. Each citizen of India is guaranteed the Right to Equality by Articles 14 to 18 of the Constitution. Article 14 encapsulates the general standards of equality under the watchful eyes of the law and restricts nonsensical and baseless separation between people. The succeeding articles layout explicit utilisation of the general standards set down in Article 14. This article goes over the Right to Equality in India, covering every one of the articles that this right exemplifies. According to Article 14, it is an obligation of the state to not deny to any person equality before the law or equal protection of the laws within the territory of India. Equality is one of the magnificent corner-stones of the Indian democracy.
Equality before the law or equal protection of the laws does not mean a similar treatment to everybody. As no two individuals are equal in all regards, a similar treatment to them in each regard would bring about unequal treatment.
What Article 14 precludes is class-legislation; however, it doesn't prohibit reasonable classification. The classification, under all circumstances, must not be discretionary or fake or shifty but should be founded on some genuine and significant qualification bearing a fair and reasonable connection to the objective looked to be accomplished by the legislation. Article 14 applies where people who are equal are dealt with contrastingly on no reasonable grounds. In a situation where equals and unequals are treated differently, Article 14 does not come into the picture. As such, this right was considered to be a negative right of an individual not to be discriminated in access to public offices or places or in public matters generally. It did not take account of the existing inequalities arising even from public policies and exercise of public powers. The makers of the Indian Constitution were not satisfied with such type of undertaking. They knew of the widespread social and economic inequalities in the country sanctioned for thousands of years by public policies and exercise of public power supported by religion and other social norms and practices. They were of the opinion that only Article 14 would not be sufficient enough to deal with these inequalities, so they introduced Articles 15-18 which states that nothing shall prevent the State from making any special provision for women and children.
Classification to be sensible must satisfy two conditions, one being that the classification must be established on clear differentia which recognises people or things that are assembled from others which are not part of the group and that the differentiation must have a balanced connection to the article looked to be accomplished by the Act.
Q. The Parliament passed a Bill stating that all females shall be entitled to a sum of Rs. 10,000 from their husband for their basic necessities. Any breach of this law shall invite a punishment up to 10 years of imprisonment. Decide whether such law is justified.
The law lacks any intelligible differentia and is therefore struck by Article 14. The words 'all females' will also include kids which shall make the law unreasonable for effective implementation.
Read the given passage and answer the question that follows.
The following passage contains an author’s opinion of an order passed by a judge of the Supreme Court in a case. Read the passage below and based on the same and the information provided in the question if any, answer the questions that follow-
(1) The time-honored principle followed across several jurisdictions is that the perception of the litigant is the material factor for determining judicial bias. What is material is not the actual existence of bias; but the reasonable apprehension in the mind of the litigant regarding the likelihood of bias.
(2) Inverting this well settled principle, Justice Arun Mishra stated that the test should be based on the self-perception of the very same judge, whom the party is apprehending to be biased. In other words, self-attestation by the judge is sufficient.
(3) It appears the actual controversy was not properly acknowledged by Justice Mishra's order. This is clear from the question framed in paragraph 11:
(4) The first question before us is whether a Judge who has expressed an opinion in a smaller Bench and the case has been referred to a larger Bench, because of the conflict of the opinion or otherwise, can hear the matter in a larger bench.
(5) This is a highly polished, sanitized version of the real question, which ought to have been - whether a judge, who played a central role in creating the judicial controversy regarding Section 24(2) of the new Land Acquisition Act by first doubting a three year old precedent and later overruling that precedent as per incuriam (ignoring the principle of stare decisis), should head a constitution bench formed to decide the correctness of the view expressed by him.
Brief History of the Case
(6) The underlying issue, in broad terms, was whether deposit of compensation by government in treasury can be deemed as payment to landowner as per Section 24(2) of the 2013 Land Acquisition Act so as to save the proceedings taken under the 1894 Land Acquisition Act from being lapsed.
(7) In 2014, a three judge bench in Pune Municipal Corporation case held that in case land owners are not willing to accept the compensation, the same has to be deposited in Court. Mere deposit of compensation in treasury cannot be regarded as payment as per Section 24(2), added the bench.
(8) This view held field for nearly over three years, until a two judges bench headed by Justice Arun Mishra doubted its correctness in December 2017 and referred it to larger bench.
(9) The larger bench (a three judge bench) which considered the reference was notably headed by Justice Arun Mishra. This three judge bench (by 2:1 majority) held the decision in Pune Municipal Corporation to be per incuriam, ignoring the principles of stare decisis and judicial discipline that a bench cannot overrule a decision laid down by another bench of equal strength.
(10) Another three-judge bench took objection to this course adopted by Justice Arun Mishra-led bench, and stayed the operation of Indore Development Authority case.
(11) It was only after this that a two-judge bench headed by Justice Arun Mishra thought it fit to refer the issue to the CJI for determination by a larger bench.
Q. As per the general principle of judicial bias as described by the author, in which of the following situations is judicial bias least likely to be presumed?
In the situation given in option ‘c’, there is no link given between the parties/ their lawyers with the judge himself. Thus, there can be no presumption of judicial bias. In option ‘a’, ‘b’ and ‘c’, the judge has some connections with the parties/ their lawyers and there can be a reasonable apprehension in the minds of one of the parties that the judge will be biased against the party concerned.
The following passage contains an author’s opinion of an order passed by a judge of the Supreme Court in a case. Read the passage below and based on the same and the information provided in the question if any, answer the questions that follow-
(1) The time-honored principle followed across several jurisdictions is that the perception of the litigant is the material factor for determining judicial bias. What is material is not the actual existence of bias; but the reasonable apprehension in the mind of the litigant regarding the likelihood of bias.
(2) Inverting this well settled principle, Justice Arun Mishra stated that the test should be based on the self-perception of the very same judge, whom the party is apprehending to be biased. In other words, self-attestation by the judge is sufficient.
(3) It appears the actual controversy was not properly acknowledged by Justice Mishra's order. This is clear from the question framed in paragraph 11:
(4) The first question before us is whether a Judge who has expressed an opinion in a smaller Bench and the case has been referred to a larger Bench, because of the conflict of the opinion or otherwise, can hear the matter in a larger bench.
(5) This is a highly polished, sanitized version of the real question, which ought to have been - whether a judge, who played a central role in creating the judicial controversy regarding Section 24(2) of the new Land Acquisition Act by first doubting a three year old precedent and later overruling that precedent as per incuriam (ignoring the principle of stare decisis), should head a constitution bench formed to decide the correctness of the view expressed by him.
Brief History of the Case
(6) The underlying issue, in broad terms, was whether deposit of compensation by government in treasury can be deemed as payment to landowner as per Section 24(2) of the 2013 Land Acquisition Act so as to save the proceedings taken under the 1894 Land Acquisition Act from being lapsed.
(7) In 2014, a three judge bench in Pune Municipal Corporation case held that in case land owners are not willing to accept the compensation, the same has to be deposited in Court. Mere deposit of compensation in treasury cannot be regarded as payment as per Section 24(2), added the bench.
(8) This view held field for nearly over three years, until a two judges bench headed by Justice Arun Mishra doubted its correctness in December 2017 and referred it to larger bench.
(9) The larger bench (a three judge bench) which considered the reference was notably headed by Justice Arun Mishra. This three judge bench (by 2:1 majority) held the decision in Pune Municipal Corporation to be per incuriam, ignoring the principles of stare decisis and judicial discipline that a bench cannot overrule a decision laid down by another bench of equal strength.
(10) Another three-judge bench took objection to this course adopted by Justice Arun Mishra-led bench, and stayed the operation of Indore Development Authority case.
(11) It was only after this that a two-judge bench headed by Justice Arun Mishra thought it fit to refer the issue to the CJI for determination by a larger bench.
Q. Regarding the test of likelihood of bias, a judge once observed-
"As to the tests of the likelihood of bias what is relevant is the reasonableness of the apprehension in that regard in the mind of the party. The proper approach for the judge is not to look at his own mind and ask himself, however, honestly. "Am I biased? "but to look at the mind of the party before him."
On which of the following points are both the judge who has made this observation and Justice Arun Mishra not likely to disagree?
The correct answer is C as The judge who wrote the paragraph seems to believe that no weight must be assigned to the judge’s own perception of his/her capability to hear the matter. According to him/her, the determining factor is whether the party/parties reasonably apprehend that there might be bias in the decisions of the judge. The self-perception of the judge is not assigned any role. On the other hand, Justice Arun Mishra takes the opposite view and says that the self-perception of the judge plays the decisive role. It cannot be said to have a ‘minor’ role. Thus, both are likely to disagree with the statement.
Read the given passage and answer the question that follows.
The following passage contains an author’s opinion of an order passed by a judge of the Supreme Court in a case. Read the passage below and based on the same and the information provided in the question if any, answer the questions that follow-
(1) The time-honored principle followed across several jurisdictions is that the perception of the litigant is the material factor for determining judicial bias. What is material is not the actual existence of bias; but the reasonable apprehension in the mind of the litigant regarding the likelihood of bias.
(2) Inverting this well settled principle, Justice Arun Mishra stated that the test should be based on the self-perception of the very same judge, whom the party is apprehending to be biased. In other words, self-attestation by the judge is sufficient.
(3) It appears the actual controversy was not properly acknowledged by Justice Mishra's order. This is clear from the question framed in paragraph 11:
(4) The first question before us is whether a Judge who has expressed an opinion in a smaller Bench and the case has been referred to a larger Bench, because of the conflict of the opinion or otherwise, can hear the matter in a larger bench.
(5) This is a highly polished, sanitized version of the real question, which ought to have been - whether a judge, who played a central role in creating the judicial controversy regarding Section 24(2) of the new Land Acquisition Act by first doubting a three year old precedent and later overruling that precedent as per incuriam (ignoring the principle of stare decisis), should head a constitution bench formed to decide the correctness of the view expressed by him.
Brief History of the Case
(6) The underlying issue, in broad terms, was whether deposit of compensation by government in treasury can be deemed as payment to landowner as per Section 24(2) of the 2013 Land Acquisition Act so as to save the proceedings taken under the 1894 Land Acquisition Act from being lapsed.
(7) In 2014, a three judge bench in Pune Municipal Corporation case held that in case land owners are not willing to accept the compensation, the same has to be deposited in Court. Mere deposit of compensation in treasury cannot be regarded as payment as per Section 24(2), added the bench.
(8) This view held field for nearly over three years, until a two judges bench headed by Justice Arun Mishra doubted its correctness in December 2017 and referred it to larger bench.
(9) The larger bench (a three judge bench) which considered the reference was notably headed by Justice Arun Mishra. This three judge bench (by 2:1 majority) held the decision in Pune Municipal Corporation to be per incuriam, ignoring the principles of stare decisis and judicial discipline that a bench cannot overrule a decision laid down by another bench of equal strength.
(10) Another three-judge bench took objection to this course adopted by Justice Arun Mishra-led bench, and stayed the operation of Indore Development Authority case.
(11) It was only after this that a two-judge bench headed by Justice Arun Mishra thought it fit to refer the issue to the CJI for determination by a larger bench.
Q. On what grounds was the decision in the Pune Municipal Corporation case overruled?
In the above passage, the exact grounds for the decision in the Pune Municipal Corporation case being declared per incuriam have not been specified. Thus, the correct answer is ‘d’.
Read the given passage and answer the question that follows.
The following passage contains an author’s opinion of an order passed by a judge of the Supreme Court in a case. Read the passage below and based on the same and the information provided in the question if any, answer the questions that follow-
(1) The time-honored principle followed across several jurisdictions is that the perception of the litigant is the material factor for determining judicial bias. What is material is not the actual existence of bias; but the reasonable apprehension in the mind of the litigant regarding the likelihood of bias.
(2) Inverting this well settled principle, Justice Arun Mishra stated that the test should be based on the self-perception of the very same judge, whom the party is apprehending to be biased. In other words, self-attestation by the judge is sufficient.
(3) It appears the actual controversy was not properly acknowledged by Justice Mishra's order. This is clear from the question framed in paragraph 11:
(4) The first question before us is whether a Judge who has expressed an opinion in a smaller Bench and the case has been referred to a larger Bench, because of the conflict of the opinion or otherwise, can hear the matter in a larger bench.
(5) This is a highly polished, sanitized version of the real question, which ought to have been - whether a judge, who played a central role in creating the judicial controversy regarding Section 24(2) of the new Land Acquisition Act by first doubting a three year old precedent and later overruling that precedent as per incuriam (ignoring the principle of stare decisis), should head a constitution bench formed to decide the correctness of the view expressed by him.
Brief History of the Case
(6) The underlying issue, in broad terms, was whether deposit of compensation by government in treasury can be deemed as payment to landowner as per Section 24(2) of the 2013 Land Acquisition Act so as to save the proceedings taken under the 1894 Land Acquisition Act from being lapsed.
(7) In 2014, a three judge bench in Pune Municipal Corporation case held that in case land owners are not willing to accept the compensation, the same has to be deposited in Court. Mere deposit of compensation in treasury cannot be regarded as payment as per Section 24(2), added the bench.
(8) This view held field for nearly over three years, until a two judges bench headed by Justice Arun Mishra doubted its correctness in December 2017 and referred it to larger bench.
(9) The larger bench (a three judge bench) which considered the reference was notably headed by Justice Arun Mishra. This three judge bench (by 2:1 majority) held the decision in Pune Municipal Corporation to be per incuriam, ignoring the principles of stare decisis and judicial discipline that a bench cannot overrule a decision laid down by another bench of equal strength.
(10) Another three-judge bench took objection to this course adopted by Justice Arun Mishra-led bench, and stayed the operation of Indore Development Authority case.
(11) It was only after this that a two-judge bench headed by Justice Arun Mishra thought it fit to refer the issue to the CJI for determination by a larger bench.
Q. Which of the following is the error which was committed by Justice Arun Mishra in the formulation of the first issue?
The primary concern that the author has regarding the formulation of the issue is that his formulation has ignored the nuances of the present case and has presented a very oversimplified version of what actually happened. That’s why the author has called it “a highly polished, sanitized version of the real question”. The formulation of the issue that has been presented by the author has added the nuances that were missing from Justice Arun Mishra’s version. ‘a’ is not correct as while this is one of the author’s assertions, it is not the error in the formulation of the issue. ‘c’ is wrong as this is not the main concern of the author: though the author has included the name of the Act in its formulation, the main problem with the wording of the issue is that it is “a highly polished, sanitized version of the real question”. ‘d’ is wrong because this is not the author’s concern with the wording of the issue as given in Paragraph 4.
Read the given passage and answer the question that follows.
The following passage contains an author’s opinion of an order passed by a judge of the Supreme Court in a case. Read the passage below and based on the same and the information provided in the question if any, answer the questions that follow-
(1) The time-honored principle followed across several jurisdictions is that the perception of the litigant is the material factor for determining judicial bias. What is material is not the actual existence of bias; but the reasonable apprehension in the mind of the litigant regarding the likelihood of bias.
(2) Inverting this well settled principle, Justice Arun Mishra stated that the test should be based on the self-perception of the very same judge, whom the party is apprehending to be biased. In other words, self-attestation by the judge is sufficient.
(3) It appears the actual controversy was not properly acknowledged by Justice Mishra's order. This is clear from the question framed in paragraph 11:
(4) The first question before us is whether a Judge who has expressed an opinion in a smaller Bench and the case has been referred to a larger Bench, because of the conflict of the opinion or otherwise, can hear the matter in a larger bench.
(5) This is a highly polished, sanitized version of the real question, which ought to have been - whether a judge, who played a central role in creating the judicial controversy regarding Section 24(2) of the new Land Acquisition Act by first doubting a three year old precedent and later overruling that precedent as per incuriam (ignoring the principle of stare decisis), should head a constitution bench formed to decide the correctness of the view expressed by him.
Brief History of the Case
(6) The underlying issue, in broad terms, was whether deposit of compensation by government in treasury can be deemed as payment to landowner as per Section 24(2) of the 2013 Land Acquisition Act so as to save the proceedings taken under the 1894 Land Acquisition Act from being lapsed.
(7) In 2014, a three judge bench in Pune Municipal Corporation case held that in case land owners are not willing to accept the compensation, the same has to be deposited in Court. Mere deposit of compensation in treasury cannot be regarded as payment as per Section 24(2), added the bench.
(8) This view held field for nearly over three years, until a two judges bench headed by Justice Arun Mishra doubted its correctness in December 2017 and referred it to larger bench.
(9) The larger bench (a three judge bench) which considered the reference was notably headed by Justice Arun Mishra. This three judge bench (by 2:1 majority) held the decision in Pune Municipal Corporation to be per incuriam, ignoring the principles of stare decisis and judicial discipline that a bench cannot overrule a decision laid down by another bench of equal strength.
(10) Another three-judge bench took objection to this course adopted by Justice Arun Mishra-led bench, and stayed the operation of Indore Development Authority case.
(11) It was only after this that a two-judge bench headed by Justice Arun Mishra thought it fit to refer the issue to the CJI for determination by a larger bench.
Q. Which of these cases is a violation of the principles of stare decisis and judicial discipline?
The principles of stare decisis and judicial discipline have been mentioned in Paragraph 9. As per the paragraph, a bench cannot overrule a decision laid down by another bench of equal strength. In option ‘c’, the impugned judgement is being declared unconstitutional and thus effectively overruled by a bench of the same strength. Thus, ‘c’ is the correct answer. ‘a’ is not the answer as in this situation, a personal attack is being made on the second judge: the judgment itself has not been overruled as such. In ‘b’, the judgment has merely been referred to a larger bench and not been overruled as such: thus it is not the answer. In ‘d’, the judgment has not been overruled: the cases have been differentiated on facts.
Read the given passage and answer the question that follows.
The question is based on the reasoning and arguments, or facts and principles set out in the passage. Some of these principles may not be true in the real or legal sense, yet you must conclusively assume that they are true for the purpose. Please answer the question on the basis of what is stated or implied in the passage. Do not rely on any principle of law other than the ones supplied to you, and do not assume any facts other than those supplied to you when answering the question. Please choose the option that most accurately and comprehensively answers the question.
The term 'negligence' means the act of being careless and in the legal sense, it signifies the failure to exercise a standard of care which the doer, as a reasonable man, should have exercised in a particular situation. Negligence in English law is emerged as an independent cause of action only in the 18th century. Similarly, the Indian law, the IPC, 1860, contained no provision for causing the death of a person by negligence which was subsequently amended in the year 1870 by inserting Section 304A.
According to Winfield and Jolowicz, negligence is the breach of a legal duty of care by the plaintiff which results in undesired damage to the plaintiff. In general sense, the extent of liability in tort is determined by the number of damages a party has incurred. Consequently, in criminal law, the extent of liability is determined by the amount and degree of negligence.
Negligence can be characterised in three forms: Nonfeasance, means the act of failure to do something which a person should have done; Misfeasance, means the act of not doing an action properly when it should have been done properly; Malfeasance, means the act of doing something which should not have been done in the first place itself.
To commit the tort of negligence, there are primarily six main essentials that are required. An act will be categorised as negligence only if all the conditions are satisfied. Duty of care is one of the essential conditions of negligence in order to make the person liable.
It means that every person owes a duty of care to another person while performing an act. Although this duty exists in all acts, in negligence, the duty is legal in nature and cannot be illegal or unlawful and also cannot be of moral, ethical, or religious nature. A duty arises when the law recognises a relationship between the defendant and the plaintiff, and requires the defendant to act in a certain manner towards the plaintiff. It is not sufficient that the defendant owed a duty of care towards the plaintiff, but it must also be established which is usually determined by the judge. It's not enough for a plaintiff to prove that the defendant owed him a duty of care, but he must also establish that the defendant breached his duty to the plaintiff. A defendant breaches such a duty by failing to exercise reasonable care in fulfilling the duty. The plaintiff who is suing the defendant for negligence has the liability to prove that the defendant's violation of duty was the actual cause of the damages incurred by him. This is often called the "but-for" causation which means that but for the defendant's actions, the plaintiff would not have incurred the damages. Proximate cause means 'legal cause', or the cause that the law recognises as the primary cause of the injury. It may not be the first event that set in motion a sequence of events that led to an injury, and it may not be the very last event before the injury occurs.
Q. Richard carried out a new project in Sunville apartment which was built in 1980s. Sunville apartment being an old building was prone to water leakages and needed cautious repairs. Richard while installing the new fire system used non-insulated wires. Sunville apartment met with a short circuit and the building caught fire. Richard was held liable for the fire. Is Richard liable for the same?
Richard will be held liable for the same as it was Richard's duty to take reasonable care of the installations done by him keeping in mind the present building situation.
It means the act of not doing an action properly when it should have been done properly.
Here, in this case, doing the repairs of an old building by using very poor quality materials creating a major probability of a collapse which injures people, makes Richard liable.
Read the given passage and answer the question that follows.
The question is based on the reasoning and arguments, or facts and principles set out in the passage. Some of these principles may not be true in the real or legal sense, yet you must conclusively assume that they are true for the purpose. Please answer the question on the basis of what is stated or implied in the passage. Do not rely on any principle of law other than the ones supplied to you, and do not assume any facts other than those supplied to you when answering the question. Please choose the option that most accurately and comprehensively answers the question.
The term 'negligence' means the act of being careless and in the legal sense, it signifies the failure to exercise a standard of care which the doer, as a reasonable man, should have exercised in a particular situation. Negligence in English law is emerged as an independent cause of action only in the 18th century. Similarly, the Indian law, the IPC, 1860, contained no provision for causing the death of a person by negligence which was subsequently amended in the year 1870 by inserting Section 304A.
According to Winfield and Jolowicz, negligence is the breach of a legal duty of care by the plaintiff which results in undesired damage to the plaintiff. In general sense, the extent of liability in tort is determined by the number of damages a party has incurred. Consequently, in criminal law, the extent of liability is determined by the amount and degree of negligence.
Negligence can be characterised in three forms: Nonfeasance, means the act of failure to do something which a person should have done; Misfeasance, means the act of not doing an action properly when it should have been done properly; Malfeasance, means the act of doing something which should not have been done in the first place itself.
To commit the tort of negligence, there are primarily six main essentials that are required. An act will be categorised as negligence only if all the conditions are satisfied. Duty of care is one of the essential conditions of negligence in order to make the person liable.
It means that every person owes a duty of care to another person while performing an act. Although this duty exists in all acts, in negligence, the duty is legal in nature and cannot be illegal or unlawful and also cannot be of moral, ethical, or religious nature. A duty arises when the law recognises a relationship between the defendant and the plaintiff, and requires the defendant to act in a certain manner towards the plaintiff. It is not sufficient that the defendant owed a duty of care towards the plaintiff, but it must also be established which is usually determined by the judge. It's not enough for a plaintiff to prove that the defendant owed him a duty of care, but he must also establish that the defendant breached his duty to the plaintiff. A defendant breaches such a duty by failing to exercise reasonable care in fulfilling the duty. The plaintiff who is suing the defendant for negligence has the liability to prove that the defendant's violation of duty was the actual cause of the damages incurred by him. This is often called the "but-for" causation which means that but for the defendant's actions, the plaintiff would not have incurred the damages. Proximate cause means 'legal cause', or the cause that the law recognises as the primary cause of the injury. It may not be the first event that set in motion a sequence of events that led to an injury, and it may not be the very last event before the injury occurs.
Q. Snowy, owner of a big german shepherd dog, requests his cousin Arnold to take care of the dog while he is away. Arnold leaves the dog unattended who attacks a passerby, badly injuring him. Who would be held liable for the negligence?
Arnold will be held liable for the act as due to Arnold's negligence, dog attacked the passerby. Here it will be said that the act occurred due to the negligence of Arnold. In criminal law, the extent of liability is determined by the amount and degree of negligence.
Read the given passage and answer the question that follows.
The question is based on the reasoning and arguments, or facts and principles set out in the passage. Some of these principles may not be true in the real or legal sense, yet you must conclusively assume that they are true for the purpose. Please answer the question on the basis of what is stated or implied in the passage. Do not rely on any principle of law other than the ones supplied to you, and do not assume any facts other than those supplied to you when answering the question. Please choose the option that most accurately and comprehensively answers the question.
The term 'negligence' means the act of being careless and in the legal sense, it signifies the failure to exercise a standard of care which the doer, as a reasonable man, should have exercised in a particular situation. Negligence in English law is emerged as an independent cause of action only in the 18th century. Similarly, the Indian law, the IPC, 1860, contained no provision for causing the death of a person by negligence which was subsequently amended in the year 1870 by inserting Section 304A.
According to Winfield and Jolowicz, negligence is the breach of a legal duty of care by the plaintiff which results in undesired damage to the plaintiff. In general sense, the extent of liability in tort is determined by the number of damages a party has incurred. Consequently, in criminal law, the extent of liability is determined by the amount and degree of negligence.
Negligence can be characterised in three forms: Nonfeasance, means the act of failure to do something which a person should have done; Misfeasance, means the act of not doing an action properly when it should have been done properly; Malfeasance, means the act of doing something which should not have been done in the first place itself.
To commit the tort of negligence, there are primarily six main essentials that are required. An act will be categorised as negligence only if all the conditions are satisfied. Duty of care is one of the essential conditions of negligence in order to make the person liable.
It means that every person owes a duty of care to another person while performing an act. Although this duty exists in all acts, in negligence, the duty is legal in nature and cannot be illegal or unlawful and also cannot be of moral, ethical, or religious nature. A duty arises when the law recognises a relationship between the defendant and the plaintiff, and requires the defendant to act in a certain manner towards the plaintiff. It is not sufficient that the defendant owed a duty of care towards the plaintiff, but it must also be established which is usually determined by the judge. It's not enough for a plaintiff to prove that the defendant owed him a duty of care, but he must also establish that the defendant breached his duty to the plaintiff. A defendant breaches such a duty by failing to exercise reasonable care in fulfilling the duty. The plaintiff who is suing the defendant for negligence has the liability to prove that the defendant's violation of duty was the actual cause of the damages incurred by him. This is often called the "but-for" causation which means that but for the defendant's actions, the plaintiff would not have incurred the damages. Proximate cause means 'legal cause', or the cause that the law recognises as the primary cause of the injury. It may not be the first event that set in motion a sequence of events that led to an injury, and it may not be the very last event before the injury occurs.
Q. Mickey, an interior designer, was engaged to carry out renovation of a house. The interior designer left the house without locking the doors or informing anyone. During her absence, a thief entered the house and stole an antique painting worth 50,000/-. Who would be held liable?
The interior designer would be held liable as she was negligent in leaving the house open and failed her duty of care. Every person owes a duty of care to another person while performing an act and if the person fails to perform such a duty shall be held liable for the tort of negligence.
Read the given passage and answer the question that follows.
The question is based on the reasoning and arguments, or facts and principles set out in the passage. Some of these principles may not be true in the real or legal sense, yet you must conclusively assume that they are true for the purpose. Please answer the question on the basis of what is stated or implied in the passage. Do not rely on any principle of law other than the ones supplied to you, and do not assume any facts other than those supplied to you when answering the question. Please choose the option that most accurately and comprehensively answers the question.
The term 'negligence' means the act of being careless and in the legal sense, it signifies the failure to exercise a standard of care which the doer, as a reasonable man, should have exercised in a particular situation. Negligence in English law is emerged as an independent cause of action only in the 18th century. Similarly, the Indian law, the IPC, 1860, contained no provision for causing the death of a person by negligence which was subsequently amended in the year 1870 by inserting Section 304A.
According to Winfield and Jolowicz, negligence is the breach of a legal duty of care by the plaintiff which results in undesired damage to the plaintiff. In general sense, the extent of liability in tort is determined by the number of damages a party has incurred. Consequently, in criminal law, the extent of liability is determined by the amount and degree of negligence.
Negligence can be characterised in three forms: Nonfeasance, means the act of failure to do something which a person should have done; Misfeasance, means the act of not doing an action properly when it should have been done properly; Malfeasance, means the act of doing something which should not have been done in the first place itself.
To commit the tort of negligence, there are primarily six main essentials that are required. An act will be categorised as negligence only if all the conditions are satisfied. Duty of care is one of the essential conditions of negligence in order to make the person liable.
It means that every person owes a duty of care to another person while performing an act. Although this duty exists in all acts, in negligence, the duty is legal in nature and cannot be illegal or unlawful and also cannot be of moral, ethical, or religious nature. A duty arises when the law recognises a relationship between the defendant and the plaintiff, and requires the defendant to act in a certain manner towards the plaintiff. It is not sufficient that the defendant owed a duty of care towards the plaintiff, but it must also be established which is usually determined by the judge. It's not enough for a plaintiff to prove that the defendant owed him a duty of care, but he must also establish that the defendant breached his duty to the plaintiff. A defendant breaches such a duty by failing to exercise reasonable care in fulfilling the duty. The plaintiff who is suing the defendant for negligence has the liability to prove that the defendant's violation of duty was the actual cause of the damages incurred by him. This is often called the "but-for" causation which means that but for the defendant's actions, the plaintiff would not have incurred the damages. Proximate cause means 'legal cause', or the cause that the law recognises as the primary cause of the injury. It may not be the first event that set in motion a sequence of events that led to an injury, and it may not be the very last event before the injury occurs.
Q. Rita got down from a tram car and while she was being helped in putting her basket on her back, a motor-cyclist after passing the tram collided with a motor car at a distance of 50 metres, which was on the other side of the tram. The motorcyclist died instantly and Rita could not witness the accident or the dead body since the tram was standing between her and the place where the accident occurred. She had only heard the sound of the collision and once the body had been removed from the place of accident, she visited the place and saw some blood which was left on the road. As a reaction to this incident, she suffered a nervous shock and gave birth to a still-born child of 8 months because of which she sued the representatives of the deceased motorcyclist. Will the motor-cyclist be held liable?
The deceased had no duty of care towards the litigant and therefore she could not claim any damages from the deceased's representatives. A duty arises when the law recognises a relationship between the defendant and the plaintiff, and requires the defendant to act in a certain manner towards the plaintiff. It is not sufficient that the defendant owed a duty of care towards the plaintiff.
Read the given passage and answer the question that follows.
The question is based on the reasoning and arguments, or facts and principles set out in the passage. Some of these principles may not be true in the real or legal sense, yet you must conclusively assume that they are true for the purpose. Please answer the question on the basis of what is stated or implied in the passage. Do not rely on any principle of law other than the ones supplied to you, and do not assume any facts other than those supplied to you when answering the question. Please choose the option that most accurately and comprehensively answers the question.
The term 'negligence' means the act of being careless and in the legal sense, it signifies the failure to exercise a standard of care which the doer, as a reasonable man, should have exercised in a particular situation. Negligence in English law is emerged as an independent cause of action only in the 18th century. Similarly, the Indian law, the IPC, 1860, contained no provision for causing the death of a person by negligence which was subsequently amended in the year 1870 by inserting Section 304A.
According to Winfield and Jolowicz, negligence is the breach of a legal duty of care by the plaintiff which results in undesired damage to the plaintiff. In general sense, the extent of liability in tort is determined by the number of damages a party has incurred. Consequently, in criminal law, the extent of liability is determined by the amount and degree of negligence.
Negligence can be characterised in three forms: Nonfeasance, means the act of failure to do something which a person should have done; Misfeasance, means the act of not doing an action properly when it should have been done properly; Malfeasance, means the act of doing something which should not have been done in the first place itself.
To commit the tort of negligence, there are primarily six main essentials that are required. An act will be categorised as negligence only if all the conditions are satisfied. Duty of care is one of the essential conditions of negligence in order to make the person liable.
It means that every person owes a duty of care to another person while performing an act. Although this duty exists in all acts, in negligence, the duty is legal in nature and cannot be illegal or unlawful and also cannot be of moral, ethical, or religious nature. A duty arises when the law recognises a relationship between the defendant and the plaintiff, and requires the defendant to act in a certain manner towards the plaintiff. It is not sufficient that the defendant owed a duty of care towards the plaintiff, but it must also be established which is usually determined by the judge. It's not enough for a plaintiff to prove that the defendant owed him a duty of care, but he must also establish that the defendant breached his duty to the plaintiff. A defendant breaches such a duty by failing to exercise reasonable care in fulfilling the duty. The plaintiff who is suing the defendant for negligence has the liability to prove that the defendant's violation of duty was the actual cause of the damages incurred by him. This is often called the "but-for" causation which means that but for the defendant's actions, the plaintiff would not have incurred the damages. Proximate cause means 'legal cause', or the cause that the law recognises as the primary cause of the injury. It may not be the first event that set in motion a sequence of events that led to an injury, and it may not be the very last event before the injury occurs.
Q. The courier company failed to maintain the compound wall of an office in good condition on the collapse of which the defendant sustained injuries. The courier company was held liable. Was the judgment justified?
It was held that courier company authorities were liable since they had a duty to maintain the office premises and due to their breach of duty to do so, the collapse occurred. Hence, they were liable.
Read the given passage and answer the question that follows.
The Scottish Parliament has passed a law criminalizing physical punishment to children. The bill, titled as Children (Equal Protection from Assault) (Scotland) Bill, intends to abolish the defence of reasonable chastisement, i.e. a common law rule that the physical punishment of a child in the exercise of a parental right or a right derived from having charge or care of the child is justifiable and is therefore not an assault.
Section 51 (physical punishment of children) of the Criminal Justice (Scotland) Act 2003 will also be repealed when the law comes into force. The main provision of the Act will come into force on the expiry of the period of 12 months beginning with the day of Royal Assent.
The Minister for Children and Young People Maree Todd, during the debate in the parliament said, “The removal of the defence of reasonable chastisement will help to ensure that that goal can be achieved. The bill places Scotland in the vanguard in the UK in providing children with the same legal protection from assault as adults. That is the kind of country that I want my children to grow up in. The Scottish Government supports the removal of the defence. It’s very name—reasonable chastisement—is outdated and unconscionable. It suggests that it is sometimes acceptable to hit a child, which is at odds with the Scottish Government's aim of helping children to grow up feeling safe. It is also at odds with the international evidence that shows that the physical punishment of children is harmful and ineffective. In line with that international evidence, many countries have already changed their laws in that area, in ways that are appropriate to their legal systems. By removing the reasonable chastisement defence, we will provide children with the same legal protection from assault as adults. Why would we not want that for our children?”
The minister also clarified that removal of the defence does not impact the ability of a parent to use restraint to prevent their child from coming to harm. "At its heart, restraint is an act of protection. Physical punishment is an act of discipline. They are fundamentally different.” she said.
Section 51 of the 2003 Act limited the common law rule by prohibiting hitting a child with an implement, hitting them on the head, or shaking them. It also put into statute common law principles about the factors a court must have regard to when considering whether an assault on a child, in exercise of a parental right or a right derived from having charge or care of a child, was justifiable.
Q. As per the passage, what is the purpose of rule of “reasonable chastisement”?
As per paragraph 3 of the passage, we can derive that the defence of reasonable chastisement is available to the parents as a justification in case of reasonable force used against the child
Read the given passage and answer the question that follows.
The Scottish Parliament has passed a law criminalizing physical punishment to children. The bill, titled as Children (Equal Protection from Assault) (Scotland) Bill, intends to abolish the defence of reasonable chastisement, i.e. a common law rule that the physical punishment of a child in the exercise of a parental right or a right derived from having charge or care of the child is justifiable and is therefore not an assault.
Section 51 (physical punishment of children) of the Criminal Justice (Scotland) Act 2003 will also be repealed when the law comes into force. The main provision of the Act will come into force on the expiry of the period of 12 months beginning with the day of Royal Assent.
The Minister for Children and Young People Maree Todd, during the debate in the parliament said, “The removal of the defence of reasonable chastisement will help to ensure that that goal can be achieved. The bill places Scotland in the vanguard in the UK in providing children with the same legal protection from assault as adults. That is the kind of country that I want my children to grow up in. The Scottish Government supports the removal of the defence. It’s very name—reasonable chastisement—is outdated and unconscionable. It suggests that it is sometimes acceptable to hit a child, which is at odds with the Scottish Government's aim of helping children to grow up feeling safe. It is also at odds with the international evidence that shows that the physical punishment of children is harmful and ineffective. In line with that international evidence, many countries have already changed their laws in that area, in ways that are appropriate to their legal systems. By removing the reasonable chastisement defence, we will provide children with the same legal protection from assault as adults. Why would we not want that for our children?”
The minister also clarified that removal of the defence does not impact the ability of a parent to use restraint to prevent their child from coming to harm. "At its heart, restraint is an act of protection. Physical punishment is an act of discipline. They are fundamentally different.” she said.
Section 51 of the 2003 Act limited the common law rule by prohibiting hitting a child with an implement, hitting them on the head, or shaking them. It also put into statute common law principles about the factors a court must have regard to when considering whether an assault on a child, in exercise of a parental right or a right derived from having charge or care of a child, was justifiable.
Q. Upon reading the passage, which of the following inferences may be rightly drawn?
(a) Physical punishment is equal to physical assault
(b) It is necessary to physically punish a child for his protection
(c) Physical punishment has usually been justified as a part of parental rights
The passage shows that the law aims to ensure providing children with the same legal protection from assault as adults in respect of physical punishments. It also states that by removing the reasonable chastisement defence, we will provide children with the same legal protection from assault as adults. Further, Children (Equal Protection from Assault) (Scotland) Bill, intends to abolish the defence of reasonable chastisement, i.e. a common law rule that the physical punishment of a child in the exercise of a parental right or a right derived from having charge or care of the child is justifiable and is therefore not an assault.
Read the given passage and answer the question that follows.
The Scottish Parliament has passed a law criminalizing physical punishment to children. The bill, titled as Children (Equal Protection from Assault) (Scotland) Bill, intends to abolish the defence of reasonable chastisement, i.e. a common law rule that the physical punishment of a child in the exercise of a parental right or a right derived from having charge or care of the child is justifiable and is therefore not an assault.
Section 51 (physical punishment of children) of the Criminal Justice (Scotland) Act 2003 will also be repealed when the law comes into force. The main provision of the Act will come into force on the expiry of the period of 12 months beginning with the day of Royal Assent.
The Minister for Children and Young People Maree Todd, during the debate in the parliament said, “The removal of the defence of reasonable chastisement will help to ensure that that goal can be achieved. The bill places Scotland in the vanguard in the UK in providing children with the same legal protection from assault as adults. That is the kind of country that I want my children to grow up in. The Scottish Government supports the removal of the defence. It’s very name—reasonable chastisement—is outdated and unconscionable. It suggests that it is sometimes acceptable to hit a child, which is at odds with the Scottish Government's aim of helping children to grow up feeling safe. It is also at odds with the international evidence that shows that the physical punishment of children is harmful and ineffective. In line with that international evidence, many countries have already changed their laws in that area, in ways that are appropriate to their legal systems. By removing the reasonable chastisement defence, we will provide children with the same legal protection from assault as adults. Why would we not want that for our children?”
The minister also clarified that removal of the defence does not impact the ability of a parent to use restraint to prevent their child from coming to harm. "At its heart, restraint is an act of protection. Physical punishment is an act of discipline. They are fundamentally different.” she said.
Section 51 of the 2003 Act limited the common law rule by prohibiting hitting a child with an implement, hitting them on the head, or shaking them. It also put into statute common law principles about the factors a court must have regard to when considering whether an assault on a child, in exercise of a parental right or a right derived from having charge or care of a child, was justifiable.
Q. If such a law is to be brought in India, then which of the following arguments will not be in support of the law?